SlideShare a Scribd company logo
1 of 59
Download to read offline
MCQ OF NEWBORN
Abdulrahman Bashire
‫اھﺪاء‬
‫ﯾﻧﺎل‬ ‫ان‬ ‫ﷲ‬ ‫ﻣن‬ ‫راﺟﯾﺎ‬ ‫اﻻطﻔﺎل‬ ‫اطﺑﺎء‬ ‫ﺟﻣﯾﻊ‬ ‫اﻟﻰ‬ ‫اﻟﻌﻣل‬ ‫ھذا‬ ‫اﻗدم‬
‫ورﺿﺎﺋﮭم‬ ‫اﻋﺟﺎﺑﮭم‬،‫واﻟﻣﻐﻔرة‬ ‫ﺑﺎﻟرﺣﻣﺔ‬ ‫اﻟدﻋﺎء‬ ‫اﻻ‬ ‫وﻻاطﻠب‬
‫روﺣﻰ‬ ‫وﺗؤام‬ ‫وﻋﺿدى‬ ‫ﻻﺧﻰ‬"‫اﺣﻣد‬".‫ﻟﮫ‬ ‫اﻏﻔر‬ ‫اﻟﻠﮭم‬
‫ﺟﻧﺎﻧك‬ ‫ﻓﺳﯾﺢ‬ ‫واﺳﻛﻧﺔ‬ ‫وارﺣﻣﺔ‬.
‫اﻣﯾن‬
Create PDF files without this message by purchasing novaPDF printer (http://www.novapdf.com)
ABDULRAHMAN BASHIRCHILDREN HOSPITAL –BENGHAZI 1
1) Newborn infants commonly have:-
A. Capillary hemangioma on the forehead.
B. Posterior cranial fontenalle.
C. Metopic sutures.
D. Impalpable coronal sutures.
E. Skin tag in front of the ear.
Ans:- ABC
2) The following should be investigated in five day old baby:-
A. Erythema Toxicum
B. Cloudy cornea
C. Divarication of rectii
D. Subconjunctival hemorrhage
E. Preauricular skin tags
Ans:-BE
3) During morning rounds in the newborn nursery, you examine a healthy infant who has
blotchy erythematous macules that are 2 to 3 cm in diameter. The macules are scattered
over the trunk, face, and proximal extremities; the palms and soles are spared. Each
macule has a 1- to 3-mm central vesicle or pustule.Of the following, the MOST likely
additional finding in this patient is
A. pigmented macules located at sites of resolving pustules
B. presence of lesions at birth
C. pustules coalescing into bullae
D. Wright stain of a smear of the vesicopustular contents revealing a predominance of
eosinophils
E. Wright stain of a smear of the vesicopustular contents revealing a predominance of
polymorphonuclear neutrophils
Preferred Response: D
The infant described in the vignette exhibits the classic presentation of erythema toxicum
neonatorum. Tiny vesicles or pustules arise from blotchy erythematous macules, with
lesions characteristically appearing at 24 to 48 hours after birth. The pustules do not coalesce into
bullae. Wright staining of the pustular contents reveals a predominance of eosinophils, not
neutrophils. Because the lesions are seen in healthy infants, it has been suggested that this benign
condition be renamed; suggested names include “benign neonatal rash” or “benign erythema
neonatorum.”
Transient neonatal pustular melanosis (TNPM) is another well-recognized benign dermal
eruption of infancy in which pustular lesions spontaneously resolve into transient pigmented
macules . TNPM may be present at birth, and examination of pustular contents reveals a
predominance of neutrophils.
Infantile acropustulosis is a chronic or recurring benign condition manifested by intensely
pruritic pustules on hands and feet. Characteristic papules and pustules may coalesce into bullae.
ABDULRAHMAN BASHIRCHILDREN HOSPITAL –BENGHAZI 2
Infantile acropustulosis frequently is confused with scabies infestation. The lesions resolve
spontaneously at 1 to 2 years of age.
4) You are examining a newborn who is the product of an uneventful pregnancy, labor, and
delivery. Apgar scores were 9 at both 1 and 5 minutes. Findings on the initial physical
examination are unremarkable except for the presence of vesicopustules and frecklelike
macules (Item Q33A), some of which have a collarette of surrounding scale. Wright stain
of a smear of the vesicopustular contents reveals a predominance of polymorphonuclear
neutrophils.Of the following, the MOST likely diagnosis is
A. congenital candidiasis
B. erythema toxicum neonatorum
C. infantile acropustulosis
D. miliaria rubra
E. transient neonatal pustular melanosis
Preferred Response: E
Characteristic lesions of transient neonatal pustular melanosis (TNPM) may be present at birth as
vesicles, pustules, or ruptured vesicles or pustules that have a collarette of surrounding scale.
Pigmented macules (Item C33A) often develop at the sites of resolving pustules or vesicles.
Primary lesions usually disappear by 5 days of age; the secondary pigmented lesions may remain
up to 3 months. TNPM occurs more commonly in African-American infants.
Lesions can occur on palms and soles. Pustular contents reveal a predominance of neutrophils on
Wright stain examination, as reported for the newborn in the vignette.
Infants who have congenital cutaneous candidiasis may present with scaling, erythematous
papules and pustules (Item C33B) at birth. Candida albicans can penetrate through the amnion and
chorion to cause congenital infections. Scrapings from lesions prepared with potassium hydroxide
document pseudohyphae (Item C33C) or budding yeast.
Term infants who have erythema toxicum neonatorum exhibit vesicopustular lesions (Item
C33D) that usually overlie erythematous macules. Lesions of erythema toxicum rarely are present
at birth, and Wright stain of smears of pustular contents reveals a predominance of eosinophils.
Infantile acropustulosis presents as pustules or vesicles (Item C33E) localized to the hands
and feet. It may be present at birth but more commonly develops in the first weeks and months
after birth, possibly continuing or recurring throughout infancy and early childhood. Lesions are
very similar to those of infantile scabies infestation. Pustular contents may reveal prominent
neutrophils and occasional eosinophils without evidence of the mites, ova, or feces seen in
scabies. An absence of hyperpigmentation in resolving lesions and a prolonged or recurring
course distinguishes infantile acropustulosis from TNPM.
Miliaria rubra (prickly heat or heat rash) is caused by intraepidermal obstruction of the sweat
ducts. A secondary local inflammatory response is responsible for the erythema (Item C33F)
associated with the papules and vesicles. Miliaria rubra occurs later than miliaria crystallina,
usually beyond thefirst postnatal week. Hyperpigmented, frecklelike lesions are not expected in
miliaria rubra.
5) The following is true regarding changes in the fetal cardiovascular system after birth:
A. There is normally immediate closure of the ductus arteriosus
ABDULRAHMAN BASHIRCHILDREN HOSPITAL –BENGHAZI 3
B. Hypoxia-induced vasoconstriction is the mechanism of closure of the ductus
arteriosus
C. The ligamentum teres is the remnant of the umbilical vein
D. Regression of right ventricular hypertrophy occurs postnatally
E. Inferior vena caval pressure falls after birth
Ans:-CDE
Occlusion of the umbilical cord removes this low resistance capillary bed from the circulation;
breathing results in a marked decrease in pulmonary vascular resistance, hence there is increased
pulmonary blood flow returning to the left atrium raising the pressure in the left atrium causing
the foramen ovale to close. As pressure in the systemic circulation rises, shunt through the ductus
arteriosus reverses. As the pO2 rises, synthesis of bradykinin and prostacyclins is inhibited, thus
causing closure of the ductus arteriosus. The ductus arteriosus can take up to 3 months to close in
normal neonates.
6) Neonatal RDS:-
A. Seen in most babies of birth weight < 2.5 kg.
B. More common in infants of diabetic mothers.
C. Associated with prolonged rupture of membrane.
D. Less sever in babies of Afro-Caribiean origin than Caucasians.
E. Exacerbated by hypothermia.
Ans:-BDE
7) The following are causes of generalized hypotonia in 2 days old infants:-
A. Prematurity
B. Hypothyroidism
C. Myotonic dystrophy
D. Spinal dysraphyism
E. Anterior horn cell disease
Ans:-ACE
8) Regarding surfactant:-
A. Production begins at 30 weeks
B. It is produced by Type II pneumocytes in the walls of the bronchi
C. Testosterone stimulates surfactant production
D. Production is increased during a stressful event like hypothermia
E. Betamathasone given to the mother improves surfactant production in the
premature baby
Ans:- E
Surfactant production begins at 20-22 weeks. It is produced by Type II pneumocytes which are in
the walls of the alveoli. The hormones testosterone and insulin inhibit surfactant production;
hence hyaline membrane disease is more common in males than females and more common in
infants of diabetic mothers. Surfactant production is suppressed if the baby is hypothermic,
hypoxic, acidotic or hypoglycemic.
Although dexamethasone is more commonly used, betamethasone has an identical effect on lung
maturation
9) Concerning fetal lung development:-
ABDULRAHMAN BASHIRCHILDREN HOSPITAL –BENGHAZI 4
A. Type ΙΙ pneumocytes are present at 24 week gestation
B. Cuboidal cells are capable of gas transfer in utero
C. There is virtually no smooth muscle in the terminal & respiratory bronchioles at
6 month of age
D. The large airways are formed at 16 week gestation
E. Alveoli are completely formed at birth
Ans:-AD
10) Congenital CMV infection:-
A. Only 10% of affected pregnancies have resulting long term sequel at birth
B. Diagnosis is by viral isolation from the urine
C. Hearing loss can develop gradually over the first 5 years
D. The affected newborn should be treated with ganciclovir
E. Intracranial calcifications are seen in a periventricular distribution
Ans:- ABCE
Congenital CMV occurs in approximately 1% of all live births and only 10% of these infections
result in clinical symptoms. Severe clinical disease is associated with primary maternal infection
in pregnancy. Infection in early gestation carries a far greater risk of severe fetal disease. In CMV
intra-cranial calcifications are in a periventricular distribution. Ganciclovir is only used if there is
CNS involvement, chorioretinitis or pneumonitis.
11) The following conditions will present with cyanosis in the first week of life:
A. Aortic stenosis
B. Transposition of the great vessels
C. Hypoplastic left heart syndrome
D. Fallot's tetralogy
E. Fallot's pentalogy
Ans:- B
Any cardiac lesion which allows a mixing of blood along with a right to left flow or any cardiac
lesion wherein pulmonary perfusion is impaired results in cyanosis. Left heart problems or
outflow tract obstructions present as cardiac failure. Fallot's pentalogy includes an ASD along
with the tetrad of infundibular pulmonary stenosis, RVH, over-riding of the aorta and a VSD.
Babies with tetralogy of Fallot usually have a patent ductus arteriosus at birth that provides
additional pulmonary blood flow, so severe cyanosis is rare early after birth.
As the ductus arteriosus closes, as it typically will in the first days of life, cyanosis can develop or
become more severe.
The degree of cyanosis is proportional to lung blood flow and thus depends upon the degree of
narrowing of the outflow tract to the pulmonary arteries.
12) Pulmonary surfactant
A. Is partly recycled by endocytosis into the synthesizing cell
B. Is produced by type Ι alveolar cells
C. Reduction in pulmonary flow can cause a decrease in surfactant production
D. Synthesis is inhibited by thyroxine
E. Synthesis is stimulated by glucocorticoids
Ans:- ACE
ABDULRAHMAN BASHIRCHILDREN HOSPITAL –BENGHAZI 5
Dipalmityl- phosphotidyl choline is the main component of surfactant and is produced by Type-ΙΙ
alveolar cells (granular pneumocytes). Its half-life is 14 hours and its main function is to reduce
the surface tension of the alveoli.
13) Lung surfactant
A. Decreases the surface tension within an alveolus
B. Causes an increase in chest wall compliance
C. Is a glycoprotein
D. Maintains the same surface tension for different sized alveoli
E. Appears only after the 1st week of life
Ans:- A
Surfactant is a dipalmitoyl-phosphatidyl choline and is a phospholipid, which prevents alveolar
collapse by reducing alveoli surface tension. It is produced by type-II pneumocytes and is seen at
about 24 weeks gestation. It causes an increase in lung compliance only (not chest wall
compliance).
14) The following organisms cause conjunctivitis:-
A. Epstein Barr virus
B. Chlamydia trachomatis
C. Adenovirus
D. Haemophilus influenzae
E. Neisseria gonorrhoeae
Ans:- BCDE
Chlamydia trachomatis causes conjunctivitis in 30-50% of neonates born to mothers with
cervicitis. It is a purulent conjunctivitis, which develops 5-14 days after birth and is
indistinguishable from gonococcal infection. It is diagnosed on a swab scraped over the lower
eyelid (to allow cells to be collected – don’t forget it is an intracellular organism) by direct
fluorescent antibody, ELISA or PCR. Tetracycline ointment topically is combined with oral
erythromycin – the oral antibiotic is to prevent relapse after ointment is discontinued and to
prevent progression to pneumonia. Gonococcal conjunctivitis presents earlier than chlamydial
disease (usually within 2 days), is diagnosed on gram stain and culture and should be treated with
IV penicillin and chloramphenicol eye drops. Don’t forget sexual health screening for the mother
and informing public health of ophthalmia neonatorum. Adenovirus causes conjunctivitis in
summer outbreaks; enterovirus, coxsackie and herpes simplex are other viral causes.
15) Concerning blood flow in the fetus:-
A. Blood flow from right to left through the foramen ovale
B. Blood in the ascending aorta has higher oxygen content than in the descending
aorta
C. The ductus arteriosus is closed
D. Pulmonary pressure equal systemic pressure
E. Hemoglobin may be 20 gm/dl
Ans:-ABE
16) -In a healthy baby the transition from fetal to neonatal circulation involves:-
A. Functional closure of the foramen ovale in the first 24 hours
ABDULRAHMAN BASHIRCHILDREN HOSPITAL –BENGHAZI 6
B. Blood flow in the ductus arteriosus continues from right to left until its closure
C. Decrease in pulmonary artery resistance following closure of the ductus arteriosus
D. The ductus arteriosus closes in response to decreased oxygen concentrations
E. The umbilical artery is a branch of the common iliac artery
Ans:- A
Functional closure of the ductus arteriosus occurs soon after birth but anatomical closure can take
upto one week. As pulmonary pressures fall after birth, blood flow in the ductus is reversed ie
from left to right. The umbilical artery is a branch of the internal iliac artery.
Factors influencing closure of the ductus include:-
1. Increased oxygen concentrations
2. Decreased prostaglandin levels
3. Drop in pulmonary artery pressures
N.B. Prostaglandin E2 keeps the ductus open.
17) A 10-day old male presents with bilious emesis. What is the most likely diagnosis?
A. Appendicitis
B. Pyloric stenosis
C. Malrotation with midgut volvulus
D. Feeding intolerance
Ans:- C
18) A term newborn is delivered by emergent cesarean section because of intrauterine
growth restriction, oligohydramnios, and nonreassuring fetal heart rate monitoring in
labor. Delivery room resuscitation includes endotracheal intubation and assisted
ventilation with 100% oxygen, chest compressions, intravenous epinephrine, and volume
expansion. Apgar scores are 1, 2, and 3 at 1, 5, and 10 minutes, respectively. An umbilical
cord arterial blood gas measurement documents a pH of 6.9 and a base deficit of 20
mmol/L. At 12 hours of age, the infant demonstrates tonic-clonic convulsive activity of
the arms and legs with a concomitant decrease in heart rate and bedside pulse oximetry
saturation. Of the following, the MOST likely cause for this infant's seizure is:-
A.Hypercalcemia.
B.Hypercarbia.
C.Hyperglycemia.
D.Hypomagnesemia.
E. Hypoxia.
Preferred Response: E
Seizures are the most frequent sign of central nervous system injury in the newborn. When
seizures occur in a newborn who has depressed neuromotor tone, reflexes, and
cardiopulmonary function at birth that requires assisted ventilation, perinatal asphyxia is likely. In
this event, Apgar scores typically are depressed to less than 3 at 5 or more minutes after birth,
and there is a severely acidotic umbilical cord arterial pH (<7.0), with evidence of metabolic
acidemia. Poor tolerance of labor and asphyxia are more common in fetuses that have
experienced intrauterine growth restriction. Because the infant in the vignette has the previously
described features, hypoxic-ischemic encephalopathy (HIE) must be considered as a cause for
the seizures.
HIE is the most common cause of seizures occurring in the first 24 hours of postnatal life
ABDULRAHMAN BASHIRCHILDREN HOSPITAL –BENGHAZI 7
and accounts for up to 67% of early neonatal seizures. Other causes of neonatal seizure
include intracranial hemorrhage, cerebrovascular accidents (stroke), or hemorrhagic infarction
(10% to 15%); intracranial malformation (<10%); transient hypoglycemia or hypocalcemia
(<10%); drug withdrawal (<5%); and inborn errors of metabolism (<5%).
When seizures occur beyond the first 24 hours after birth, especially in the absence of any
history of fetal or neonatal asphyxia, the evaluation should focus on potential causes other than
HIE. An additional cause for later seizures is infection (meningitis, encephalitis).
Asphyxia may result in hypocalcemia and hypoglycemia; hyperglycemia and hypercalcemia
are not associated with HIE and do not typically cause seizures. Hypomagnesemia may
accompany hypocalcemia in the infant of a diabetic mother, but it is not common following
asphyxia and is not associated with neonatal seizures. Hypercarbia may occur in the depressed
newborn who has inadequate ventilation, but it is not associated with seizures unless there is
corresponding hypoxia.
19) A 2-week-old-male presents with lethargy and vomiting. His electrolytes reveal sodium
of 121 meq/L, potassium of 7.0-meq/l and blood glucose of 40 mg/dl. What is the most
likely diagnosis?
A. Dehydration
B. Congenital adrenal hyperplasia
C. Inborn error of metabolism
D. Pyloric stenosis
Ans:- B
20) The following cause the onset of persistent vomiting in 3 week old child:-
A. Disaccharidase intolerance
B. Duodenal atresia
C. Pyloric stenosis
D. Hiatus hernia
E. Lactose intolerance
Ans:-C
21) The drug of choice for treating a newborn with presumed ductal-dependant cyanotic
congenital heart disease is:
A. Morphine
B. Dobutamine
C. Prostaglandin E1
D. Indomethacin
Ans:- C
The drug of choice for ductal dependent cyanotic heart disease is prostaglandin E1- with a
starting infusion of 0.05 - 1ug/kg/min. There is a risk of apnea associated with its use so be
prepared to intubate, other complications include seizures and fever. Patients with congenital
heart disease present with poor feeding, sweating with feeds, tachypnea, sudden onset of
cyanosis or pallor that may worsen with crying, lethargy, or failure to thrive. Patients with
cyanotic congenital heart disease are hypoxic but typically have a minimal response to oxygen
therapy; whereas patients with a pulmonary process causing hypoxia will have an increase in
ABDULRAHMAN BASHIRCHILDREN HOSPITAL –BENGHAZI 8
their oxygen saturation when oxygen is administered. Indomethacin is used to close a patent
ductus.
22) How should a neonate with lethargy and a blood sugar of 20mg/dl be treated?
A. Oral feeds with apple juice
B. 25% dextrose solution
C. 10% dextrose solution
D. 50% dextrose solution
Ans:- C
Newborns with hypoglycemia should be treated with D10W solution with a range of 2-
10cc/kg. Higher concentrations should not be used as they can cause vein sclerosis and
intracranial hemorrhage. Infants and young children should be treated with D25 2-4cc/kg.
23) Neonatal convulsion can be caused by:-
A. Maternal hyperparathyroidism
B. Subdural hematoma
C. Birth asphyxia
D. Hyponatremia
E. Wilson's disease
Ans:-ABCD
24) A 2-day-old female presents with abdominal distension and vomiting. She has not yet
passed a meconium stool. What is the most likely diagnosis?
A. Hirschsprung Disease
B. Malrotation with midgut volvulus
C. Necrotizing enterocolitis
D. Constipation
Ans:-A
Suspect Hirschsprung Disease in a newborn who has not yet passed a meconium stool. Other
possibilities include an imperforate anus or meconium plugging. Older children present with a
history of chronic constipation. Hirschsprung disease is the absence of intramural ganglion cells in
the rectum which extends to the sigmoid colon in 77% of patients and involves the entire colon in
15% of patients. The incidence is 1/5,000 live-births, with a male to female ratio of 4:1. The
diagnosis should be suspected if the patient presents with lack of meconium stool within the first
24 to 48 hours of life. Vomiting and abdominal distension may also be present.
25) A 3-week-old female presents with persistent seizures despite aggressive management
with benzodiazepines and phenobarbital. The mother reports giving her daughter some
water to "stop her from getting dehydrated." What is the most likely cause of her status
epilepticus?
A. Hypoglycemia
B. Diabetes insipidus
C. Hyponatremia
D. Hypokalemi
Ans:-C
ABDULRAHMAN BASHIRCHILDREN HOSPITAL –BENGHAZI 9
Excessive free water intake can result in hyponatremic seizures. Infants less than 6 months of age
are particularly susceptible to these types of seizures and commonly have intractable seizures
requiring intubation and hypothermia. Immediate treatment includes the administration of 3%
saline 4cc/kg.
26) A 1-week-old male presents with some mild erythema around his umbilicus extending
onto the abdominal wall. Which of the following is the correct management for this
patient?
A. Reassurance and continue with alcohol wipes of umbilicus
B. Topical antibiotic ointment and recheck the patient the next day
C. Discharge on cephalexin and recheck the next day
D. Perform a full septic workup and admit the patient
Ans:- D
This patient has Omphalitis and should undergo a full septic evaluation, administration of
antibiotics and hospital admission. Surgical debridement may be required for severe cases.
Omphalitis is inflammation and infection surrounding the umbilicus that can spread to the liver or
peritoneum. Patients can present with symptoms ranging from mild erythema to necrosing lesions
surrounding the umbilicus on the abdominal wall. Fever may be present
27) A 5-day old, well-appearing male is brought to the ED by his mother today because she
noted that he has a cluster of vesicles on his scalp. Which of the following should be the
management approach?
A. Skin biopsy
B. IV acyclovir and a full septic workup
C. Oral acyclovir
D. Discharge, with next day follow up
Ans:-B
This patient is at risk for herpes encephalitis and should undergo a complete septic workup and IV
acyclovir should be initiated in the ED. Begin acyclovir (20mg/kg every 8 hours IV) if there is a
positive maternal history of herpes, a vesicular rash, focal neurologic findings, CSF pleocytosis or
elevated CSF protein without organisms on gram stain.
28) The following maternal condition can cause disease in the fetus/newborn:-
A. Hyperparathyroidism
B. ITP
C. Myasthenia gravis
D. Diabetus mellitus
E. Thyrotoxicosis
Ans:-ABCDE
29) Which of the following heart rates is most suggestive of supraventricular tachycardia in
a newborn?
A. 180 BPM
B. 230 BPM
C. 150 BPM
D. 210 BPM
ABDULRAHMAN BASHIRCHILDREN HOSPITAL –BENGHAZI 10
Ans:-B
PALS defines SVT in infants as a heart rate of greater than 220 BPM. In older children the
heart rate for SVT is greater than 180 BPM. The ECG demonstrates a narrow complex
tachycardia without discernible p waves or beat-to-beat variability. In the stable patient, vagal
maneuvers are the first treatment of choice (ice to the face, or blowing through an occluded
straw in older children). Adenosine given as centrally as possible is the first drug of choice (
0.1mg/kg up to 6mg for the first dose and then 0.2mg/kg for the second dose up to 12 mg) If
this is not successful, then amiodarone 5mg/kg given over 20-60 minutes or procainamide
15mg/kg given over 30-60 minutes are the next drugs of choice. Unstable patients should
undergo cardioversion with 0.5-1J/kg followed by 2J/kg. If an IV is accessible, a dose of
adenosine can be given while setting up for the cardioversion.
30) Vaginal bleeding in a 3-day-old female is:
A. Is always indicative of child abuse
B. May be due to withdrawal of maternal hormones
C. Is suspicious for gonorrhea
D. Is most commonly due to a vaginal foreign body-such as baby wipes
Ans:- B
31) Cyanosis in the first week of life can be caused by:-
A. Tetralogy of Fallot
B. Pulmonary stenosis
C. Eisenmenger syndrome
D. TAPVD
E. Ebistein's anomaly
Ans:-ABDE
32) A 3-week old male presenting to the emergency department with vomiting and altered
mental status and acidosis. What additional laboratory test should be included in your
evaluation?
A. Ammonia level
B. Cortisol level
C. Serum acetone
D. Thyroid function test
Ans:- A
Suspect an inborn error of metabolism in patients who have an altered level of consciousness.
These patients may or may not be acidotic depending on the type of inborn error that is present.
Patients with a urea cycle defect typically have a normal blood gas but an elevated ammonia
level. Patients with organic acidemias will be acidotic but may or may not have an elevated
ammonia level. Patients with galactosemia will have a normal blood gas and ammonia level but
will have reducing substances in the urine.
33) Which of the following are causes of shock in the newborn?
A. Infection
B. Inborn errors of metabolism
C. Child abuse
D. Thyrotoxicosis
ABDULRAHMAN BASHIRCHILDREN HOSPITAL –BENGHAZI 11
E. All of the above
Ans:- E
34) In neonatal RDS (respiratory distress syndrome):
A. Surfactant is useful in the treatment.
B. It is rare in infants below 28 weeks gestation.
C. Antenatal steroids are beneficial.
D. Maternal opiate abuse increases the risk.
E. Maternal diabetes increases the risk.
Ans:- ACE
35) The risk of neonatal jaundice is increase by:
A. Prematurity.
B. Trisomy 21.
C. Elective caesarean section.
D. Congenital hypothyroidism.
E. Cephalahaematoma.
Ans:-ABDE
36) At birth the blood volume is approximately:-
A. 65 ml/kg body weight
B. 85 ml/kg body weight
C. 110 ml/kg body weight
D. 125 ml/kg body weight
E. 150 ml/kg body weight
Ans:- B
37) Newborn infants commonly have:-
A. Papulovesicles over the trunk.
B. Posterior fusion of the labia minora.
C. An adherent foreskin
D. Breast enlargement
E. Shallow sacral dimple
Ans:-ACDE
38) The following conditions signify disease in the newborn;-
A. Peeling of the skin of the hands and feet
B. Blanched on one side of the body and pink on the opposite side
C. Pinhead lesion on the nose ( milia)
D. Peripheral cyanosis
E. Oedema of one arm
Ans:- ALL FALS
39) Apreviously healthy full term infant have several episode of duskiness and feeding
difficulties during the second day of life . She is noted to have increasing jaundice, which
of the following tests will be the least helpful in making diagnosis:-
A. CSF
B. Urine C/ S
C. Total bilirubin
D. Endotracheal aspirate C/S
E. Venous blood
ABDULRAHMAN BASHIRCHILDREN HOSPITAL –BENGHAZI 12
Ans:- D
40) Difference between infant born to heroin-abusing mothers and infant born to
phenobarbitone-abusing mothers is that infant in the later group:-
A. Don't have withdrawal symptoms
B. Have withdrawal symptoms appearing earlier than heroin withdrawal
C. Don't develop tremor
D. Have high incidence of jaundice
E. Are usually term and full size
Ans:- E
41) Which of the following organisms is the most frequent causes of neonatal meningitis:-
A. GBS
B. E . coli
C. L . monocytogenes
D. H .influenza
E. S. pneumoniae
Ans:- A
42) One should be concerned about term infant who has not passed meconium stool:-
A. During the process of birth
B. Within few min of birth
C. By 1-2 hour of life
D. By 6-12 hour of life
E. By 24 hour of life
Ans:- E
43) The initial lesion of incontinentia pigmenti are:-
A. Deeply pigmented
B. Scaly
C. Waxy papules
D. Inflammatory bullae
E. Small vesicles
Ans:- D
44) Factor that appear to lower threshold for neurologic damage and kernicterus from
unconjugated hyperbilirubinemia include all of the following except:-
A. Acidosis
B. Asphyxia
C. Sepsis
D. Postmaturity
E. Hypothermia
Ans:- D
45) Infant born to diabetic mothers are at risk of all of the following except:-
A. Polycythemia
B. Hyperglycemic dehydration
C. Hypocalcemia
D. Congenital malformation
E. Cardiomyopathy
Ans:- B
ABDULRAHMAN BASHIRCHILDREN HOSPITAL –BENGHAZI 13
46) The Apt test is used for what purpose:-
A. Crude test for carbon monoxide poisoning
B. Semiquantitative test for lead poisoning
C. Qualitative test for fetal hemoglobin
D. Screening test for S hemoglobin
E. Test for blood viscosity
Ans:- C
47) Birth injury account for the majority of the following conditions detected in early
infancy:-
A. Intraventricular hemorrhage
B. Cephalohematoma
C. Hydrocephalus
D. Facial nerve palsy
E. Pneumothorax
Ans:-BD
48) In birth trauma:-
A. Erb's palsy involve C6,C7 & C8
B. Klumpke's palsy involve C7,C8 &T1
C. Facial nerve palsy lead to persistently closed eye
D. Sciatic nerve involvement is common
E. Cephalohematoma is characteristically present at birth
Ans:- B
In Erb-Duchenne paralysis, the injury is limited to the 5th and 6th cervical nerves. The infant
loses the power to abduct the arm from the shoulder, rotate the arm externally, and supinate the
forearm. The characteristic position consists of adduction and internal rotation of the arm with
pronation of the forearm. Klumpke paralysis is a rare form of brachial palsy; injury to the 7th
and 8th cervical nerves and the 1st thoracic nerve produces a paralyzed hand and ipsilateral ptosis,
anhidrosis and miosis (Horner syndrome) if the sympathetic fibers of the 1st thoracic root are
also injured (which reflects damage to the stellate ganglion adjacent to T1.)
49) Birth injury:-
A. Paralysis of the upper arm has better prognosis than paralysis of the lower
B. In nerve injury, neuroplasty is advised at the end of first year of life
C. In phrenic nerve paralysis, spontaneous recovery is expected
D. Facial nerve paralysis will result from the nuclear agenesis of the facial nerve
Ans:-AC
If the paralysis persists without improvement for 3–6 mo, neuroplasty, neurolysis, end-to-end
anastomosis, and nerve grafting offer hope for partial recovery. function usually returns in a few
months. Total disruption of nerves (neurotmesis) or root avulsion is the most severe, especially if
it involves C5–T1; microsurgical repair may be indicated. Fortunately, most (75%) injuries are at
the root level C5–C6, involve neurapraxia and axonotmesis, and should heal spontaneously.
Botulism toxin may be used to treat biceps-triceps co-contractions.
50) The following are useful in the assessment of gestational age in preterm:-
A. Presence of palmar creases
ABDULRAHMAN BASHIRCHILDREN HOSPITAL –BENGHAZI 14
B. Breast size
C. Sacral edema
D. The scarf sign
E. Muscle ton
Ans:-BDE
51) Cephalohematoma:-
A. Must be differentiated from subperiosteal hemorrhage
B. Is usually visible at birth
C. May calcify
D. May be associated with underling fracture of the skull
E. Should be managed surgically
Ans:-CD
52) Caput succedaneum is characterized by all of the following except:-
A. Diffuse edematous swelling of the soft tissues of the scalp involving the
portion presenting during vertex delivery
B. It may extend across the midline
C. It may extend across the suture lines
D. Edema usually disappear within 2-3 months
E. The scalp overlying the area may show mild bruising
Ans:- D
53) In newborn with oral moniliasis the most common primary source of infection
A. Maternal source ( vaginal )
B. Contaminated fomites
C. Following use of AgNO2
D. Contact by hospital carriers
E. Systemic antibiotic therapy
Ans:- A
54) Meconium impaction is associated with:-
A. Cretinism
B. Cystic fibrosis
C. Thrush
D. HMD
E. Trisomy 21 syndromes
Ans;- B
55) Premature infant is delivered precipitiously and appear asphyxiated . The infant is
cyanotic, there are no respiratory efforts,and the heart rate is 80 / min .The infant is
meconium stained and thick particulate meconium is noted in the amniotic fluid and in
infant mouth. At this point you should:-
A. Pass an umbilical artery catheter to measure the PH & PO2
B. Start bag-mask ventilation with 100% oxygen
C. Suction the oropharynx & trachea with ETT to remove the meconium
D. Intubate the trachea & begin the ventilation with 100 % oxygen
E. Establish monitoring with ECG & pulse oximeter
Ans:- C
56) Hypoglycemia has been observed in all of the following except;-
ABDULRAHMAN BASHIRCHILDREN HOSPITAL –BENGHAZI 15
A. With LBW and RDS
B. With anoxic injury
C. With hypothermia
D. Who are SGA
E. With high PO2
Ans:- E
57) All of the following are usually associated with cretinism except:-
A. Constipation
B. Prolonged jaundice
C. Lethargy
D. Tetany
E. Hypotonia
Ans:- D
58) All of the following are characteristic of single umbilical artery except:-
A. Presence in about 5 of 1000 birth
B. About 1/3 of such infants have congenital abnormalities
C. 21 trisomy is frequently found
D. Among twin ,the rate of occurrence is 35 / 1000
E. The associated congenital abnormalities may involve the genitourinary tract
Ans:- C
59) Two minutes after normal term delivary:-
A. The ductus venosus will be closed
B. The pulmonary arterial pressure will have fallen
C. The pressure in the left atrium will have fallen
D. The arterial oxygen tension will have risen
E. Regular breathing will have begun
Ans :-ABDE
60) Established neonatal resuscitation procedures include:-
A. Directing cold stream of oxygen at the nose
B. Administration of drugs with respiratory stimulant properties
C. Oropharyngeal suction
D. Bag and face mask ventilation
E. Prompt cooling
Ans:-ACD
61) The Apgar score :-
A. At 1 min is reliable measure of asphyxia
B. At 1 min is reliable measure of respiratory failure
C. At 10 min is strongly correlated with later neurological deficit
D. Includes the infant response to pharyngeal suction catheter
E. Isn't application after 10 min of age
Ans:- BCD
62) Criteria used in the Apgar score include :-
A. Core temperature
B. Heart rate
C. Respiratory rate
ABDULRAHMAN BASHIRCHILDREN HOSPITAL –BENGHAZI 16
D. Skin thickness
E. Muscle tone
Ans :-BE
63) Pink newborn with HR of 88/min is actively gasping , he has good muscle tone and
respond to nasal catheter stimulation with facial grimace, the apgar score is:-
A. 5
B. 6
C. 7
D. 8
E. 9
Ans:- D
64) Mongolian spots are characterized by all of the following except:-
A. They are permanent
B. They are usually of a slate blue pigmentation
C. They are generally observed over the buttocks
D. The area of pigmentation is well demarcated
E. They aren't associated with trisomy syndromes
Ans:-A
65) All of the following physical signs may be useful in estimating the gestational age at birth
except:-
A. There are only one or two transverse skin creases on the sole of the foot until
36 week of gestation
B. The breast nodule is usually not palpable at 33 or 34 weeks
C. The breast nodule is usually 4-10 mm in term infant
D. The testes are descending and rugae cover the entire scrotal surface by 34
weeks
E. The texture of scalp hair
Ans:- D
The testes are usually not completely descended until after 36 weeks & scrotal rugae are few and
limited the anterior and inferior aspect of relatively small scrotum . By 34 weeks the areola
become raised and between 36-37 weeks the breast bud is 1 – 2 mm reaching size of 4-10 mm at
term. The transversr foot creases develop at 31 – 32 weeks .By 36 weeks creases cover the
anterior two-third of the sole .
66) Newborn infant who remain centrally cyanosed after intubation and IPPV may have:-
A. Diaphragmatic hernia
B. Choanal atresia
C. Tension pneumothorax
D. Drug induced respiratory center impairment
E. Profound anaemia
Ans:-AC
67) The following statement about pulmonary hypertension are true :-
A. It recognized complication of group B streptococcal sepsis
B. Hyperventilation is an effective treatment
C. Tolazoline is potent pulmonary vasoconstriction
ABDULRAHMAN BASHIRCHILDREN HOSPITAL –BENGHAZI 17
D. Radial arterial PaO2 is lower than umbilical artery PaO2
E. Birth asphyxia is a risk factor
Ans:-ABE
68) Concerning air leak syndromes in the newborn:-
A. Underwater seal drain is only required if the pneumothorax is under tension
B. In term baby with small pneumothorax giving oxygen at high concentration
can worsen it
C. Increasing the I; E ratio in ventilated baby decrease the risk of pneumothorax
D. Pneumomediastenum is usually fatal
E. They can be asymptomatic
Ans;- E
69) Recognized problem of infants born at term SGA include:-
A. Hypothermia
B. Sepsis
C. Polycythemia
D. Hypoglycemia
E. Retinopathy of prematurity
Ans:-ABCD
70) Complication of steroid therapy in the newborn include:-
A. Leucopenia
B. Hypoglycemia
C. Cataract
D. Sepsis
E. Gastric perforation
Ans:-CDE
71) Concerning NEC :-
A. Exchange transfusion is predisposing factor
B. Clostridium welchii is implicated in the pathogenesis
C. It is most common in infants born less than 1500 gm
D. Oral antibiotics are useful
E. Complications include short bowel syndrome
Ans:-ACE
72) The following congenital condition require immediate ( within first week) treatment
after birth:-
A. TEF
B. Cleft lip
C. Spina bifida
D. Exomphalos
E. Hydrcephalus
Ans:-ACD
73) Preterm infant at increased risk from :-
A. Conjugated hyperbilirubinemia
B. Meconium aspiration
C. Periventricular leucomalacia
ABDULRAHMAN BASHIRCHILDREN HOSPITAL –BENGHAZI 18
D. Necrotizing enterocloitis
E. Child abuse
Ans:- CDE
74) Peri- or intraventricular cerebral hemorrhage
A. Occur in less than 10% of VLBW infant
B. Arise most commonly in the first 72 hour after delivary
C. Is direct result of impaired vitamin K supply
D. Is the single most common cause of congenital cerebral palsy
E. May result in rapidly evolving hydrocephalus
Ans:-BE
75) Feature typical of physiological jaundice include:-
A. Recognizable jaundice in the first 48 hours
B. Peak plasma bilirubin at 4-5 days
C. Persistent beyond first week
D. Irritability
E. Pale stool
Ans:- B
76) Jaundice on day 1 is often caused by ;-
A. Metabolic disorder
B. TORCH infection
C. Gastrointestinal obstruction
D. Hemolysis
E. Physiological factor
Ans:-BD
77) Persistant, conjugated hyperbilirubinaemia may be caused by:
A. Alpha-1-antitrypsin deficiency.
B. Hypothyroidism.
C. Haemolytic disease.
D. Cytomegalovirus infection.
E. Cystic fibrosis.
Ans:-ABDE
Explanation:- all cases need further investigation.
Causes of conjugated hyperbilirubinaemia in a neonte are:
1)-Intrahepatic cholestasis
-Infections.
-Congenital infections (STORCH-syphlis, toxoplasma, rubella,
cytomegalovirus, hepatitis, herpes virus infection)
-Acquired infections- septicaemia, UTI.
-Metabolic disorders –cystic fibrosis, alpha-1- antitrypsin deficiency, galactosaemia,
fructosaemia, lysosomal storage disorders, peroxisomal disorders.
-Endocrine disorders – hypothyroidism, hypopituitarism, hypoadrenalism.
-Anatomical disorders – intrahepatic:bile duct hypoplasia.
-Miscellaneous – idiopathic neonatal hepatitis, chromosomal abnormalities, trisomy 21, 18
and 13.
2)-Extrahepatic cholestasis:- biliary atresia, choledochal cyst, spontaneous bile duct
ABDULRAHMAN BASHIRCHILDREN HOSPITAL –BENGHAZI 19
perforation, inspissated bile syndrome.
Unconjugated neonatal jaundice is normally physiological and resolves spontaneously.
78) All of the following are characteristic of jaundice associated with breast feeding except :-
A. Significant elevation of unconjugated bilirubin
B. Rapid fall in serum bilirubin after discontinuation of nursing
C. Nursing can be resumed after several days without return of hyperbilirbinemia
D. Significant elevation of conjugated bilirubin
E. Kernicterus has never been reported to occur as result of breast milk jaundice alone
Ans :- D
79) Persistent jaundice during the first month of life may associated with all of the following
except:-
A. Cytomegalic inclusion disease
B. Congenital atresia of the bile duct
C. Galactosemia
D. Rh incompatibility
E. Penicillin treatment
Ans:- E
80) A 26-week gestation preterm infant is now 6 weeks old and weighs 1,250 g. He is
receiving full-volume enteral nutrition. The only significant finding on physical
examination is pallor. He has anemia (hematocrit of 28% [0.28]; reticulocyte count of 8%
[0.08]) and receives iron supplementation. He is receiving a formula that is high in
polyunsaturated fatty acids.Of the following, the MOST correct statement about his need
for vitamin E is that it
A. does not need to be supplemented in infancy
B. has no effect on anemia
C. needs to be supplemented now
D. will be needed when the infant is 3 months old
E. will prevent anemia
Preferred Response: C
Historically, inadequate vitamin E, a high level of polyunsaturated fatty acids (PUFAs) in infant
formula, and exposure to the oxidizing effects of iron supplementation contributed to a hemolytic
anemia seen in preterm infants. In the United States and many developed countries, infant
formulas now provide an adequate vitamin E:PUFA ratio to eliminate this risk. However, preterm
infants continue to have low vitamin E levels due to limited stores, especially when the birth is
extremely premature, and limited enteral feedings early in their postnatal neonatal intensive care
unit course.
The neonate who has anemia and is receiving iron supplementation, such as the infant described
in the vignette, requires the antioxidant effect of vitamin E to reduce red blood cell hemolysis. A
total daily requirement of 10 to 25 IU of vitamin E meets the infant’s needs, only 50% of which is
provided by dietary formula. Of note, human milk also is an incomplete source of vitamin E.
81) Amniocentesis is useful in establishing the prenatal diagnosis of:-
A. Down syndrome
B. Meningomyelocele
ABDULRAHMAN BASHIRCHILDREN HOSPITAL –BENGHAZI 20
C. Erythroblastosis fetalis
D. Achondroplasia
Ans:-ABC
82) USS can be used during pregnancy to:-
A. Determine crown rump length
B. Determine fetal sex
C. Determine biparietal diameter
D. Accurately determine fetal weight
Ans ;-AC
83) Gestation that produce multiple births:-
A. Are classified as high risk
B. Are always delivered by cesarean section
C. Can produce infant with discordance in body size at birth
D. Aren't associated with the premature onset of labour
Ans:-AC
84) For newborn infant, the least important factor to consider in assessing the risk of
kernicterus is:-
A. Breast feeding
B. Acidosis
C. Sepsis
D. Albumin level
E. Moxalactam, cephalosporin type antibiotic
Ans:- A
85) The following maternal conditions are known to cause adverse effects on the neonate:
A. Diabetes insipidus.
B. Toxoplasmosis.
C. Chronic myeloid leukaemia.
D. SLE.
E. Hyperthyroidism.
Ans:-BCDE
Explanation:- Maternal SLE is a risk factor for neonatal lupus syndrome. The neonate presents
with clinical features of SLE due to transplacental passage of maternal Ab. The skin is frequently
involved with malar rashes and there can be haematological and cardiac abnormalities. The most
frequent heart abnormality is congenital heart block – 90% of mothers whose infants have
congenital heart block are anti-Ro(SSA)Ab positive.
Toxoplasmosis leads to congenital infection by transplacental transmission. The rate of
transmission is +/- 60% of third trimester infections and 20%-30% during the first two trimesters.
The ewborn presents with the classic triad of hydrocephalus, chorioretinitis, and cerebral
calcification.
Maternal chronic myeloid leukaemia can have adverse effect on pregnancy – fetal/neonatal
mortality is 16-38%. Pathology can be secondary to placental leukaemic infiltrates, anaemia and
infectious complications. Splenomegaly can restrict intrauterine growth and lead to premature
delivery.
ABDULRAHMAN BASHIRCHILDREN HOSPITAL –BENGHAZI 21
If hyperthyroidism is due to Grave’s disease or Hashimoto thyroiditis, the neonate may present
with thyrotoxicosis due to the transplacental passage of TSI. These symptoms are frequentely
short-lived i.e as long as the circulating antibodies persist in the baby’s circulation.
86) Each of the following statement about GBS infection in the newborn is true except:-
A. Incidence is correlates inversely with the presence of maternal antibodies
B. The exotoxin has powerful pulmonary vasoconstrictive effects
C. The disease has two distinct pattern; early & late onset
D. Both the incidence & severity of the disease can be lessened by pretreatment of the
mother who is colonized
E. The risk of invasive disease isn't related to the amount of inoculum received by the
infant
Ans:- E
87) Newborn infant may present with bile stained vomiting and abdominal distension as the
result of
A. Oesophageal atresia
B. Duodenal atresia
C. Birth asphyxia
D. Electrolyte disturbance
E. Cystic fibrosis
Ans:-BCDE
88) The following feature are consistent with newborn infant having oesophageal atresia and
tracheo-oesophageal fistula:-
A. Maternal polyhydramniose
B. Passage of wide bore orogastric catheter into the stomach
C. Plain x-ray evidence of air in the stomach and small bowel
D. Plain x-ray evidence of hemivertebra
E. Excessive mucus in the nostrils or mouth.
Ans:-ACDE
89) Vitamin K
A. Is an essential cofactor for the synthesis of coagulation factor II , VII, IC, C
B. Is readily transported across the placenta
C. Is present in the breast milk at higher concentration than in cow milk.
D. Given in single oral dose after delivary effectively prevent s hemorrhagic disease
E. Related hemorrhage in the newborn is commoner when mother have taken
anticonvulsant during pregnancy
Ans:-AE
90) The following are recognizable causes of neonatal convulsion:-
A. Birth asphyxia
B. Hypoglycemia
C. Hypothermia
D. Opiate withdrawal
E. Hypernatremia
Ans:-ABDE
91) With regard to Apgar score:-
ABDULRAHMAN BASHIRCHILDREN HOSPITAL –BENGHAZI 22
A. 2 points given to pulse of 88/min
B. 1 point is given for irregular gasps
C. An initial satisfactory score gurarantees an eventual perinatal period
D. Score of 2 at 10 min carries worse prognosis than score of 2 at 5 min
E. The lowest possible score is 1
Ans:-BD
92) Intrauterine posture is commonly responsible for:-
A. Congenital dislocation of the hip
B. Plagiocephaly
C. Sternomastoid shortening
D. Syndactyly
E. Mandibular asymmetry
Ans:-ABCE
93) Established neonatal screening tests include:-
A. Umbilical cord blood analysis to detect phenylketonuria
B. Umbilical cord blood analysis to detect galactose
C. Umbilical cord blood analysis to detect sickle cell disease
D. Capillary blood analysis at 6-8 days to detect elevated TSH
E. Capillary blood analysis at 6-8 days to detect elevated immunoreactive trypsin
Ans:-CDE
94) Maternal condition that may have effect in the neonatal period include:-
A. ITP
B. Multiple sclerosis
C. DM
D. Varicella zoster
E. Bornholm disease
Ans:-ACDE
95) Neonatal polycythemia :-
A. Occur in small for date infant as response to placental insufficiency
B. Has an increased incidence if maternal diabetes is poorly controlled
C. Carry an increased risk of cerebral venous sinus thrombosis
D. Is recognizable feature of congenital hypothyroidism
E. May occur as result of feto-maternal transfusion
Ans:-ABCD
96) Polycythemia on the newborn is characterized by all of the following except:-
A. Increased the incidence in IDM
B. Placental transfusion decreased by late clamping of the umbilical cord
C. Clinical presentation that includes jitteriness & poor feeding
D. Complication including heart failure & NEC
E. Elevated viscosity of whole blood
Ans;- B
97) Causes of neonatal polycythemia include:-
A. Congenital rubella infection
ABDULRAHMAN BASHIRCHILDREN HOSPITAL –BENGHAZI 23
B. Pre-eclampsia
C. Maternal diabtus mellitus
D. Delayed clamping of umbilical cord
E. Congenital adrenal hyperplasia
Ans:- BCDE
98) Meconium aspiration pneumonia:-
A. Occur with equal frequency in term and preterm infant
B. In infant required ventilation for this condition, combination of high PEEP and
rapid rate is advisable
C. Has high risk of developing even if liquor is only thinly stained
D. High dose steroid are the mainstay of treatment
E. Antibiotic treatment is an important part of treatment
Ans :- all False
99) The following statement are true regarding hemolytic disease of newborn ( HDN ):_
A. Hemolytic disease should be suspected if jaundice is noted in the first 24 hour of
life
B. HDN may occur if mother is group A +ve and the baby is group O+ve
C. If due to Rh –incompatibility the severity of the hemolysis typically increase with
each affected pregnancy
D. HDN due to ABO incompatibility can be detected at 36 week gestation by
amniocentesis
E. As long as the level of unconjugated bilirubin never rises above20 mg/dl there is no
dangerous of kernicterus
Ans:-AC
100) Full term infant has tachypnea with grunting , chest X ray show well expanded lung
with streaky shadows radiating from the bilateral hilar region , the most likely diagnosis
is :-
A. TTN
B. Congenital pneumonia
C. BPD
D. Meconium aspiration
E. Aspiration pneumonia
Ans:- A
101) Condition associated with polyhydramniose include:-
A. Oesophageal atresia
B. Down's syndrome
C. Renal agenesis
D. Cord around the neck
E. CNS malformation
Ans:- ABE
102) The following are causes of polyhydramnios:
A. Maternal diabetes mellitus.
B. Potter syndrome.
C. Anencephaly.
D. Oesophageal atresia.
ABDULRAHMAN BASHIRCHILDREN HOSPITAL –BENGHAZI 24
E. Polycystic kidneys.
Ans:-ACD
103) The following are causes of polyhydramniose:-
A. Pottrer syndrome
B. TEF and oesophageal atresia
C. Rh – incompatibility
D. Anencephaly
E. Maternal diabetus
Ans:-BDE
104) For necrotizing enterocloitis:-
A. Term baby are particularly at risk
B. Perinatal asphyxia is risk factor
C. Expressed breast milk from milk bank confer no protection
D. Failure of temperature control is late sign
E. High platelet count is frequently seen
Ans:- B
105) You diagnose necrotizing enterocolitis in a preterm neonate who has abdominal
distention and blood in the stool. You decide that this infant should be placed on a 14-day
regimen of parenteral nutrition.Of the following, the micronutrient for which weekly
monitoring is MOST recommended during this infant’s parenteral nutrition therapy is
A. copper
B. iron
C. phosphorus
D. selenium
E. zinc
Preferred Response: C
Parenteral nutrition (PN), the intravenous administration of carbohydrates, lipids, amino acids,
and micronutrients, is an important component in the management of a variety of chronic
disorders, including surgical conditions (eg, omphalocele, gastroschisis, diaphragmatic hernia,
short bowel syndrome), inflammatory conditions (eg, Crohn disease, ulcerative colitis,
pseudomembranous colitis, pancreatitis, graft versus host disease), hypermetabolic states (eg,
burns, trauma), and intestinal motility disorders (eg, pseudo-obstruction). PN is especially
important in the support of very low-birthweight infants, who frequently have increased caloric
requirements, decreased oral intake, and immature intestinal motility.
In addition to glucose, amino acids, sodium, potassium, and chloride, PN provides additional
mineral supplements, including calcium, phosphorus, magnesium, zinc, copper, selenium,
chromium, manganese, molybdenum, and iodide. In preterm infants, the administration of both
calcium and phosphorus is important to prevent metabolic bone disease.
However, the amount of calcium and phosphorus that can be administered in PN must be limited
because of the risk of precipitation in the PN and formation of calcium-phosphorus complexes
that could embolize. It generally is thought that the chance of precipitation is high if the product
of the concentrations (in mmol/L) of the calcium and phosphorus in PN is greater than 40.
Because hypophosphatemia can have significant consequences (including impaired cardiac
function, muscle weakness, and hemolysis), it is recommended that serum phosphorus
concentrations be assessed at least once weekly for patients receiving PN. The serum
ABDULRAHMAN BASHIRCHILDREN HOSPITAL –BENGHAZI 25
concentrations of copper, iron, selenium, and zinc, which are trace elements, require only periodic
assessment (eg, once a month or less).
106) An 1,800-g preterm infant is recovering from surgery for gastroschisis. Of the
following, the MOST correct statement regarding this infant’s immediate daily
nutritional requirements is that he
A. can meet energy needs from fat through enteral feeding
B. can meet protein needs for growth through enteral feeding
C. requires lysine supplementation to promote protein synthesis
D. requires more protein per kilogram than a term infant
E. requires 2.5 g/kg per day of protein
Preferred Response: D
Milk remains the principal source of nutrition for infants, who consume 120 to 150 mL/kg per day
in the newborn period.
Human milk has 67 kcal/100 mL, and most term infant formulas have a similar composition. The
energy derived from ingesting milk comes primarily from fat calories (3.8 g/100 mL = 34kcal/100
mL), followed by carbohydrates (lactose, 7 g/100 mL = 28 kcal/100 mL) and minimally from
protein (1.3 g/100 mL = 5 kcal/100 mL). Conditions such as prematurity, lung disease, or surgery
may increase both caloric and specific nutrient requirements for newborns.
The newborn described in the vignette can only take limited enteral nutrition and has an
increased need for both protein and calories to facilitate healing and growth. The normal 2 to 3
g/kg per day of protein ingested by the term infant who is either formula- or breastfed is
inadequate for this low-birthweight, preterm newborn. He will grow and heal best receiving 3.5 to
4.0 g/kg per day of protein. The potential benefits of single amino-acid supplementation, such
as lysine, remain unknown.
107) Hypoxic-ischemic encephalopathy all are correct except:-
A. Result from excessive use of oxytocin
B. IUGR is be first indication of fetal hyoxia
C. Associated with increased beat to beat variability
D. Prognosis depend on gestational age
E. Persistent of abnormal neurological sign at two week indicate poor prognosis
Ans:- C
108) polyhydramniose:-
A. Is defined as an amniotic fluid volume of more than 500 ml
B. Occur with increased frequency in diabetic pregnancy
C. Is associated with renal agenesis
D. Is associated with tracheo-oesophageal fistula
E. Is associated with increased risk of premature labor
Ans :-BDE
109) Polyhydramnios is associated with all of the following condition except:-
A. Amniotic fluid volume between 500-2000 ml
B. Maternal diabetes
ABDULRAHMAN BASHIRCHILDREN HOSPITAL –BENGHAZI 26
C. Twins
D. Erythroblastosis fetalis
E. Down syndrome
Ans:- A
110) Condition associated with oligohydramnios include:-
A. Infantile polycystic kidney
B. Congenital heart disease
C. IUGR
D. High intestinal obstruction
E. Posterior urethral valve
Ans:- ACE
111) Regarding hyaline membrane disease ( HMD)
A. Can occur in infants of diabetic mother of 37-40 week gestation
B. Infant born to mothers who are heroin addicts are at increased risk of HMD
C. Light-for –date infant of 33 weeks gestation has greater risk of developing
HMD than 33 week infant of appropriate weight
D. Administration of artificial surfactant is curative
E. Chest x-ray finding are markedly different between cases of HMD and Group
B streptococcal pneumonia
Ans:- A
112) Hyaline membrane disease:-
A. Is more common in babies of diabetic mother
B. Is due to surfactant deficiency
C. Is not seen in term babies
D. Occur most commonly at 12 hour post-delivary
E. Always require ventilation
Ans:- AB
113) The following symptoms in an infants in the first month of life should alert one to
possibility of hypothyroidism:
A. Prolonged jaundice
B. Vomiting
C. Diarrhea
D. Hoarse cry
E. Voracious appetite
Ans:- AD
114) The following malformation can occur in children born to mother who have IDDM:
A. Cleft lip/ palate
B. Caudal regression syndrome
C. Femoral hypoplasia
D. Holopresencaphaly
E. Polydactyly
Ans:- BCDE
115) With regard to RDS of the newborn
ABDULRAHMAN BASHIRCHILDREN HOSPITAL –BENGHAZI 27
A. Meconium aspiration pneumonia is especially likely to be the cause if the infant is
preterm
B. HMD is unlikely to be the cause if the prepartum lecithin sphingomylin (L/S) ratio
in amniotic fluid is > 2;1
C. Patchy opacities on chest X-ray are evidence in favour of diagnosis HMD
D. In the artificial ventilation of infant with HMD , inflation pressure must never
exceed 30 cm of water
E. CPAP is an effective in treatment for Pneumothorax
Ans : B
116) In RDS due to meconium aspiration
A. Antibiotic therapy is of crucial importance
B. Steroid therapy improve the prognosis
C. There is high risk of Pneumothorax
D. Chest X-ray typically shows ground glass opacity
E. The infant may also suffer from cerebral oedema
Ans:-C
117) Recognized association of small-for-date babies include
A. Maternal smoking in pregnancy
B. Hemolytic disease due to ABO incompatibility
C. Pre-eclamptic toxaemia
D. Congenital rubella infection
E. Fetal alcohol syndrome
Ans:- ACDE
118) Prolonged neonatal jaundice is recognized feature in infants with
A. CMV infection
B. Congenital hypothyroidism
C. Untreated urinary tract infection
D. Tracheo-oesophageal fistula
E. Galactosemia
Ans:- ABCE
119) In baby of 32 week gestation who has tachypnea and sternal recession at 4 hour of age:
A. The ductus arteriosus is likely to be patent
B. Meconium aspiration pneumonia is likely to be diagnosis
C. The L/S ratio is likely to be low
D. Fluid level on chest X-ray suggest pneumonia due to Group B streptococcus
E. The presence of bowel shadows on the left side of the chest on X-ray is
diagnostic of Tracheo-oesophageal fistula
Ans :-AC
120) The following condition characteristically cause jaundice within the first 24 hour of life:-
A. G6PD deficiency
B. Congenital hypothyroidism
C. Sever congenital CMV infection
D. Choledochal cyst
E. Primary tyrosinaemia
Ans:- AC
ABDULRAHMAN BASHIRCHILDREN HOSPITAL –BENGHAZI 28
121) Hypothermia can cause the following in LBW infant:-
A. Decrease synthesis of surfactant
B. Hypernatremia
C. Hypoglycemia
D. Increased oxygen consumption
E. Hypercalcemia
Ans :- ACD
122) The following substance are freely transmitted across the placenta:-
A. Carbimazole
B. Diazepam
C. Warfarin
D. Pethidine
E. IgM antibodies
Ans:-ABCD
123) In congenital diaphragmatic hernia:-
A. Hernia most commonly occur on the left
B. Pulmonary hypoplasia is the major cause of death
C. Persistent fetal circulation occurs uncommonly
D. Associated congenital anomalies are common
E. Most present between 12-24 hours of life
Ans:- AB
The incidence of CDH is between 1/2,000 and 1/5,000 live births, with females affected twice as
often as males. Defects are more common on the left (85%) and are occasionally (<5%) bilateral.
Pulmonary hypoplasia and malrotation of the intestine are part of the lesion, not associated
anomalies. Most cases of CDH are sporadic, but familial cases have been reported. Associated
anomalies have been reported in up to 30% of cases; these include central nervous system lesions,
esophageal atresia, omphalocele, and cardiovascular lesions. CDH is recognized as part of several
chromosomal syndromes: Trisomy 21, Trisomy 13, Trisomy 18, Fryn, Brachmann–de Lange,
Pallister-Killian, and Turner. Relative predictors of a poor prognosis include an associated major
anomaly, symptoms before 24 hr of age, severe pulmonary hypoplasia, herniation to the
contralateral lung, and the need for ECMO.
124) In the normal newborn infant in the first 24 hour of life:-
A. Many significant heart defect may be clinically undetectable
B. Normal arterial PaO2 help the ductus to close
C. Hypoxia causes pulmonary artery vasoconstriction
D. All children with murmur heard in the first 24 hour should be followed up for
at least 6 months
E. The systolic blood pressure is between 40-80 mmHg
Ans:-ABCE
125) A light –for-date full term baby is at particular risk from the following condition:-
A. HMD
B. Physiological jaundice
C. Milk aspiration
D. Hypoglycemia
ABDULRAHMAN BASHIRCHILDREN HOSPITAL –BENGHAZI 29
E. Apnoeic attacke
Ans:- D
126) Recurrent apnoea of prematurity:-
A. Characteristically develop within the first 24 hour of life
B. Is more likely to occur in infant of < 32 week gestation
C. Usually respond to naloxone
D. Should be treated with 100% oxygen during attack
E. May be accentuated by the presence of nasogastric tube
Ans:- BE
127) In the management of HMD:
A. Tolazoline may lead to systemic hypertension
B. Early use of CPAP may reduce the need for subsequent ventilatory support
C. Antibiotic improve ventilation- perfusion ratio
D. The illness may be expected to increase in the severity for the first 5 days
E. Corticosteroid used postnatally have beneficial effect on the course of the
disease
Ans:- B
128) Characteristic finding in preterm baby include:-
A. Chin reaching only to the tip of shoulder
B. Full wrist flexion
C. Flat on couch when lying prone
D. Incomplete ankle dorsiflexion
E. Incomplete knee extension with hip fully flexed
Ans:- C
129) Drug effects on the fetus:-
A. Isotretinion can lead to CNS defects mainly if given in the third trimester
B. Phenytoin can lead to cleft lip, finger and toe abnormalities mainly if given in
the second trimester
C. Carbimazole cause goiter, mainly if given in the third trimester
D. Warfarine can lead to neonatal hemorrhage mainly if given in the first
trimester
E. Valproate can lead to neural tube defects mainly if given in the third trimester
Ans:- C
130) The external criteria for the Dubowitz score for gestational age include:-
A. Breast size
B. Skin texture
C. External genitalia
D. Langue hair
E. Nose firmness
Ans:-ABCD
131) Problem of babies born to mothers with poorly controlled diabetus mellitus:
A. Hypermagnesemia
B. Polycythemia
C. Hypocalcemia
D. Sacral agenesis
ABDULRAHMAN BASHIRCHILDREN HOSPITAL –BENGHAZI 30
E. Respiratory distress
Ans:-BCDE
132) The following are recognised associations of maternal diabetes:
A. Sacral agenesis
B. Intrauterine growth retardation
C. Macrosomia
D. Hyaline membrane disease
E. Hypertrophic cardiomyopathy
Ans:-ABCDE
Comments:
Overall, malformations occur in 6%, with an increased incidence of cardiac malformations, sacral
agenesis, hyperplastic left colon. Intrauterine growth retardation is 3 times as common due to
small vessel disease in the mother. More common is macrosomia, and this is related to the degree
of maternal hyperglycaemia. The glucose crosses the placenta while the insulin does not, so the
fetus increases its production of insulin. This results in increased cell number and
size. 25% of IDM are greater than 4kg compared with only 8% of non-diabetics. This gives
problems with delivery such as CPD, shoulder dystosia resulting in an increased incidence of birth
asphyxia and trauma.
In the neonatal period, hypoglycaemia, respiratory distress and reversible hypotrophic
cardiomyopathy and polychthaemia are all more common. Gestational diabetes is when
carbohydrate intolerance occurs only during pregnancy. It is commonest in obese women and
those from Afro-Caribbean and Asian ethnic groups. In these women there is no increase in
congenital malformations, thoug macrosomia and other complications remain similar in
frequency.
133) Characteristic finding in full term baby are:-
A. Full knee extension with hips fully flexed
B. Momentary neck extension when held sitting
C. Chin reach beyond tip of shoulder
D. Hip abducts and legs flat on couch when lying supine
E. Full ankle dorsiflexion
Ans:- BE
134) The external criteria for the Dubowitz score for gestational age include:-
A. Nipple formation
B. Ear form & firmness
C. Skin colour & opacity
D. Protruding tongue
E. Planter creases
Ans:- ABCE
135) In fetal circulation:-
A. 30% of the fetal cardiac output goes through the placenta
B. Oxygenated blood from the placenta passes through the ductus arteriosus toward
the right atrium
C. The oxygen saturation of blood in umbilical arteries is approximately 60%
ABDULRAHMAN BASHIRCHILDREN HOSPITAL –BENGHAZI 31
D. Blood entering the heart from the inferior vena cava is diverted directly to the left
atrium via patent foramen ovale
E. There is one umbilical vein
Ans:- CDE
136) The following disease can now be diagnosed prenatally:-
A. Phenylketonuria
B. Homocystinuria
C. Sever combined immunodeficiency
D. Fanconi's anaemia
E. Lesh-Nyhan syndrome
Ans:- ABCDE
137) Resuscitation of neonate with coarctation of aorta may require the use of :-
A. Frusemide
B. Bicarbonate
C. Indomethacin
D. Dopamine
E. Prostaglandin E
Ans:- ABDE
138) Recognized causes of the floppy baby include:-
A. Trisomy 21
B. Zellweger syndrome
C. Becker muscular dystrophy
D. Spinal muscular atrophy
E. Hypothyroidism
Ans:- ABDE
139) Which of the following may cause apnoea in preterm infant
A. Hypocalcemia
B. Hypoglycemia
C. RSV infection
D. Caffeine
E. Intraventricular hemorrahage
Ans:- ABCE
140) Failure of resuscitation of newborn may be due to :-
A. PDA
B. VSD
C. Congenital diaphragmatic hernia
D. Pulmonary hypoplasia
E. HMD
Ans:-CD
141) Feature of IUGR include:-
A. Neutropenia
B. Hypoglycemia
C. Necrotizing Enterocolitis
D. Weight loss > 10% of birth weight in first week
E. Thermal instability
ABDULRAHMAN BASHIRCHILDREN HOSPITAL –BENGHAZI 32
Ans:- ABCE
142) Newborn infant has breathing difficulty with central cyanosis .Nitrogen washout test
raised the PO2 to 15 kPa possible diagnosis include:-
A. Fallot's tetralogy
B. VSD
C. Pneumonia
D. Tricuspid atresia
E. HMD
Ans:- CE
143) Newborn infant has central cyanosis and fit with slow & shallow breathing , Nitogen
washout test produce slight raise in the PO2 , the most likely cause of cyanosis is:-
A. Methemoglobinemia
B. Cerebral disorder
C. Persistent fetal circulation
D. Congenital cyanotic heart disease
E. Lung disease
Ans:- B
144) The following disease can now be diagnosed prenatally:-
A. Hypercholesterolemia
B. Polycystic kidney disease
C. Cystinosis
D. Gaucher's disease
E. Beta thalassaemia
Ans:-ABCDE
145) Common causes of seizure in the neonatal period include:-
A. Intracranial hemorrhage
B. Electrolyte disturbance
C. Infection
D. Drug withdrawal
E. Febrile convulsion
Ans:- ABCD
146) Neonatal convulsion may be caused by :-
A. Hypomagnesaemia
B. Hyperkalamia
C. Pyridoxine dependency
D. Cephalohematoma
E. HSV infection
Ans:- ACE
147) The following disease can now be diagnosed prenatally:-
A. Sickle cell disease
B. Maple syrup urine disease
C. Retinoblastoma
D. Hypophosphaturia
E. Von-Willebrand disease
ABDULRAHMAN BASHIRCHILDREN HOSPITAL –BENGHAZI 33
Ans:- ABCDE
148) Which of the following statement regarding surfactant therapy is correct :-
A. The incidence of Pneumothorax is reduced
B. The incidence of intraventricular hemorrhage is reduced
C. Early therapy ( within 4 hour) is more beneficial than later administration ( after
12 hour)
D. Prolonged courses of surfactant therapy confer advantages
E. The incidence of chronic lung disease is increased
Ans:- ABC
149) Which of the following statement regarding periventricular hemorrhage are correct:
A. Most hemorrhage occur after the fourth day of life
B. Over 80% of hemorrhage progress to ventricular dilatation
C. Most cases of posthemorrhagic hydrocephalous are communicating
D. Early ventricular tapping improve the neurological outcome
E. Hemorrhage is usually caused by hypoglycemia
Ans:- C
150) Complication of phototherapy include:-
A. Diarrhea
B. Erythematosus rash
C. Hyponatremia
D. Skin discoloration
E. Peripheral desqumation
Ans:- ABD
151) Which of the following are characteristic haemodynamic changes encountered at birth:-
A. Increase the pulmonary vascular pressure
B. Right to left flow through the ductus arteriosus
C. Closure of the ductus arteriosus
D. Increased right ventricular end diastolic pressure
E. Reversal of flow across the foramen ovale
Ans:- C
152) Feature suggestive of TEF with oesophageal atresia include:-
A. FTT
B. Recurrent pneumonia
C. Oligohydramniose
D. Large amount of mucus in the pharynx at delivary
E. Slow to establish feed
Ans:- D
153) The development of Pneumothorax is associated with :-
A. Surfactant therapy
B. Artificial paralysis
C. Patient triggered ventilation
D. Meconium aspiration
E. Pulmonary interstitial emphysema
Ans:- DE
ABDULRAHMAN BASHIRCHILDREN HOSPITAL –BENGHAZI 34
154) Cyanotic congenital heart disease in the newborn include:-
A. TGA
B. Pulmonary stenosis
C. Fallot's tetrallogy
D. VSD
E. Tricuspid atresia
Ans:- AE
155) The following congenital disorder require immediate ( within first week) treatment after
birth:-
A. Hemangioma
B. Imperforate anus
C. Choanal atresia
D. Hypospadius
E. Congenital diaphragmatic hernia
Ans:-BCE
156) Causes of neonatal hypoglycemia include;-
A. Erythroblastosis fetalis
B. Glycogen storage disease type I
C. Maternal treatment with sodium valproate
D. Galactosemia
E. Congenital adrenal hypoplasia
Ans:- ABDE
Plasma glucose level of <40 mg/dL, During gestation, glucose is freely transferred across the
placenta by the process of facilitated diffusion. However, after birth, the infant must adjust to the
sudden withdrawal of this transplacental supply. In all infants, there is a nadir in blood sugar
between 1 and 3 hours of life. During the first 12-24 hours of life, newborns are at increased risk
for hypoglycemia because gluconeogenesis and especially ketogenesis are incompletely
developed. These factors are accentuated in preterm infants, infants of diabetic mothers, infants
with erythroblastosis fetalis, asphyxiated infants, and infants who are small or large for gestational
age.
Transient hypoglycemia causes:- prematurity, hypothermia, Birth asphyxia, sepsis,
erythroblastosis fetalis, infants with Beckwith-Wiedemann syndrome, maternal diabetes,
maternal glucose infusion in labor, and intrauterine growth restriction (IUGR)and maternal
drugs( b sympathomimetic and chlorpropamide).
Persistent hypoglycemia may be due to: - inborn error of metabolism (Glycogen storage
disease type I , P & IV, maple syrup urine disease, Nesidioblastosis, galactosemia,and
mitochondrial fatty acid oxidation defects such as MCAD deficiency. Hormonal deficiency
suc as congenital hypopituitrism,congenital glucagons deficiency and cortisol deficiency
states including congenital adrenal hypoplasia.
Features on physical examination suggest the etiology of hypoglycemia:-
· Macrosomia: This occurs in infants of diabetic mothers, infants with severe congenital
hyperinsulinism, and infants with Beckwith-Wiedemann syndrome; recall that insulin is a
growth factor and that hyperinsulinism leads to macrosomia.
· Midline defects: Congenital pituitary deficiency can be associated with midline defects
such as cleft lip, cleft palate, single central incisor, and micro-ophthalmia.
ABDULRAHMAN BASHIRCHILDREN HOSPITAL –BENGHAZI 35
· Micropenis: Congenital gonadotropin deficiency and possible pituitary abnormalities cause
this condition.
· Hepatomegaly: This is associated with glycogen storage diseases and fatty acid oxidation
disorders.
157) Healthy term neonate differ from adult in the following way:-
A. Less complement
B. Decrease IgG level
C. Fewer B lymphocytes
D. Lower level of secretory IgA
E. Higher level of C reactive protein
Ans:- AD
158) Necrotizing enterocloitis is recognized complication of:-
A. Polycythemia
B. Umbilical arterial catheterization
C. Parenteral feeding
D. Maternal Crohn's disease
E. Birth asphyxia in term neonate
Ans:- ABE
159) NEC is associated with:-
A. Epidemic
B. Thrombocytopenia
C. Malabsorption
D. Bile stained aspirate
E. Air in the portal tree on abdominal X –ray
Ans:- ABCDE
160) The presenting feature of NEC include:-
A. Apnoea
B. Bradycardia
C. Abdominal distension
D. Intramural gas on abdominal X ray
E. Bloody stool
Ans:- ABCDE
161) A 3-week-old breastfed infant presents to the emergency department with irritability,
fever, jaundice, and hepatomegaly. A laboratory evaluation shows a normal complete blood
count and a bilirubin concentration of 6.5 mg/dL (111.2 mcmol/L). A urinalysis is positive for
reducing substances. A blood culture is positive for Escherichia coli. You initiate antibiotic
therapy Of the following, the MOST appropriate dietary management of this patient is to
A. continue breastfeeding
B. switch to a cow milk-based formula
C. switch to a soy-based formula
D. switch to a whey hydrolysate formula
E. switch to an elemental formula
Preferred Response: C
ABDULRAHMAN BASHIRCHILDREN HOSPITAL –BENGHAZI 36
The clinical features of jaundice, hepatomegaly, and invasive Escherichia coli infection described
for the neonate in the vignette suggest the possible diagnosis of galactosemia.
Galactosemia is an autosomal recessive disorder most commonly caused by a deficiency of the
enzyme galactose-1-phosphate uridyltransferase. The reducing substances in the urine represent
the accumulation of galactose. In addition to recognizing and treating the gram-negative infection
in the infant, it is important to remove lactose, which is comprised of glucose and galactose,
from the diet as soon as the diagnosis is suspected.
Soy protein formulas are the first choice of nutrition for infants who have suspected or proven
galactosemia because the carbohydrate source in these formulas is sucrose or corn syrup rather
than lactose. Protein hydrolysate and elemental formulas also contain other carbohydrates than
lactose, but they can be more expensive and less readily available than soy formulas. Lactose is
the primary carbohydrate in human milk, cow milk-based formulas, and most whey hydrolysate
formulas.
162) Hydrops fetalis may be associated with :-
A. Rhesus isoimmunization
B. Paroxysmal supraventricular tachycardia
C. CMV infection
D. Achondroplasia
E. Renal vein thrombosis
Ans:- ABCDE
163) The following drugs are correctly paired with their potential teratogenic effect:-
A. Alcohol & macrocephaly with congenital heart disease
B. Phenytoin & meningomyelocele
C. Isotretinoic acid & cutis laxis syndrome
D. Penicillamine & facial abnormalities with pinna defects
E. Lithium & Ebtein's anomaly
Ans:-BCDE
Warfarine Hypoplastic nasal bridge ,chrondroplasi puncta
Isotretinoic acid facial , ear , cardiovascular abnormality
Phenytoin Hypoplastic nail, IUGR, typical facies & may be associated with
neural tube defect
Tetracyclin Enamel hypoplasia
Sodium Valproate Neural tube defect
164) The neonate of mother with SLE may demonstrate:-
A. Polythycemia
B. Rash
C. Neutropenia
D. Atrial fibrillation
E. Anti-Ro antibodies
Ans:- BCE
165) Pulmonary hypoplasia is consequence of :-
ABDULRAHMAN BASHIRCHILDREN HOSPITAL –BENGHAZI 37
A. Congenital varicella zoster
B. Anencephaly
C. Posterior urethral valve
D. Congenital diaphragmatic hernia
E. Exomphalos
Ans:- BCDE
166) The following conditions may present with bile-stained vomiting in the first week of life:-
A. Duodenal atresia
B. Cystic fibrosis
C. Inguinal hernia
D. GER
E. NEC
Ans:- ABCE
167) Renal immaturity in normal neonate born at term is manifested as:-
A. Reduced number of nephrons
B. Decreased glucose reabsorption
C. Increased glomerular filteration rate
D. Decreased renal bicarbonate reabsorption
E. Decreased urea excretion
Ans:- BDE
168) The following maternal factors increase the incidence of surfactant deficient RDS:-
A. Steroid therapy
B. Opiate
C. Placental insufficiency leading to IUGR
D. Diabetus
E. Alcoholism
Ans:- D
169) Bacteria commonly isolated in case of neonatal meningitis include:-
A. Escherichia coli
B. Haemophilus infuenzae
C. Group B streptococcus
D. Staphylococcu epidermidis
E. Neisseria meningitis
Ans:- AC
170) In the infant of diabetic mother :-
A. The infants brain size is increased beyond normal
B. The infant's liver size is increased beyond normal
C. He can be small for gestational age
D. There is an increased incidence of polycythemia
E. He has an increased incidence of hypertrophic Cardiomyopathy
Ans:- BCDE
171) Galactosaemia:-
A. Is caused by deficiency of the enzyme galactokinase
ABDULRAHMAN BASHIRCHILDREN HOSPITAL –BENGHAZI 38
B. Causes jaundice in the newborn
C. May present with cataract at birth
D. Is associated with Escherichia coli septicemia
E. Is diagnosed as result of screening in the majority of cases.
Ans:- BCD
172) SVT in neonate:-
A. Is the most common abnormal tachycardia
B. Reflect underlying congenital heart disease in the majority of cases
C. Show regular rate of 160-220 beats /min on the ECG
D. Recurrent episodes usually persist into adulthood
E. May be stopped with rapid I/V bolus of adenosine
Ans:- A E
173) Neonates suffering withdrawal from in utero exposure to narcotics may show sign of
A. Irritability
B. Vomiting
C. Photophobia
D. Hypotonia
E. Diarrhea
Ans:- ABCE
174) Clinical feature of congenital hypothyroidism diagnosed in the newborn period include:-
A. Large tongue
B. Presence of third fontenalle
C. Umbilical hernia
D. Loose stools
E. High incidence of mental retardation
Ans:- A C
175) Causes of persistent neonatal unconjugated hyperbilirubinemia after 2 weeks include:-
A. Rhesus incompatibility
B. Hypothyroidism
C. Breast milk jaundice
D. Rotor syndrome
E. Sepsis
Ans:- ABCE
176) Growth retardation babies are at increased risk of:-
A. Polycythemia
B. HMD
C. Hypoglycemia
D. Group B streptococcal infection
E. Sudden intrapartum death
Ans:- ACE
177) Oesophageal atresia is associated with:-
A. Maternal polyhydramniose
B. Vertebral anomalies
C. Diaphragmatic hernia
ABDULRAHMAN BASHIRCHILDREN HOSPITAL –BENGHAZI 39
D. Low birthweight
E. Duplex collecting system
Ans:- ABDE
178) Congenital Rubella:-
A. Is associated with cerebral calcification
B. Frequently lead to cataract
C. Is associated with VSD
D. Rarely occur following maternal infection in the third trimester
E. Should be prevented by vaccinating women found to be seronegative during the
first trimester
Ans:- B D
179) Human breast milk contain:-
A. Secretory IgA
B. Macrophages
C. Lysozyme
D. Vitamin c
E. Zinc
Ans:- ABCDE
180) The following are recommended daily requirement for 1 month old child /hg of B wt:-
A. Fluids 150-180 ml orally
B. Calories 90-115 kcal
C. Sodium 1.25-2.5 mmol
D. Protein 2.2-3.5 gm
E. Potassium 2.0-3.5 mmol
Ans:- ABCDE
181) Compared with cow's milk, human breast milk contains:
A. Less sodium.
B. Less calcium.
C. Less protein.
D. Less fat.
E. Less carbohydrate.
Ans:-A B C
182) UK infants are routinely immunised against:
A. Meningococcus type B.
B. Diphtheria.
C. Cholera.
D. Polio.Pertussis.
Ans:-B D E
183) Mature breast milk contain per 100 ml;-
A. 60-75 kcal
B. 8-10 gm of protein
C. 0.1-0.5 gm of fat
D. 0.1-0.5 gm of carbohydrate
ABDULRAHMAN BASHIRCHILDREN HOSPITAL –BENGHAZI 40
E. 5.2 mg of folic acid
Ans:- AE
MATURE BREAST
MILK
COWS
MILK
carbohydrate 7.4 gm 4.6 gm
Fat 4.2 gm 3.9 gm
protein 1.1 gm 3.4 gm
Calories 70 67
folic acid 5.2 mg 3.7 mg
184) Treatment of PDA with Indomethacin:-
A. Should be first line therapy
B. Is safe in renal failure
C. Should not be given if there is thrombocythenia
D. Should be avoided in jaundiced babies
E. Should not be given in the presence of IVH
Ans:- D E
185) Retinopathy of prematurity ;-
A. Develop in the first week of life
B. Is more likely to occur in VLBW infants
C. Is recognized complication of hypoglycemia
D. Rarely resolve spontaneously
E. May be treated effectively with Laser therapy
Ans:- B E
186) Alpha fetoprotein level from maternal serum may be helpful in diagnosing:-
A. Congenital cardiac malformation
B. Prune-belly syndrome
C. Cleft lip & palate
D. Down syndrome
E. Fetal alcohol syndrome
Ans:- D
187) A 2-week-old infant is jaundiced. Findings include weight and length at the 75th
percentile for age; icterus; with hepatosplenomegaly; total bilirubin, 6.3 mg/dL; direct
bilirubin, 5.5 mg/dL; alanine aminotransferase activity, 130 U/L; aspartate
aminotransferase activity, 143 U/L; and gamma-glutamyl transpeptidase activity, 950 U/L.
Of the following, the BEST study to evaluate the excretion of bile from the liver is
A. computed tomography of the liver
B. hepatic ultrasonography
C. hepatobiliary scintigraphy
D. measurement of galactose-1-phosphate uridyltransferase activity
E. measurement of the serum alpha1-antitrypsin level
Ans:- C
ABDULRAHMAN BASHIRCHILDREN HOSPITAL –BENGHAZI 1
188) Which of the following constellations of features BEST describes the fetal alcohol
syndrome?
A. Elfin facies, irritability, and supravalvular aortic stenosis
B. Growth deficiency with microcephaly, developmental delay, and short palpebral
fissures
C. Intrauterine growth retardation, triangular-shaped face, and clinodactyly of the
fifth finger
D. Short stature, webbed neck, and pulmonic stenosis
E. Weakness, club feet, immobile face, and inadequate respirations
Ans:- B
189) A female infant born to a 24-year-old woman has been diagnosed clinically as having
Down syndrome. The mother is concerned about her risk of having another child who has a
chromosomal abnormalityThe statement that you are MOST likely to include in your
discussion is that her risk
A. can be estimated by determination of maternal serum alpha-fetoprotein in all
future pregnancies
B. cannot be estimated until her infant's chromosome complement has been
determined
C. is increased for Down syndrome, but not for any other chromosomal abnormality
D. is no greater than that of any other woman her age
E. is not increased until she reaches the age of 35
Ans:- B
190) During delivery of an infant who has an estimated gestational age of 42 weeks, you note
that the amniotic fluid looks like pea soup and contains thick particles of meconium.
Of the following, the MOST important initial step in resuscitation of the infant is to :-
A. aspirate the gastric contents
B. determine the Apgar score initiate tracheal intubation
C. provide positive pressure ventilation
D. suction the hypopharynx
Ans:- D
191) In addition to irritability, sweating, and difficulty breathing with feeding, the symptom
that is MOST indicative of congestive heart failure in a 3-week-old infant is
A. ascites
B. cough
C. cyanosis
D. diminished feeding volume
E. pretibial edema
Ans:- D
192) The decreased incidence of enteric infections noted in breastfed infants compared with
formula-fed infants is MOST likely due to the
A. more alkaline stool pH in breastfed infants
B. nutritional benefits of human milk on the infant's immune system
C. predominance of Bacteroides and Clostridium in the gut of breastfed infants
D. presence of protective antibodies against enteric infection in human milk
ABDULRAHMAN BASHIRCHILDREN HOSPITAL –BENGHAZI 42
E. sterility of human milk
Ans:- D
193) A newborn who weighs 600 g and whose estimated gestational age is 24 weeks at birth is
admitted to the neonatal intensive care unit after successful resuscitation in the delivery
room. Arterial blood gas measurements on room air are: pH, 7.35; PCO2, 42 mm Hg; PO2,
68 mm Hg; base deficit, 2 mEq/L. Of the following, the MOST appropriate initial
management is to
A. begin intravenous vancomycin
B. begin phototherapy
C. initiate enteral feeding
D. provide bicarbonate infusion
E. provide glucose infusion
Ans:- E
194) A 3-day-old infant presents to the emergency department with vomiting, lethargy,
hypotonia, and jaundice. Physical examination reveals hepatomegaly and neurologic
depression. A full sepsis evaluation is undertaken, and the Gram stain of the cerebrospinal
fluid reveals gram-negative organisms. Of the following, the BEST additional laboratory
test to obtain is
A. erythrocyte galactose-1-phosphate
B. liver glycogen content
C. plasma insulin level
D. plasma very long-chain fatty acids.
E. stool porphyrins
Ans:- A
195) You are examining a term newborn in the nursery. His weight is 3.27 kg (50th
percentile), and his length is 50.5 cm (50th percentile). The pregnancy, labor, and delivery
were unremarkable. There are no significant findings on physical examination. The MOST
likely head circumference in this child, if it is consistent with his other growth parameters, is
A. 31 cm
B. 33 cm
C. 35 cm
D. 37 cm
E. 39 cm
Ans:- D
196) previously healthy 5-day-old male who was born at home develops bruising and melena.
The pregnancy, delivery, and postnatal course were unremarkable. The infant is
breastfeeding vigorously every 2 hours. Findings on physical examination are unremarkable
except for several large bruises. Laboratory testing reveals: hemoglobin, 81 g/L (8.1 g/dL);
white blood cell count, 9.4 x 109/L (9,400/mm3); prothrombin time, 37 seconds; partial
thromboplastin time, 98 seconds; platelet count, 242 x 109/L (242,000/mm3); and fibrinogen,
2.34 g/L (234 mg/dL). Of the following, the MOST likely cause of the bleeding is
A. disseminated intravascular coagulation
B. factor VIII deficiency hemophilia
C. liver disease
D. vitamin K deficiency
ABDULRAHMAN BASHIRCHILDREN HOSPITAL –BENGHAZI 43
E. von Willebrand disease
Ans:- D
197) A newborn whose estimated gestational age is 42 weeks is stained with meconium.
Tracheal intubation reveals meconium in the hypopharynx as well as below the vocal cords.
The infant has respiratory distress. A chest radiograph is obtained. Of the following, the
MOST likely radiographic finding is
A. coarse infiltrates
B. decreased lung volumes
C. mediastinal shift
D. pleural effusion
E. reticulogranular pattern
Ans:- A
198) A 20-year-old primigravida at 30 weeks of gestation has a blood pressure of 160/112 mm
Hg, serum total bilirubin level of 44.5 mcmol/L (2.6 mg/dL), serum alanine
aminotransferase level of 150 U/L, and platelet count of 75 x 109/L (75,000/mm3). She is
hospitalized for observation and electronic fetal heart rate monitoring. Of the following, the
MOST ominous sign of fetal distress during monitoring would be
A. early decelerations
B. increased beat-to-beat variability
C. late decelerations
D. spontaneous accelerations
E. variable decelerations
Ans:- C
199) A term newborn presents with bilious vomiting shortly after birth. Her abdomen is
distended slightly, and facial features are characteristic of Down syndrome. She has passed
a normal meconium stool. The pregnancy was complicated by polyhydramnios. Of the
following, the MOST likely diagnosis is
A. duodenal atresia
B. Hirschsprung disease
C. meconium ileus
D. midgut volvulus
E. pyloric stenosis
Ans:- A
200) An 18-year-old primigravida at 32 weeks' gestation has a blood pressure of 148/96 mm
Hg, proteinuria, oliguria, and visual disturbances. Labor is induced, and the infant is
delivered. His weight is 850 g (<10th percentile), crown-heel length is 38 cm (10th
percentile), and head circumference is 30 cm (50th percentile). Of the following, the MOST
likely complication in this infant is
A. anemia of prematurity
B. hyaline membrane disease
C. hyperglycemia
D. meconium aspiration
E. perinatal asphyxia
Ans:- E
ABDULRAHMAN BASHIRCHILDREN HOSPITAL –BENGHAZI 44
201) An infant is born at 27 weeks' gestation following a pregnancy complicated by preterm
labor that progressed despite administration of a tocolytic agent. Of the following, the
most appropriate INITIAL management is to
A. measure transcutaneous oxygen saturation
B. perform endotracheal intubation
C. place an umbilical arterial catheter
D. place the infant in an open bed warmer
E. provide nasal continuous positive airway pressure
Ans:- D
202) Of the following, the MOST important determinant of neurodevelopmental outcome of
VLBW infants is
A. antenatal obstetric management
B. infant gender
C. length of gestation
D. maternal education
E. socioeconomic status
Ans:- C
203) You are called to the newborn nursery to examine an infant who appears dysmorphic.
On physical examination, the baby is normally grown and vigorous. You note overfolded
pinnae, deviation of the nose to one side (Item Q25A), and a small chin. The feet are
maintained in dorsiflexion (Item Q25B), but can be corrected passively. You review the
pregnancy history. Of the following pregnancy complications, the one MOST likely to be
associated with this baby’s features is
A. maternal hypertension
B. polyhydramnios
C. preeclampsia
D. transverse lie
E. vaginal bleeding
Preferred Response: D
The infant described in the vignette displays the characteristic features of a deformation sequence.
Unlike malformations, which occur due to intrinsic problems within a developing structure,
deformations are due to mechanical forces acting on an otherwise normally developing embryo or
fetus. Causes of fetal deformation are many and include oligohydramnios, prolonged breech
positioning, a small or malformed uterus, fibroid tumors of the uterus, and multiple gestations.
Often, the affected infant has a pugilistic facies, with deviation of the nose to one side. Limb
positioning defects are common.
Barring any association with malformations or disruptions, the appearance of the affected infant
typically normalizes over time.
Although maternal hypertension and preeclampsia can be associated with placental
insufficiency and, ultimately, decreased fetal movement, typically they are not associated
with fetal deformation. Polyhydramnios usually is associated with unrestricted fetal movement,
and fetuses are not deformed. Vaginal bleeding has multiple potential causes, most of which are
not associated with deformation.
ABDULRAHMAN BASHIRCHILDREN HOSPITAL –BENGHAZI 45
Transverse lie is the only pregnancy complication listed that is likely to be associated with
deformation due to unusual fetal position
204) A 10-day-old infant who weighed 1,750 g at birth and whose gestational age was 34 weeks
is jaundiced. His total serum bilirubin concentration is 10.0 mg/dL and the direct fraction is
0.8 mg/dL. He is receiving intermittent orogastric feeding of expressed human milk and
supplemental parenteral nutrition. Of the following, the MOST likely explanation for these
findings is
A. Crigler-Najjar syndrome
B. jaundice due to parenteral nutrition
C. neonatal hepatitis
D. physiologic jaundice
E. pyloric stenosis
Ans:- D
205) Early hospital discharge is defined as the discharge of a newborn earlier than 48 hours
following vaginal delivery or 96 hours following cesarean delivery. Of the following, the
MOST common reason for readmission to the hospital within 7 days following an early
discharge is
A. bacterial sepsis
B. congenital heart disease
C. gastrointestinal malformation
D. hyperbilirubinemia
E. metabolic disorders
Ans:- D
206) A newborn infant is delivered by emergent cesarean section at 41 weeks' gestation
following a pregnancy complicated by a prolapsed umbilical cord and meconium-stained
amniotic fluid. At 6 hours of age, the infant has a generalized tonic-clonic seizure. Of the
following, the MOST likely explanation for this seizure is
A. hyponatremia
B. hypoxic-ischemic encephalopathy
C. intracranial hemorrhage
D. meningitis
E. pyridoxine dependency
Ans:- B
207) A 4-hour-old newborn has copious oral secretions and episodes of coughing, choking, and
cyanosis. The pregnancy was complicated by polyhydramnios. You suspect esophageal
atresia with tracheoesophageal fistula.
Of the following, the MOST helpful test to confirm the diagnosis is to
A. inject a contrast medium through an orogastric catheter and obtain a neck
radiograph
B. obtain computed tomography of the neck
C. perform flexible bronchoscopy
D. place an endotracheal tube and examine the endotracheal fluid
E. place an orogastric suction catheter and obtain a chest radiograph
Ans:- E
ABDULRAHMAN BASHIRCHILDREN HOSPITAL –BENGHAZI 46
208) A newborn is delivered by emergent cesarean section because of fetal distress following
acute abruption of the placenta. The infant is resuscitated and transferred to the nursery.
On physical examination, she appears pale, and her extremities are cold to touch. The
capillary refill is 8 seconds. Results of an arterial blood gas analysis show a Po2 of 48 mm
Hg. Of the following, the MOST likely additional finding is
A. decreased bicarbonate concentration
B. hypercalcemia
C. hyperglycemia
D. hyponatremia
E. normal anion gap
Ans:- A
209) Of the following, erythromycin prophylaxis is MOST likely to prevent ocular infection
due to
A. Chlamydia trachomatis
B. group B streptococci
C. Neisseria gonorrhoeae
D. Staphylococcus aureus
E. Trichomonas vaginalis
Ans:- C
210) A 1,300 g infant who is born at 34 weeks' gestation has a head circumference of 27 cm
and crown-heel length of 40 cm. At 48 hours of age, she is irritable, tremulous, and
inconsolable. Her systolic blood pressure is 65 mm Hg and heart rate is 180 beats/min. Her
face appears normal, and her cry is high-pitched. Cranial ultrasonography reveals bilateral
echo densities suggestive of periventricular leukomalacia. Of the following, the MOST
likely explanation for the findings in this infant is maternal exposure to
A. alcohol
B. barbiturates
C. cocaine
D. marijuana
E. opiates
Ans:- C
211) A 4.3 kg infant is delivered to a woman whose diabetes mellitus is poorly controlled. Of
the following, the MOST likely neonatal manifestation of maternal diabetes is
A. diabetic ketoacidosis
B. Hirschsprung disease
C. hypercalcemia
D. polycythemia
E. renal vein thrombosis
Ans:- D
212) Of the following, the MOST helpful finding to distinguish GBS pneumonia from RDS is
A. a normal C-reactive protein level
B. an elevated erythrocyte sedimentation rate
C. diffuse alveolar infiltrates on chest radiography
D. increased ratio of bands to segmented neutrophils
ABDULRAHMAN BASHIRCHILDREN HOSPITAL –BENGHAZI 47
E. persistent hypoxemia on blood gas analysis
Ans:- D
213) An 18-hour-old infant of a diabetic mother develops abdominal distension. Physical
examination reveals a protuberant, firm, but nontender abdomen; patent anus; and no
grossly visible anomalies. The infant has passed no meconium stool since birth. A supine
abdominal radiograph reveals multiple dilated loops of intestine.
Of the following, a contrast enema would MOST likely confirm a diagnosis of
A. atresia of the colon
B. Hirschsprung disease
C. hypoplastic left colon syndrome
D. meconium ileus
E. midgut volvulus with malrotation
Ans:- C
214) You are attending the emergency delivery by cesarean section of a primiparous woman.
The gestation was complicated by pregnancy-induced hypertension. Deep variable fetal
heart rate decelerations were noted during labor. At delivery, the infant is acrocyanotic with
poor tone; spontaneous movement and minimal respiratory effort are present. Of the
following, your INITIAL management is to
A. ascertain the heart rate and assign a 1-minute Apgar score
B. begin tactile stimulation and provide blow-by oxygen supplementation
C. dry all skin surfaces and clear the oropharynx
D. initiate bag-mask ventilation
E. insert an umbilical catheter and administer naloxone
Ans:-C
215) A vigorous, normal-appearing term male newborn has not voided by 18 hours after
delivery. Perinatal history is negative for maternal illness or medications. Amniotic fluid
volume was reportedly normal, and the delivery was uneventful, with Apgar scores of 6 and
9 at 1 and 5 minutes, respectively. Of the following, the MOST likely reason why this 18-
hour-old infant has not voided is
A. bilateral ureteropelvic junction obstruction
B. intravascular volume depletion
C. neurogenic bladder
D. posterior urethral valve
E. undocumented void in the delivery room
Ans:- E
216) A 2-week-old neonate who was born at 32 weeks’ gestation has recovered from
respiratory distress syndrome. He has been tolerating increasing volumes of enteral feedings
via gavage. Over the past several feedings, abdominal distension, gastric residuals, and
stools that are positive for blood have been noted. Of the following, the radiographic finding
MOST supportive of the diagnosis of necrotizing enterocolitis is
A. absence of luminal bowel gas
B. generalized bowel distension
C. intraperitoneal fluid
D. pneumatosis intestinalis
E. thickening of the bowel wall
ABDULRAHMAN BASHIRCHILDREN HOSPITAL –BENGHAZI 48
Ans:- D
217) A term infant is placed under a radiant warmer, the skin is dried, and the oropharynx
and nose are suctioned. After tactile stimulation, there is minimal respiratory effort, dusky
color, and a heart rate of 86 beats/min. Bag/mask ventilation is performed for 30 seconds
with 100% oxygen at a rate of 40 to 60 breaths/min. The heart rate increases to 100
beats/min. Of the following, the NEXT best step is to:
A. administer sodium bicarbonate
B. continue bag/mask ventilation at a rate of 20 to 40 breaths/min
C .continue ventilation and begin chest compressions
D. observes for spontaneous respiration and discontinues ventilation
E. perform endotracheal intubation
Ans:-D
218) A 900 g male infant is delivered vaginally to a woman who had no prenatal care.
Of the following, the physical finding that is MOST consistent with prematurity rather than
intrauterine growth restriction is
A. creases over entire sole of foot
B. descended testes with deep rugae of the scrotum
C. formed and firm pinna with instant recoil
D. gelatinous translucent skin
E. raised areola and 3 mm breast buds
Ans:-D
219) A 1-day-old infant develops bilious vomiting and gastric distension. She has been afebrile
and has been passing meconium-laden stools. Of the following, the most appropriate
INITIAL step in the management of this infant is
A. abdominal radiography to look for the “double-bubble” sign
B .culture of a catheterized urine specimen
C .insertion of a rectal tube for decompression
D .placement of a nasogastric tube and initiation of intravenous fluid therapy
E. upper gastrointestinal radiographic series to look for malrotation of the small bowel
Ans:- D
220) While performing ultrasonography on a 31-week fetus, an obstetrician notes that the
fetal heart rate ranges from 62 to 66 beats/min. Fetal growth appears normal, and no
structural cardiac anomalies are identified. Fetal echocardiography reveals that the fetal
atria appear to be contracting at 140 beats/min, with a ventricular rate of 65 beats/min. Of
the following, the NEXT step in the management of this infant is to:-
A. administer beta-agonist drug therapy to the mother
B. assess the cardiac status of the infant following labor and delivery
C. counsel the parents that intrauterine fetal death is likely
D. perform amniocentesis to confirm lung maturity and if mature, perform immediate
cesarean section
E. repeat the fetal echocardiography and fetal ultrasonography in 1 week
Ans:-E
221) The mother of a 2-week-old infant is concerned because her baby has not had a stool in 7 days.
She has been exclusively breastfeeding him every 2 to 3 hours since her milk came in, but the
ABDULRAHMAN BASHIRCHILDREN HOSPITAL –BENGHAZI 49
baby has only passed flatus. The infant is gaining weight well. Results of physical examination
are normal.
Of the following, the best course of INITIAL management for this infant
is……………………………………
222) Term infant is cyanotic and requires intubation. Findings include: heart rate, 175
beats/min; blood pressure, 60/30 mm Hg; increased right ventricular activity; single S2;
short systolic murmur; and equal arm and leg pulses; chest radiography, normal heart size
and pulmonary congestion. Arterial blood gases (right radial artery on 100% FIO2): pH,
7.31; PO2, 43 torr; PCO2, 48 torr. Of the following, the MOST likely diagnosis is:-
A. hyaline membrane disease
B. hypoplastic left heart
C. intrauterine constriction of the ductus arteriosus
D. tetralogy of Fallot
E. total anomalous pulmonary venous connection
Ans:-E
223) A 7-day-old infant has copious purulent discharge from both eyes. The 17-year-old
mother currently complains of a yellowish vaginal discharge. The only medications received
by the infant were vitamin K and topical erythromycin prophylaxis following delivery.
Giemsa stain of a conjunctival scraping reveals intracytoplasmic inclusions. After obtaining
appropriate diagnostic studies, the BEST management includes treatment with :-
A. oral erythromycin
B. oral penicillin
C. topical erythromycin
D. topical gentamicin
E. topical sulfonamide
Ans:-A
224) Of the following, the condition that is MOST likely to present with seizures during the
first 24 hours of life is
A .fetal alcohol syndrome
B .herpes simplex infection
C. hypoxic-ischemic encephalopathy
D. organic acidemia
E. urea cycle defect
Ans:-C
225) Maternal serum alpha fetoprotein measured at 12 weeks gestation
A. If normal then neural tube defect are excluded
B. Is increased in twin pregnancy
C. Is reduced in Turner syndrome
D. Is increased in Down syndrome
E. AFT is the main blood protein fraction in the first trimester
Ans :- BE
Serum a fetoprotein is usually measured at 16-18 week gestation, normal screening would not
rule out neural tube defect or any other lesion such as twins, Turner's syndrome, Exomphalus,
which can all have raised value .
Newborn mcq
Newborn mcq
Newborn mcq
Newborn mcq
Newborn mcq
Newborn mcq
Newborn mcq
Newborn mcq

More Related Content

What's hot

What's hot (20)

Respiratory distress in newborn
Respiratory distress in newbornRespiratory distress in newborn
Respiratory distress in newborn
 
HIE
HIEHIE
HIE
 
Respiratory Distress in The Newborn
Respiratory Distress in The NewbornRespiratory Distress in The Newborn
Respiratory Distress in The Newborn
 
Neonatal Apnea
Neonatal ApneaNeonatal Apnea
Neonatal Apnea
 
Neurological Examination of an infant
Neurological Examination  of an infantNeurological Examination  of an infant
Neurological Examination of an infant
 
Respiratory distress syndrome
Respiratory distress syndromeRespiratory distress syndrome
Respiratory distress syndrome
 
Respiratory Distress in New born
Respiratory Distress in New bornRespiratory Distress in New born
Respiratory Distress in New born
 
Meconium Aspiration Syndrome
Meconium Aspiration SyndromeMeconium Aspiration Syndrome
Meconium Aspiration Syndrome
 
MCQ IN PEDIATRICS
MCQ IN PEDIATRICSMCQ IN PEDIATRICS
MCQ IN PEDIATRICS
 
Meconium aspiration syndrome
Meconium aspiration syndromeMeconium aspiration syndrome
Meconium aspiration syndrome
 
Practical pediatric quiz - Kaun Banega Winner
Practical pediatric quiz - Kaun Banega WinnerPractical pediatric quiz - Kaun Banega Winner
Practical pediatric quiz - Kaun Banega Winner
 
Neonatal mechanical ventilation
Neonatal mechanical ventilationNeonatal mechanical ventilation
Neonatal mechanical ventilation
 
Meconium Aspiration syndrome.pptx
Meconium Aspiration syndrome.pptxMeconium Aspiration syndrome.pptx
Meconium Aspiration syndrome.pptx
 
RIPE 2012 Pediatrics OSCE
RIPE 2012 Pediatrics OSCERIPE 2012 Pediatrics OSCE
RIPE 2012 Pediatrics OSCE
 
Pphn
PphnPphn
Pphn
 
Necrotising Enterocolitis(NEC)
Necrotising Enterocolitis(NEC)Necrotising Enterocolitis(NEC)
Necrotising Enterocolitis(NEC)
 
Neonatal sepsis...ppt
Neonatal sepsis...pptNeonatal sepsis...ppt
Neonatal sepsis...ppt
 
Croup in children
Croup in childrenCroup in children
Croup in children
 
Intraventricular hemorrhage
Intraventricular hemorrhageIntraventricular hemorrhage
Intraventricular hemorrhage
 
Respiratory distress in newborn
Respiratory distress in newbornRespiratory distress in newborn
Respiratory distress in newborn
 

Similar to Newborn mcq

Similar to Newborn mcq (20)

Congenital malformations of the central nervous system
 Congenital malformations of the central nervous system Congenital malformations of the central nervous system
Congenital malformations of the central nervous system
 
Ectopic Pregnancy
Ectopic PregnancyEctopic Pregnancy
Ectopic Pregnancy
 
ETAS_MCQ_09 pediatric dermatology
ETAS_MCQ_09 pediatric dermatologyETAS_MCQ_09 pediatric dermatology
ETAS_MCQ_09 pediatric dermatology
 
Birthinjuriesandicterusneonatarum
BirthinjuriesandicterusneonatarumBirthinjuriesandicterusneonatarum
Birthinjuriesandicterusneonatarum
 
My Clouding Cornea
My Clouding CorneaMy Clouding Cornea
My Clouding Cornea
 
Hydatidiform mole
Hydatidiform moleHydatidiform mole
Hydatidiform mole
 
Neonatal Infections
Neonatal InfectionsNeonatal Infections
Neonatal Infections
 
Coccidian parasite
Coccidian parasiteCoccidian parasite
Coccidian parasite
 
Handbook of neonatology
Handbook of neonatologyHandbook of neonatology
Handbook of neonatology
 
Anaesthesia for foetal surgeries
Anaesthesia for foetal surgeriesAnaesthesia for foetal surgeries
Anaesthesia for foetal surgeries
 
Radiology 5th year, 8th & 9th lectures (Dr. Nasrin Alatrushi)
Radiology 5th year, 8th & 9th lectures (Dr. Nasrin Alatrushi)Radiology 5th year, 8th & 9th lectures (Dr. Nasrin Alatrushi)
Radiology 5th year, 8th & 9th lectures (Dr. Nasrin Alatrushi)
 
Empyema dr yusuf imran
Empyema dr yusuf imranEmpyema dr yusuf imran
Empyema dr yusuf imran
 
Hydrocephalus
HydrocephalusHydrocephalus
Hydrocephalus
 
Birth injuries and icterus neonatarum
Birth injuries and icterus neonatarumBirth injuries and icterus neonatarum
Birth injuries and icterus neonatarum
 
Skin lesion in neonate
Skin lesion in neonateSkin lesion in neonate
Skin lesion in neonate
 
Abortion presentation
Abortion presentationAbortion presentation
Abortion presentation
 
quiz pg.pptx
quiz pg.pptxquiz pg.pptx
quiz pg.pptx
 
Meconium Aspiration. In neonates pptx...
Meconium Aspiration. In neonates pptx...Meconium Aspiration. In neonates pptx...
Meconium Aspiration. In neonates pptx...
 
Ganyang MCQ Infectious dss
Ganyang MCQ Infectious dssGanyang MCQ Infectious dss
Ganyang MCQ Infectious dss
 
Management of Fetal hydrocephalus
Management of Fetal hydrocephalus Management of Fetal hydrocephalus
Management of Fetal hydrocephalus
 

Recently uploaded

Vip Call Girls Anna Salai Chennai 👉 8250192130 ❣️💯 Top Class Girls Available
Vip Call Girls Anna Salai Chennai 👉 8250192130 ❣️💯 Top Class Girls AvailableVip Call Girls Anna Salai Chennai 👉 8250192130 ❣️💯 Top Class Girls Available
Vip Call Girls Anna Salai Chennai 👉 8250192130 ❣️💯 Top Class Girls AvailableNehru place Escorts
 
♛VVIP Hyderabad Call Girls Chintalkunta🖕7001035870🖕Riya Kappor Top Call Girl ...
♛VVIP Hyderabad Call Girls Chintalkunta🖕7001035870🖕Riya Kappor Top Call Girl ...♛VVIP Hyderabad Call Girls Chintalkunta🖕7001035870🖕Riya Kappor Top Call Girl ...
♛VVIP Hyderabad Call Girls Chintalkunta🖕7001035870🖕Riya Kappor Top Call Girl ...astropune
 
(👑VVIP ISHAAN ) Russian Call Girls Service Navi Mumbai🖕9920874524🖕Independent...
(👑VVIP ISHAAN ) Russian Call Girls Service Navi Mumbai🖕9920874524🖕Independent...(👑VVIP ISHAAN ) Russian Call Girls Service Navi Mumbai🖕9920874524🖕Independent...
(👑VVIP ISHAAN ) Russian Call Girls Service Navi Mumbai🖕9920874524🖕Independent...Taniya Sharma
 
💎VVIP Kolkata Call Girls Parganas🩱7001035870🩱Independent Girl ( Ac Rooms Avai...
💎VVIP Kolkata Call Girls Parganas🩱7001035870🩱Independent Girl ( Ac Rooms Avai...💎VVIP Kolkata Call Girls Parganas🩱7001035870🩱Independent Girl ( Ac Rooms Avai...
💎VVIP Kolkata Call Girls Parganas🩱7001035870🩱Independent Girl ( Ac Rooms Avai...Taniya Sharma
 
Call Girls Colaba Mumbai ❤️ 9920874524 👈 Cash on Delivery
Call Girls Colaba Mumbai ❤️ 9920874524 👈 Cash on DeliveryCall Girls Colaba Mumbai ❤️ 9920874524 👈 Cash on Delivery
Call Girls Colaba Mumbai ❤️ 9920874524 👈 Cash on Deliverynehamumbai
 
Russian Call Girls in Pune Riya 9907093804 Short 1500 Night 6000 Best call gi...
Russian Call Girls in Pune Riya 9907093804 Short 1500 Night 6000 Best call gi...Russian Call Girls in Pune Riya 9907093804 Short 1500 Night 6000 Best call gi...
Russian Call Girls in Pune Riya 9907093804 Short 1500 Night 6000 Best call gi...Miss joya
 
(Rocky) Jaipur Call Girl - 9521753030 Escorts Service 50% Off with Cash ON De...
(Rocky) Jaipur Call Girl - 9521753030 Escorts Service 50% Off with Cash ON De...(Rocky) Jaipur Call Girl - 9521753030 Escorts Service 50% Off with Cash ON De...
(Rocky) Jaipur Call Girl - 9521753030 Escorts Service 50% Off with Cash ON De...indiancallgirl4rent
 
Kesar Bagh Call Girl Price 9548273370 , Lucknow Call Girls Service
Kesar Bagh Call Girl Price 9548273370 , Lucknow Call Girls ServiceKesar Bagh Call Girl Price 9548273370 , Lucknow Call Girls Service
Kesar Bagh Call Girl Price 9548273370 , Lucknow Call Girls Servicemakika9823
 
Artifacts in Nuclear Medicine with Identifying and resolving artifacts.
Artifacts in Nuclear Medicine with Identifying and resolving artifacts.Artifacts in Nuclear Medicine with Identifying and resolving artifacts.
Artifacts in Nuclear Medicine with Identifying and resolving artifacts.MiadAlsulami
 
Call Girls Service Navi Mumbai Samaira 8617697112 Independent Escort Service ...
Call Girls Service Navi Mumbai Samaira 8617697112 Independent Escort Service ...Call Girls Service Navi Mumbai Samaira 8617697112 Independent Escort Service ...
Call Girls Service Navi Mumbai Samaira 8617697112 Independent Escort Service ...Call girls in Ahmedabad High profile
 
Russian Call Girls in Bangalore Manisha 7001305949 Independent Escort Service...
Russian Call Girls in Bangalore Manisha 7001305949 Independent Escort Service...Russian Call Girls in Bangalore Manisha 7001305949 Independent Escort Service...
Russian Call Girls in Bangalore Manisha 7001305949 Independent Escort Service...narwatsonia7
 
Call Girls Service Surat Samaira ❤️🍑 8250192130 👄 Independent Escort Service ...
Call Girls Service Surat Samaira ❤️🍑 8250192130 👄 Independent Escort Service ...Call Girls Service Surat Samaira ❤️🍑 8250192130 👄 Independent Escort Service ...
Call Girls Service Surat Samaira ❤️🍑 8250192130 👄 Independent Escort Service ...CALL GIRLS
 
Call Girls Service Pune Vaishnavi 9907093804 Short 1500 Night 6000 Best call ...
Call Girls Service Pune Vaishnavi 9907093804 Short 1500 Night 6000 Best call ...Call Girls Service Pune Vaishnavi 9907093804 Short 1500 Night 6000 Best call ...
Call Girls Service Pune Vaishnavi 9907093804 Short 1500 Night 6000 Best call ...Miss joya
 
Call Girl Coimbatore Prisha☎️ 8250192130 Independent Escort Service Coimbatore
Call Girl Coimbatore Prisha☎️  8250192130 Independent Escort Service CoimbatoreCall Girl Coimbatore Prisha☎️  8250192130 Independent Escort Service Coimbatore
Call Girl Coimbatore Prisha☎️ 8250192130 Independent Escort Service Coimbatorenarwatsonia7
 
VIP Mumbai Call Girls Hiranandani Gardens Just Call 9920874524 with A/C Room ...
VIP Mumbai Call Girls Hiranandani Gardens Just Call 9920874524 with A/C Room ...VIP Mumbai Call Girls Hiranandani Gardens Just Call 9920874524 with A/C Room ...
VIP Mumbai Call Girls Hiranandani Gardens Just Call 9920874524 with A/C Room ...Garima Khatri
 
Bangalore Call Girls Marathahalli 📞 9907093804 High Profile Service 100% Safe
Bangalore Call Girls Marathahalli 📞 9907093804 High Profile Service 100% SafeBangalore Call Girls Marathahalli 📞 9907093804 High Profile Service 100% Safe
Bangalore Call Girls Marathahalli 📞 9907093804 High Profile Service 100% Safenarwatsonia7
 
Call Girls Yelahanka Bangalore 📲 9907093804 💞 Full Night Enjoy
Call Girls Yelahanka Bangalore 📲 9907093804 💞 Full Night EnjoyCall Girls Yelahanka Bangalore 📲 9907093804 💞 Full Night Enjoy
Call Girls Yelahanka Bangalore 📲 9907093804 💞 Full Night Enjoynarwatsonia7
 
Russian Escorts Girls Nehru Place ZINATHI 🔝9711199012 ☪ 24/7 Call Girls Delhi
Russian Escorts Girls  Nehru Place ZINATHI 🔝9711199012 ☪ 24/7 Call Girls DelhiRussian Escorts Girls  Nehru Place ZINATHI 🔝9711199012 ☪ 24/7 Call Girls Delhi
Russian Escorts Girls Nehru Place ZINATHI 🔝9711199012 ☪ 24/7 Call Girls DelhiAlinaDevecerski
 
Call Girl Number in Panvel Mumbai📲 9833363713 💞 Full Night Enjoy
Call Girl Number in Panvel Mumbai📲 9833363713 💞 Full Night EnjoyCall Girl Number in Panvel Mumbai📲 9833363713 💞 Full Night Enjoy
Call Girl Number in Panvel Mumbai📲 9833363713 💞 Full Night Enjoybabeytanya
 

Recently uploaded (20)

Vip Call Girls Anna Salai Chennai 👉 8250192130 ❣️💯 Top Class Girls Available
Vip Call Girls Anna Salai Chennai 👉 8250192130 ❣️💯 Top Class Girls AvailableVip Call Girls Anna Salai Chennai 👉 8250192130 ❣️💯 Top Class Girls Available
Vip Call Girls Anna Salai Chennai 👉 8250192130 ❣️💯 Top Class Girls Available
 
♛VVIP Hyderabad Call Girls Chintalkunta🖕7001035870🖕Riya Kappor Top Call Girl ...
♛VVIP Hyderabad Call Girls Chintalkunta🖕7001035870🖕Riya Kappor Top Call Girl ...♛VVIP Hyderabad Call Girls Chintalkunta🖕7001035870🖕Riya Kappor Top Call Girl ...
♛VVIP Hyderabad Call Girls Chintalkunta🖕7001035870🖕Riya Kappor Top Call Girl ...
 
(👑VVIP ISHAAN ) Russian Call Girls Service Navi Mumbai🖕9920874524🖕Independent...
(👑VVIP ISHAAN ) Russian Call Girls Service Navi Mumbai🖕9920874524🖕Independent...(👑VVIP ISHAAN ) Russian Call Girls Service Navi Mumbai🖕9920874524🖕Independent...
(👑VVIP ISHAAN ) Russian Call Girls Service Navi Mumbai🖕9920874524🖕Independent...
 
💎VVIP Kolkata Call Girls Parganas🩱7001035870🩱Independent Girl ( Ac Rooms Avai...
💎VVIP Kolkata Call Girls Parganas🩱7001035870🩱Independent Girl ( Ac Rooms Avai...💎VVIP Kolkata Call Girls Parganas🩱7001035870🩱Independent Girl ( Ac Rooms Avai...
💎VVIP Kolkata Call Girls Parganas🩱7001035870🩱Independent Girl ( Ac Rooms Avai...
 
Call Girls Colaba Mumbai ❤️ 9920874524 👈 Cash on Delivery
Call Girls Colaba Mumbai ❤️ 9920874524 👈 Cash on DeliveryCall Girls Colaba Mumbai ❤️ 9920874524 👈 Cash on Delivery
Call Girls Colaba Mumbai ❤️ 9920874524 👈 Cash on Delivery
 
Russian Call Girls in Pune Riya 9907093804 Short 1500 Night 6000 Best call gi...
Russian Call Girls in Pune Riya 9907093804 Short 1500 Night 6000 Best call gi...Russian Call Girls in Pune Riya 9907093804 Short 1500 Night 6000 Best call gi...
Russian Call Girls in Pune Riya 9907093804 Short 1500 Night 6000 Best call gi...
 
(Rocky) Jaipur Call Girl - 9521753030 Escorts Service 50% Off with Cash ON De...
(Rocky) Jaipur Call Girl - 9521753030 Escorts Service 50% Off with Cash ON De...(Rocky) Jaipur Call Girl - 9521753030 Escorts Service 50% Off with Cash ON De...
(Rocky) Jaipur Call Girl - 9521753030 Escorts Service 50% Off with Cash ON De...
 
Kesar Bagh Call Girl Price 9548273370 , Lucknow Call Girls Service
Kesar Bagh Call Girl Price 9548273370 , Lucknow Call Girls ServiceKesar Bagh Call Girl Price 9548273370 , Lucknow Call Girls Service
Kesar Bagh Call Girl Price 9548273370 , Lucknow Call Girls Service
 
Artifacts in Nuclear Medicine with Identifying and resolving artifacts.
Artifacts in Nuclear Medicine with Identifying and resolving artifacts.Artifacts in Nuclear Medicine with Identifying and resolving artifacts.
Artifacts in Nuclear Medicine with Identifying and resolving artifacts.
 
Call Girls Service Navi Mumbai Samaira 8617697112 Independent Escort Service ...
Call Girls Service Navi Mumbai Samaira 8617697112 Independent Escort Service ...Call Girls Service Navi Mumbai Samaira 8617697112 Independent Escort Service ...
Call Girls Service Navi Mumbai Samaira 8617697112 Independent Escort Service ...
 
sauth delhi call girls in Bhajanpura 🔝 9953056974 🔝 escort Service
sauth delhi call girls in Bhajanpura 🔝 9953056974 🔝 escort Servicesauth delhi call girls in Bhajanpura 🔝 9953056974 🔝 escort Service
sauth delhi call girls in Bhajanpura 🔝 9953056974 🔝 escort Service
 
Russian Call Girls in Bangalore Manisha 7001305949 Independent Escort Service...
Russian Call Girls in Bangalore Manisha 7001305949 Independent Escort Service...Russian Call Girls in Bangalore Manisha 7001305949 Independent Escort Service...
Russian Call Girls in Bangalore Manisha 7001305949 Independent Escort Service...
 
Call Girls Service Surat Samaira ❤️🍑 8250192130 👄 Independent Escort Service ...
Call Girls Service Surat Samaira ❤️🍑 8250192130 👄 Independent Escort Service ...Call Girls Service Surat Samaira ❤️🍑 8250192130 👄 Independent Escort Service ...
Call Girls Service Surat Samaira ❤️🍑 8250192130 👄 Independent Escort Service ...
 
Call Girls Service Pune Vaishnavi 9907093804 Short 1500 Night 6000 Best call ...
Call Girls Service Pune Vaishnavi 9907093804 Short 1500 Night 6000 Best call ...Call Girls Service Pune Vaishnavi 9907093804 Short 1500 Night 6000 Best call ...
Call Girls Service Pune Vaishnavi 9907093804 Short 1500 Night 6000 Best call ...
 
Call Girl Coimbatore Prisha☎️ 8250192130 Independent Escort Service Coimbatore
Call Girl Coimbatore Prisha☎️  8250192130 Independent Escort Service CoimbatoreCall Girl Coimbatore Prisha☎️  8250192130 Independent Escort Service Coimbatore
Call Girl Coimbatore Prisha☎️ 8250192130 Independent Escort Service Coimbatore
 
VIP Mumbai Call Girls Hiranandani Gardens Just Call 9920874524 with A/C Room ...
VIP Mumbai Call Girls Hiranandani Gardens Just Call 9920874524 with A/C Room ...VIP Mumbai Call Girls Hiranandani Gardens Just Call 9920874524 with A/C Room ...
VIP Mumbai Call Girls Hiranandani Gardens Just Call 9920874524 with A/C Room ...
 
Bangalore Call Girls Marathahalli 📞 9907093804 High Profile Service 100% Safe
Bangalore Call Girls Marathahalli 📞 9907093804 High Profile Service 100% SafeBangalore Call Girls Marathahalli 📞 9907093804 High Profile Service 100% Safe
Bangalore Call Girls Marathahalli 📞 9907093804 High Profile Service 100% Safe
 
Call Girls Yelahanka Bangalore 📲 9907093804 💞 Full Night Enjoy
Call Girls Yelahanka Bangalore 📲 9907093804 💞 Full Night EnjoyCall Girls Yelahanka Bangalore 📲 9907093804 💞 Full Night Enjoy
Call Girls Yelahanka Bangalore 📲 9907093804 💞 Full Night Enjoy
 
Russian Escorts Girls Nehru Place ZINATHI 🔝9711199012 ☪ 24/7 Call Girls Delhi
Russian Escorts Girls  Nehru Place ZINATHI 🔝9711199012 ☪ 24/7 Call Girls DelhiRussian Escorts Girls  Nehru Place ZINATHI 🔝9711199012 ☪ 24/7 Call Girls Delhi
Russian Escorts Girls Nehru Place ZINATHI 🔝9711199012 ☪ 24/7 Call Girls Delhi
 
Call Girl Number in Panvel Mumbai📲 9833363713 💞 Full Night Enjoy
Call Girl Number in Panvel Mumbai📲 9833363713 💞 Full Night EnjoyCall Girl Number in Panvel Mumbai📲 9833363713 💞 Full Night Enjoy
Call Girl Number in Panvel Mumbai📲 9833363713 💞 Full Night Enjoy
 

Newborn mcq

  • 2. ‫اھﺪاء‬ ‫ﯾﻧﺎل‬ ‫ان‬ ‫ﷲ‬ ‫ﻣن‬ ‫راﺟﯾﺎ‬ ‫اﻻطﻔﺎل‬ ‫اطﺑﺎء‬ ‫ﺟﻣﯾﻊ‬ ‫اﻟﻰ‬ ‫اﻟﻌﻣل‬ ‫ھذا‬ ‫اﻗدم‬ ‫ورﺿﺎﺋﮭم‬ ‫اﻋﺟﺎﺑﮭم‬،‫واﻟﻣﻐﻔرة‬ ‫ﺑﺎﻟرﺣﻣﺔ‬ ‫اﻟدﻋﺎء‬ ‫اﻻ‬ ‫وﻻاطﻠب‬ ‫روﺣﻰ‬ ‫وﺗؤام‬ ‫وﻋﺿدى‬ ‫ﻻﺧﻰ‬"‫اﺣﻣد‬".‫ﻟﮫ‬ ‫اﻏﻔر‬ ‫اﻟﻠﮭم‬ ‫ﺟﻧﺎﻧك‬ ‫ﻓﺳﯾﺢ‬ ‫واﺳﻛﻧﺔ‬ ‫وارﺣﻣﺔ‬. ‫اﻣﯾن‬ Create PDF files without this message by purchasing novaPDF printer (http://www.novapdf.com)
  • 3. ABDULRAHMAN BASHIRCHILDREN HOSPITAL –BENGHAZI 1 1) Newborn infants commonly have:- A. Capillary hemangioma on the forehead. B. Posterior cranial fontenalle. C. Metopic sutures. D. Impalpable coronal sutures. E. Skin tag in front of the ear. Ans:- ABC 2) The following should be investigated in five day old baby:- A. Erythema Toxicum B. Cloudy cornea C. Divarication of rectii D. Subconjunctival hemorrhage E. Preauricular skin tags Ans:-BE 3) During morning rounds in the newborn nursery, you examine a healthy infant who has blotchy erythematous macules that are 2 to 3 cm in diameter. The macules are scattered over the trunk, face, and proximal extremities; the palms and soles are spared. Each macule has a 1- to 3-mm central vesicle or pustule.Of the following, the MOST likely additional finding in this patient is A. pigmented macules located at sites of resolving pustules B. presence of lesions at birth C. pustules coalescing into bullae D. Wright stain of a smear of the vesicopustular contents revealing a predominance of eosinophils E. Wright stain of a smear of the vesicopustular contents revealing a predominance of polymorphonuclear neutrophils Preferred Response: D The infant described in the vignette exhibits the classic presentation of erythema toxicum neonatorum. Tiny vesicles or pustules arise from blotchy erythematous macules, with lesions characteristically appearing at 24 to 48 hours after birth. The pustules do not coalesce into bullae. Wright staining of the pustular contents reveals a predominance of eosinophils, not neutrophils. Because the lesions are seen in healthy infants, it has been suggested that this benign condition be renamed; suggested names include “benign neonatal rash” or “benign erythema neonatorum.” Transient neonatal pustular melanosis (TNPM) is another well-recognized benign dermal eruption of infancy in which pustular lesions spontaneously resolve into transient pigmented macules . TNPM may be present at birth, and examination of pustular contents reveals a predominance of neutrophils. Infantile acropustulosis is a chronic or recurring benign condition manifested by intensely pruritic pustules on hands and feet. Characteristic papules and pustules may coalesce into bullae.
  • 4. ABDULRAHMAN BASHIRCHILDREN HOSPITAL –BENGHAZI 2 Infantile acropustulosis frequently is confused with scabies infestation. The lesions resolve spontaneously at 1 to 2 years of age. 4) You are examining a newborn who is the product of an uneventful pregnancy, labor, and delivery. Apgar scores were 9 at both 1 and 5 minutes. Findings on the initial physical examination are unremarkable except for the presence of vesicopustules and frecklelike macules (Item Q33A), some of which have a collarette of surrounding scale. Wright stain of a smear of the vesicopustular contents reveals a predominance of polymorphonuclear neutrophils.Of the following, the MOST likely diagnosis is A. congenital candidiasis B. erythema toxicum neonatorum C. infantile acropustulosis D. miliaria rubra E. transient neonatal pustular melanosis Preferred Response: E Characteristic lesions of transient neonatal pustular melanosis (TNPM) may be present at birth as vesicles, pustules, or ruptured vesicles or pustules that have a collarette of surrounding scale. Pigmented macules (Item C33A) often develop at the sites of resolving pustules or vesicles. Primary lesions usually disappear by 5 days of age; the secondary pigmented lesions may remain up to 3 months. TNPM occurs more commonly in African-American infants. Lesions can occur on palms and soles. Pustular contents reveal a predominance of neutrophils on Wright stain examination, as reported for the newborn in the vignette. Infants who have congenital cutaneous candidiasis may present with scaling, erythematous papules and pustules (Item C33B) at birth. Candida albicans can penetrate through the amnion and chorion to cause congenital infections. Scrapings from lesions prepared with potassium hydroxide document pseudohyphae (Item C33C) or budding yeast. Term infants who have erythema toxicum neonatorum exhibit vesicopustular lesions (Item C33D) that usually overlie erythematous macules. Lesions of erythema toxicum rarely are present at birth, and Wright stain of smears of pustular contents reveals a predominance of eosinophils. Infantile acropustulosis presents as pustules or vesicles (Item C33E) localized to the hands and feet. It may be present at birth but more commonly develops in the first weeks and months after birth, possibly continuing or recurring throughout infancy and early childhood. Lesions are very similar to those of infantile scabies infestation. Pustular contents may reveal prominent neutrophils and occasional eosinophils without evidence of the mites, ova, or feces seen in scabies. An absence of hyperpigmentation in resolving lesions and a prolonged or recurring course distinguishes infantile acropustulosis from TNPM. Miliaria rubra (prickly heat or heat rash) is caused by intraepidermal obstruction of the sweat ducts. A secondary local inflammatory response is responsible for the erythema (Item C33F) associated with the papules and vesicles. Miliaria rubra occurs later than miliaria crystallina, usually beyond thefirst postnatal week. Hyperpigmented, frecklelike lesions are not expected in miliaria rubra. 5) The following is true regarding changes in the fetal cardiovascular system after birth: A. There is normally immediate closure of the ductus arteriosus
  • 5. ABDULRAHMAN BASHIRCHILDREN HOSPITAL –BENGHAZI 3 B. Hypoxia-induced vasoconstriction is the mechanism of closure of the ductus arteriosus C. The ligamentum teres is the remnant of the umbilical vein D. Regression of right ventricular hypertrophy occurs postnatally E. Inferior vena caval pressure falls after birth Ans:-CDE Occlusion of the umbilical cord removes this low resistance capillary bed from the circulation; breathing results in a marked decrease in pulmonary vascular resistance, hence there is increased pulmonary blood flow returning to the left atrium raising the pressure in the left atrium causing the foramen ovale to close. As pressure in the systemic circulation rises, shunt through the ductus arteriosus reverses. As the pO2 rises, synthesis of bradykinin and prostacyclins is inhibited, thus causing closure of the ductus arteriosus. The ductus arteriosus can take up to 3 months to close in normal neonates. 6) Neonatal RDS:- A. Seen in most babies of birth weight < 2.5 kg. B. More common in infants of diabetic mothers. C. Associated with prolonged rupture of membrane. D. Less sever in babies of Afro-Caribiean origin than Caucasians. E. Exacerbated by hypothermia. Ans:-BDE 7) The following are causes of generalized hypotonia in 2 days old infants:- A. Prematurity B. Hypothyroidism C. Myotonic dystrophy D. Spinal dysraphyism E. Anterior horn cell disease Ans:-ACE 8) Regarding surfactant:- A. Production begins at 30 weeks B. It is produced by Type II pneumocytes in the walls of the bronchi C. Testosterone stimulates surfactant production D. Production is increased during a stressful event like hypothermia E. Betamathasone given to the mother improves surfactant production in the premature baby Ans:- E Surfactant production begins at 20-22 weeks. It is produced by Type II pneumocytes which are in the walls of the alveoli. The hormones testosterone and insulin inhibit surfactant production; hence hyaline membrane disease is more common in males than females and more common in infants of diabetic mothers. Surfactant production is suppressed if the baby is hypothermic, hypoxic, acidotic or hypoglycemic. Although dexamethasone is more commonly used, betamethasone has an identical effect on lung maturation 9) Concerning fetal lung development:-
  • 6. ABDULRAHMAN BASHIRCHILDREN HOSPITAL –BENGHAZI 4 A. Type ΙΙ pneumocytes are present at 24 week gestation B. Cuboidal cells are capable of gas transfer in utero C. There is virtually no smooth muscle in the terminal & respiratory bronchioles at 6 month of age D. The large airways are formed at 16 week gestation E. Alveoli are completely formed at birth Ans:-AD 10) Congenital CMV infection:- A. Only 10% of affected pregnancies have resulting long term sequel at birth B. Diagnosis is by viral isolation from the urine C. Hearing loss can develop gradually over the first 5 years D. The affected newborn should be treated with ganciclovir E. Intracranial calcifications are seen in a periventricular distribution Ans:- ABCE Congenital CMV occurs in approximately 1% of all live births and only 10% of these infections result in clinical symptoms. Severe clinical disease is associated with primary maternal infection in pregnancy. Infection in early gestation carries a far greater risk of severe fetal disease. In CMV intra-cranial calcifications are in a periventricular distribution. Ganciclovir is only used if there is CNS involvement, chorioretinitis or pneumonitis. 11) The following conditions will present with cyanosis in the first week of life: A. Aortic stenosis B. Transposition of the great vessels C. Hypoplastic left heart syndrome D. Fallot's tetralogy E. Fallot's pentalogy Ans:- B Any cardiac lesion which allows a mixing of blood along with a right to left flow or any cardiac lesion wherein pulmonary perfusion is impaired results in cyanosis. Left heart problems or outflow tract obstructions present as cardiac failure. Fallot's pentalogy includes an ASD along with the tetrad of infundibular pulmonary stenosis, RVH, over-riding of the aorta and a VSD. Babies with tetralogy of Fallot usually have a patent ductus arteriosus at birth that provides additional pulmonary blood flow, so severe cyanosis is rare early after birth. As the ductus arteriosus closes, as it typically will in the first days of life, cyanosis can develop or become more severe. The degree of cyanosis is proportional to lung blood flow and thus depends upon the degree of narrowing of the outflow tract to the pulmonary arteries. 12) Pulmonary surfactant A. Is partly recycled by endocytosis into the synthesizing cell B. Is produced by type Ι alveolar cells C. Reduction in pulmonary flow can cause a decrease in surfactant production D. Synthesis is inhibited by thyroxine E. Synthesis is stimulated by glucocorticoids Ans:- ACE
  • 7. ABDULRAHMAN BASHIRCHILDREN HOSPITAL –BENGHAZI 5 Dipalmityl- phosphotidyl choline is the main component of surfactant and is produced by Type-ΙΙ alveolar cells (granular pneumocytes). Its half-life is 14 hours and its main function is to reduce the surface tension of the alveoli. 13) Lung surfactant A. Decreases the surface tension within an alveolus B. Causes an increase in chest wall compliance C. Is a glycoprotein D. Maintains the same surface tension for different sized alveoli E. Appears only after the 1st week of life Ans:- A Surfactant is a dipalmitoyl-phosphatidyl choline and is a phospholipid, which prevents alveolar collapse by reducing alveoli surface tension. It is produced by type-II pneumocytes and is seen at about 24 weeks gestation. It causes an increase in lung compliance only (not chest wall compliance). 14) The following organisms cause conjunctivitis:- A. Epstein Barr virus B. Chlamydia trachomatis C. Adenovirus D. Haemophilus influenzae E. Neisseria gonorrhoeae Ans:- BCDE Chlamydia trachomatis causes conjunctivitis in 30-50% of neonates born to mothers with cervicitis. It is a purulent conjunctivitis, which develops 5-14 days after birth and is indistinguishable from gonococcal infection. It is diagnosed on a swab scraped over the lower eyelid (to allow cells to be collected – don’t forget it is an intracellular organism) by direct fluorescent antibody, ELISA or PCR. Tetracycline ointment topically is combined with oral erythromycin – the oral antibiotic is to prevent relapse after ointment is discontinued and to prevent progression to pneumonia. Gonococcal conjunctivitis presents earlier than chlamydial disease (usually within 2 days), is diagnosed on gram stain and culture and should be treated with IV penicillin and chloramphenicol eye drops. Don’t forget sexual health screening for the mother and informing public health of ophthalmia neonatorum. Adenovirus causes conjunctivitis in summer outbreaks; enterovirus, coxsackie and herpes simplex are other viral causes. 15) Concerning blood flow in the fetus:- A. Blood flow from right to left through the foramen ovale B. Blood in the ascending aorta has higher oxygen content than in the descending aorta C. The ductus arteriosus is closed D. Pulmonary pressure equal systemic pressure E. Hemoglobin may be 20 gm/dl Ans:-ABE 16) -In a healthy baby the transition from fetal to neonatal circulation involves:- A. Functional closure of the foramen ovale in the first 24 hours
  • 8. ABDULRAHMAN BASHIRCHILDREN HOSPITAL –BENGHAZI 6 B. Blood flow in the ductus arteriosus continues from right to left until its closure C. Decrease in pulmonary artery resistance following closure of the ductus arteriosus D. The ductus arteriosus closes in response to decreased oxygen concentrations E. The umbilical artery is a branch of the common iliac artery Ans:- A Functional closure of the ductus arteriosus occurs soon after birth but anatomical closure can take upto one week. As pulmonary pressures fall after birth, blood flow in the ductus is reversed ie from left to right. The umbilical artery is a branch of the internal iliac artery. Factors influencing closure of the ductus include:- 1. Increased oxygen concentrations 2. Decreased prostaglandin levels 3. Drop in pulmonary artery pressures N.B. Prostaglandin E2 keeps the ductus open. 17) A 10-day old male presents with bilious emesis. What is the most likely diagnosis? A. Appendicitis B. Pyloric stenosis C. Malrotation with midgut volvulus D. Feeding intolerance Ans:- C 18) A term newborn is delivered by emergent cesarean section because of intrauterine growth restriction, oligohydramnios, and nonreassuring fetal heart rate monitoring in labor. Delivery room resuscitation includes endotracheal intubation and assisted ventilation with 100% oxygen, chest compressions, intravenous epinephrine, and volume expansion. Apgar scores are 1, 2, and 3 at 1, 5, and 10 minutes, respectively. An umbilical cord arterial blood gas measurement documents a pH of 6.9 and a base deficit of 20 mmol/L. At 12 hours of age, the infant demonstrates tonic-clonic convulsive activity of the arms and legs with a concomitant decrease in heart rate and bedside pulse oximetry saturation. Of the following, the MOST likely cause for this infant's seizure is:- A.Hypercalcemia. B.Hypercarbia. C.Hyperglycemia. D.Hypomagnesemia. E. Hypoxia. Preferred Response: E Seizures are the most frequent sign of central nervous system injury in the newborn. When seizures occur in a newborn who has depressed neuromotor tone, reflexes, and cardiopulmonary function at birth that requires assisted ventilation, perinatal asphyxia is likely. In this event, Apgar scores typically are depressed to less than 3 at 5 or more minutes after birth, and there is a severely acidotic umbilical cord arterial pH (<7.0), with evidence of metabolic acidemia. Poor tolerance of labor and asphyxia are more common in fetuses that have experienced intrauterine growth restriction. Because the infant in the vignette has the previously described features, hypoxic-ischemic encephalopathy (HIE) must be considered as a cause for the seizures. HIE is the most common cause of seizures occurring in the first 24 hours of postnatal life
  • 9. ABDULRAHMAN BASHIRCHILDREN HOSPITAL –BENGHAZI 7 and accounts for up to 67% of early neonatal seizures. Other causes of neonatal seizure include intracranial hemorrhage, cerebrovascular accidents (stroke), or hemorrhagic infarction (10% to 15%); intracranial malformation (<10%); transient hypoglycemia or hypocalcemia (<10%); drug withdrawal (<5%); and inborn errors of metabolism (<5%). When seizures occur beyond the first 24 hours after birth, especially in the absence of any history of fetal or neonatal asphyxia, the evaluation should focus on potential causes other than HIE. An additional cause for later seizures is infection (meningitis, encephalitis). Asphyxia may result in hypocalcemia and hypoglycemia; hyperglycemia and hypercalcemia are not associated with HIE and do not typically cause seizures. Hypomagnesemia may accompany hypocalcemia in the infant of a diabetic mother, but it is not common following asphyxia and is not associated with neonatal seizures. Hypercarbia may occur in the depressed newborn who has inadequate ventilation, but it is not associated with seizures unless there is corresponding hypoxia. 19) A 2-week-old-male presents with lethargy and vomiting. His electrolytes reveal sodium of 121 meq/L, potassium of 7.0-meq/l and blood glucose of 40 mg/dl. What is the most likely diagnosis? A. Dehydration B. Congenital adrenal hyperplasia C. Inborn error of metabolism D. Pyloric stenosis Ans:- B 20) The following cause the onset of persistent vomiting in 3 week old child:- A. Disaccharidase intolerance B. Duodenal atresia C. Pyloric stenosis D. Hiatus hernia E. Lactose intolerance Ans:-C 21) The drug of choice for treating a newborn with presumed ductal-dependant cyanotic congenital heart disease is: A. Morphine B. Dobutamine C. Prostaglandin E1 D. Indomethacin Ans:- C The drug of choice for ductal dependent cyanotic heart disease is prostaglandin E1- with a starting infusion of 0.05 - 1ug/kg/min. There is a risk of apnea associated with its use so be prepared to intubate, other complications include seizures and fever. Patients with congenital heart disease present with poor feeding, sweating with feeds, tachypnea, sudden onset of cyanosis or pallor that may worsen with crying, lethargy, or failure to thrive. Patients with cyanotic congenital heart disease are hypoxic but typically have a minimal response to oxygen therapy; whereas patients with a pulmonary process causing hypoxia will have an increase in
  • 10. ABDULRAHMAN BASHIRCHILDREN HOSPITAL –BENGHAZI 8 their oxygen saturation when oxygen is administered. Indomethacin is used to close a patent ductus. 22) How should a neonate with lethargy and a blood sugar of 20mg/dl be treated? A. Oral feeds with apple juice B. 25% dextrose solution C. 10% dextrose solution D. 50% dextrose solution Ans:- C Newborns with hypoglycemia should be treated with D10W solution with a range of 2- 10cc/kg. Higher concentrations should not be used as they can cause vein sclerosis and intracranial hemorrhage. Infants and young children should be treated with D25 2-4cc/kg. 23) Neonatal convulsion can be caused by:- A. Maternal hyperparathyroidism B. Subdural hematoma C. Birth asphyxia D. Hyponatremia E. Wilson's disease Ans:-ABCD 24) A 2-day-old female presents with abdominal distension and vomiting. She has not yet passed a meconium stool. What is the most likely diagnosis? A. Hirschsprung Disease B. Malrotation with midgut volvulus C. Necrotizing enterocolitis D. Constipation Ans:-A Suspect Hirschsprung Disease in a newborn who has not yet passed a meconium stool. Other possibilities include an imperforate anus or meconium plugging. Older children present with a history of chronic constipation. Hirschsprung disease is the absence of intramural ganglion cells in the rectum which extends to the sigmoid colon in 77% of patients and involves the entire colon in 15% of patients. The incidence is 1/5,000 live-births, with a male to female ratio of 4:1. The diagnosis should be suspected if the patient presents with lack of meconium stool within the first 24 to 48 hours of life. Vomiting and abdominal distension may also be present. 25) A 3-week-old female presents with persistent seizures despite aggressive management with benzodiazepines and phenobarbital. The mother reports giving her daughter some water to "stop her from getting dehydrated." What is the most likely cause of her status epilepticus? A. Hypoglycemia B. Diabetes insipidus C. Hyponatremia D. Hypokalemi Ans:-C
  • 11. ABDULRAHMAN BASHIRCHILDREN HOSPITAL –BENGHAZI 9 Excessive free water intake can result in hyponatremic seizures. Infants less than 6 months of age are particularly susceptible to these types of seizures and commonly have intractable seizures requiring intubation and hypothermia. Immediate treatment includes the administration of 3% saline 4cc/kg. 26) A 1-week-old male presents with some mild erythema around his umbilicus extending onto the abdominal wall. Which of the following is the correct management for this patient? A. Reassurance and continue with alcohol wipes of umbilicus B. Topical antibiotic ointment and recheck the patient the next day C. Discharge on cephalexin and recheck the next day D. Perform a full septic workup and admit the patient Ans:- D This patient has Omphalitis and should undergo a full septic evaluation, administration of antibiotics and hospital admission. Surgical debridement may be required for severe cases. Omphalitis is inflammation and infection surrounding the umbilicus that can spread to the liver or peritoneum. Patients can present with symptoms ranging from mild erythema to necrosing lesions surrounding the umbilicus on the abdominal wall. Fever may be present 27) A 5-day old, well-appearing male is brought to the ED by his mother today because she noted that he has a cluster of vesicles on his scalp. Which of the following should be the management approach? A. Skin biopsy B. IV acyclovir and a full septic workup C. Oral acyclovir D. Discharge, with next day follow up Ans:-B This patient is at risk for herpes encephalitis and should undergo a complete septic workup and IV acyclovir should be initiated in the ED. Begin acyclovir (20mg/kg every 8 hours IV) if there is a positive maternal history of herpes, a vesicular rash, focal neurologic findings, CSF pleocytosis or elevated CSF protein without organisms on gram stain. 28) The following maternal condition can cause disease in the fetus/newborn:- A. Hyperparathyroidism B. ITP C. Myasthenia gravis D. Diabetus mellitus E. Thyrotoxicosis Ans:-ABCDE 29) Which of the following heart rates is most suggestive of supraventricular tachycardia in a newborn? A. 180 BPM B. 230 BPM C. 150 BPM D. 210 BPM
  • 12. ABDULRAHMAN BASHIRCHILDREN HOSPITAL –BENGHAZI 10 Ans:-B PALS defines SVT in infants as a heart rate of greater than 220 BPM. In older children the heart rate for SVT is greater than 180 BPM. The ECG demonstrates a narrow complex tachycardia without discernible p waves or beat-to-beat variability. In the stable patient, vagal maneuvers are the first treatment of choice (ice to the face, or blowing through an occluded straw in older children). Adenosine given as centrally as possible is the first drug of choice ( 0.1mg/kg up to 6mg for the first dose and then 0.2mg/kg for the second dose up to 12 mg) If this is not successful, then amiodarone 5mg/kg given over 20-60 minutes or procainamide 15mg/kg given over 30-60 minutes are the next drugs of choice. Unstable patients should undergo cardioversion with 0.5-1J/kg followed by 2J/kg. If an IV is accessible, a dose of adenosine can be given while setting up for the cardioversion. 30) Vaginal bleeding in a 3-day-old female is: A. Is always indicative of child abuse B. May be due to withdrawal of maternal hormones C. Is suspicious for gonorrhea D. Is most commonly due to a vaginal foreign body-such as baby wipes Ans:- B 31) Cyanosis in the first week of life can be caused by:- A. Tetralogy of Fallot B. Pulmonary stenosis C. Eisenmenger syndrome D. TAPVD E. Ebistein's anomaly Ans:-ABDE 32) A 3-week old male presenting to the emergency department with vomiting and altered mental status and acidosis. What additional laboratory test should be included in your evaluation? A. Ammonia level B. Cortisol level C. Serum acetone D. Thyroid function test Ans:- A Suspect an inborn error of metabolism in patients who have an altered level of consciousness. These patients may or may not be acidotic depending on the type of inborn error that is present. Patients with a urea cycle defect typically have a normal blood gas but an elevated ammonia level. Patients with organic acidemias will be acidotic but may or may not have an elevated ammonia level. Patients with galactosemia will have a normal blood gas and ammonia level but will have reducing substances in the urine. 33) Which of the following are causes of shock in the newborn? A. Infection B. Inborn errors of metabolism C. Child abuse D. Thyrotoxicosis
  • 13. ABDULRAHMAN BASHIRCHILDREN HOSPITAL –BENGHAZI 11 E. All of the above Ans:- E 34) In neonatal RDS (respiratory distress syndrome): A. Surfactant is useful in the treatment. B. It is rare in infants below 28 weeks gestation. C. Antenatal steroids are beneficial. D. Maternal opiate abuse increases the risk. E. Maternal diabetes increases the risk. Ans:- ACE 35) The risk of neonatal jaundice is increase by: A. Prematurity. B. Trisomy 21. C. Elective caesarean section. D. Congenital hypothyroidism. E. Cephalahaematoma. Ans:-ABDE 36) At birth the blood volume is approximately:- A. 65 ml/kg body weight B. 85 ml/kg body weight C. 110 ml/kg body weight D. 125 ml/kg body weight E. 150 ml/kg body weight Ans:- B 37) Newborn infants commonly have:- A. Papulovesicles over the trunk. B. Posterior fusion of the labia minora. C. An adherent foreskin D. Breast enlargement E. Shallow sacral dimple Ans:-ACDE 38) The following conditions signify disease in the newborn;- A. Peeling of the skin of the hands and feet B. Blanched on one side of the body and pink on the opposite side C. Pinhead lesion on the nose ( milia) D. Peripheral cyanosis E. Oedema of one arm Ans:- ALL FALS 39) Apreviously healthy full term infant have several episode of duskiness and feeding difficulties during the second day of life . She is noted to have increasing jaundice, which of the following tests will be the least helpful in making diagnosis:- A. CSF B. Urine C/ S C. Total bilirubin D. Endotracheal aspirate C/S E. Venous blood
  • 14. ABDULRAHMAN BASHIRCHILDREN HOSPITAL –BENGHAZI 12 Ans:- D 40) Difference between infant born to heroin-abusing mothers and infant born to phenobarbitone-abusing mothers is that infant in the later group:- A. Don't have withdrawal symptoms B. Have withdrawal symptoms appearing earlier than heroin withdrawal C. Don't develop tremor D. Have high incidence of jaundice E. Are usually term and full size Ans:- E 41) Which of the following organisms is the most frequent causes of neonatal meningitis:- A. GBS B. E . coli C. L . monocytogenes D. H .influenza E. S. pneumoniae Ans:- A 42) One should be concerned about term infant who has not passed meconium stool:- A. During the process of birth B. Within few min of birth C. By 1-2 hour of life D. By 6-12 hour of life E. By 24 hour of life Ans:- E 43) The initial lesion of incontinentia pigmenti are:- A. Deeply pigmented B. Scaly C. Waxy papules D. Inflammatory bullae E. Small vesicles Ans:- D 44) Factor that appear to lower threshold for neurologic damage and kernicterus from unconjugated hyperbilirubinemia include all of the following except:- A. Acidosis B. Asphyxia C. Sepsis D. Postmaturity E. Hypothermia Ans:- D 45) Infant born to diabetic mothers are at risk of all of the following except:- A. Polycythemia B. Hyperglycemic dehydration C. Hypocalcemia D. Congenital malformation E. Cardiomyopathy Ans:- B
  • 15. ABDULRAHMAN BASHIRCHILDREN HOSPITAL –BENGHAZI 13 46) The Apt test is used for what purpose:- A. Crude test for carbon monoxide poisoning B. Semiquantitative test for lead poisoning C. Qualitative test for fetal hemoglobin D. Screening test for S hemoglobin E. Test for blood viscosity Ans:- C 47) Birth injury account for the majority of the following conditions detected in early infancy:- A. Intraventricular hemorrhage B. Cephalohematoma C. Hydrocephalus D. Facial nerve palsy E. Pneumothorax Ans:-BD 48) In birth trauma:- A. Erb's palsy involve C6,C7 & C8 B. Klumpke's palsy involve C7,C8 &T1 C. Facial nerve palsy lead to persistently closed eye D. Sciatic nerve involvement is common E. Cephalohematoma is characteristically present at birth Ans:- B In Erb-Duchenne paralysis, the injury is limited to the 5th and 6th cervical nerves. The infant loses the power to abduct the arm from the shoulder, rotate the arm externally, and supinate the forearm. The characteristic position consists of adduction and internal rotation of the arm with pronation of the forearm. Klumpke paralysis is a rare form of brachial palsy; injury to the 7th and 8th cervical nerves and the 1st thoracic nerve produces a paralyzed hand and ipsilateral ptosis, anhidrosis and miosis (Horner syndrome) if the sympathetic fibers of the 1st thoracic root are also injured (which reflects damage to the stellate ganglion adjacent to T1.) 49) Birth injury:- A. Paralysis of the upper arm has better prognosis than paralysis of the lower B. In nerve injury, neuroplasty is advised at the end of first year of life C. In phrenic nerve paralysis, spontaneous recovery is expected D. Facial nerve paralysis will result from the nuclear agenesis of the facial nerve Ans:-AC If the paralysis persists without improvement for 3–6 mo, neuroplasty, neurolysis, end-to-end anastomosis, and nerve grafting offer hope for partial recovery. function usually returns in a few months. Total disruption of nerves (neurotmesis) or root avulsion is the most severe, especially if it involves C5–T1; microsurgical repair may be indicated. Fortunately, most (75%) injuries are at the root level C5–C6, involve neurapraxia and axonotmesis, and should heal spontaneously. Botulism toxin may be used to treat biceps-triceps co-contractions. 50) The following are useful in the assessment of gestational age in preterm:- A. Presence of palmar creases
  • 16. ABDULRAHMAN BASHIRCHILDREN HOSPITAL –BENGHAZI 14 B. Breast size C. Sacral edema D. The scarf sign E. Muscle ton Ans:-BDE 51) Cephalohematoma:- A. Must be differentiated from subperiosteal hemorrhage B. Is usually visible at birth C. May calcify D. May be associated with underling fracture of the skull E. Should be managed surgically Ans:-CD 52) Caput succedaneum is characterized by all of the following except:- A. Diffuse edematous swelling of the soft tissues of the scalp involving the portion presenting during vertex delivery B. It may extend across the midline C. It may extend across the suture lines D. Edema usually disappear within 2-3 months E. The scalp overlying the area may show mild bruising Ans:- D 53) In newborn with oral moniliasis the most common primary source of infection A. Maternal source ( vaginal ) B. Contaminated fomites C. Following use of AgNO2 D. Contact by hospital carriers E. Systemic antibiotic therapy Ans:- A 54) Meconium impaction is associated with:- A. Cretinism B. Cystic fibrosis C. Thrush D. HMD E. Trisomy 21 syndromes Ans;- B 55) Premature infant is delivered precipitiously and appear asphyxiated . The infant is cyanotic, there are no respiratory efforts,and the heart rate is 80 / min .The infant is meconium stained and thick particulate meconium is noted in the amniotic fluid and in infant mouth. At this point you should:- A. Pass an umbilical artery catheter to measure the PH & PO2 B. Start bag-mask ventilation with 100% oxygen C. Suction the oropharynx & trachea with ETT to remove the meconium D. Intubate the trachea & begin the ventilation with 100 % oxygen E. Establish monitoring with ECG & pulse oximeter Ans:- C 56) Hypoglycemia has been observed in all of the following except;-
  • 17. ABDULRAHMAN BASHIRCHILDREN HOSPITAL –BENGHAZI 15 A. With LBW and RDS B. With anoxic injury C. With hypothermia D. Who are SGA E. With high PO2 Ans:- E 57) All of the following are usually associated with cretinism except:- A. Constipation B. Prolonged jaundice C. Lethargy D. Tetany E. Hypotonia Ans:- D 58) All of the following are characteristic of single umbilical artery except:- A. Presence in about 5 of 1000 birth B. About 1/3 of such infants have congenital abnormalities C. 21 trisomy is frequently found D. Among twin ,the rate of occurrence is 35 / 1000 E. The associated congenital abnormalities may involve the genitourinary tract Ans:- C 59) Two minutes after normal term delivary:- A. The ductus venosus will be closed B. The pulmonary arterial pressure will have fallen C. The pressure in the left atrium will have fallen D. The arterial oxygen tension will have risen E. Regular breathing will have begun Ans :-ABDE 60) Established neonatal resuscitation procedures include:- A. Directing cold stream of oxygen at the nose B. Administration of drugs with respiratory stimulant properties C. Oropharyngeal suction D. Bag and face mask ventilation E. Prompt cooling Ans:-ACD 61) The Apgar score :- A. At 1 min is reliable measure of asphyxia B. At 1 min is reliable measure of respiratory failure C. At 10 min is strongly correlated with later neurological deficit D. Includes the infant response to pharyngeal suction catheter E. Isn't application after 10 min of age Ans:- BCD 62) Criteria used in the Apgar score include :- A. Core temperature B. Heart rate C. Respiratory rate
  • 18. ABDULRAHMAN BASHIRCHILDREN HOSPITAL –BENGHAZI 16 D. Skin thickness E. Muscle tone Ans :-BE 63) Pink newborn with HR of 88/min is actively gasping , he has good muscle tone and respond to nasal catheter stimulation with facial grimace, the apgar score is:- A. 5 B. 6 C. 7 D. 8 E. 9 Ans:- D 64) Mongolian spots are characterized by all of the following except:- A. They are permanent B. They are usually of a slate blue pigmentation C. They are generally observed over the buttocks D. The area of pigmentation is well demarcated E. They aren't associated with trisomy syndromes Ans:-A 65) All of the following physical signs may be useful in estimating the gestational age at birth except:- A. There are only one or two transverse skin creases on the sole of the foot until 36 week of gestation B. The breast nodule is usually not palpable at 33 or 34 weeks C. The breast nodule is usually 4-10 mm in term infant D. The testes are descending and rugae cover the entire scrotal surface by 34 weeks E. The texture of scalp hair Ans:- D The testes are usually not completely descended until after 36 weeks & scrotal rugae are few and limited the anterior and inferior aspect of relatively small scrotum . By 34 weeks the areola become raised and between 36-37 weeks the breast bud is 1 – 2 mm reaching size of 4-10 mm at term. The transversr foot creases develop at 31 – 32 weeks .By 36 weeks creases cover the anterior two-third of the sole . 66) Newborn infant who remain centrally cyanosed after intubation and IPPV may have:- A. Diaphragmatic hernia B. Choanal atresia C. Tension pneumothorax D. Drug induced respiratory center impairment E. Profound anaemia Ans:-AC 67) The following statement about pulmonary hypertension are true :- A. It recognized complication of group B streptococcal sepsis B. Hyperventilation is an effective treatment C. Tolazoline is potent pulmonary vasoconstriction
  • 19. ABDULRAHMAN BASHIRCHILDREN HOSPITAL –BENGHAZI 17 D. Radial arterial PaO2 is lower than umbilical artery PaO2 E. Birth asphyxia is a risk factor Ans:-ABE 68) Concerning air leak syndromes in the newborn:- A. Underwater seal drain is only required if the pneumothorax is under tension B. In term baby with small pneumothorax giving oxygen at high concentration can worsen it C. Increasing the I; E ratio in ventilated baby decrease the risk of pneumothorax D. Pneumomediastenum is usually fatal E. They can be asymptomatic Ans;- E 69) Recognized problem of infants born at term SGA include:- A. Hypothermia B. Sepsis C. Polycythemia D. Hypoglycemia E. Retinopathy of prematurity Ans:-ABCD 70) Complication of steroid therapy in the newborn include:- A. Leucopenia B. Hypoglycemia C. Cataract D. Sepsis E. Gastric perforation Ans:-CDE 71) Concerning NEC :- A. Exchange transfusion is predisposing factor B. Clostridium welchii is implicated in the pathogenesis C. It is most common in infants born less than 1500 gm D. Oral antibiotics are useful E. Complications include short bowel syndrome Ans:-ACE 72) The following congenital condition require immediate ( within first week) treatment after birth:- A. TEF B. Cleft lip C. Spina bifida D. Exomphalos E. Hydrcephalus Ans:-ACD 73) Preterm infant at increased risk from :- A. Conjugated hyperbilirubinemia B. Meconium aspiration C. Periventricular leucomalacia
  • 20. ABDULRAHMAN BASHIRCHILDREN HOSPITAL –BENGHAZI 18 D. Necrotizing enterocloitis E. Child abuse Ans:- CDE 74) Peri- or intraventricular cerebral hemorrhage A. Occur in less than 10% of VLBW infant B. Arise most commonly in the first 72 hour after delivary C. Is direct result of impaired vitamin K supply D. Is the single most common cause of congenital cerebral palsy E. May result in rapidly evolving hydrocephalus Ans:-BE 75) Feature typical of physiological jaundice include:- A. Recognizable jaundice in the first 48 hours B. Peak plasma bilirubin at 4-5 days C. Persistent beyond first week D. Irritability E. Pale stool Ans:- B 76) Jaundice on day 1 is often caused by ;- A. Metabolic disorder B. TORCH infection C. Gastrointestinal obstruction D. Hemolysis E. Physiological factor Ans:-BD 77) Persistant, conjugated hyperbilirubinaemia may be caused by: A. Alpha-1-antitrypsin deficiency. B. Hypothyroidism. C. Haemolytic disease. D. Cytomegalovirus infection. E. Cystic fibrosis. Ans:-ABDE Explanation:- all cases need further investigation. Causes of conjugated hyperbilirubinaemia in a neonte are: 1)-Intrahepatic cholestasis -Infections. -Congenital infections (STORCH-syphlis, toxoplasma, rubella, cytomegalovirus, hepatitis, herpes virus infection) -Acquired infections- septicaemia, UTI. -Metabolic disorders –cystic fibrosis, alpha-1- antitrypsin deficiency, galactosaemia, fructosaemia, lysosomal storage disorders, peroxisomal disorders. -Endocrine disorders – hypothyroidism, hypopituitarism, hypoadrenalism. -Anatomical disorders – intrahepatic:bile duct hypoplasia. -Miscellaneous – idiopathic neonatal hepatitis, chromosomal abnormalities, trisomy 21, 18 and 13. 2)-Extrahepatic cholestasis:- biliary atresia, choledochal cyst, spontaneous bile duct
  • 21. ABDULRAHMAN BASHIRCHILDREN HOSPITAL –BENGHAZI 19 perforation, inspissated bile syndrome. Unconjugated neonatal jaundice is normally physiological and resolves spontaneously. 78) All of the following are characteristic of jaundice associated with breast feeding except :- A. Significant elevation of unconjugated bilirubin B. Rapid fall in serum bilirubin after discontinuation of nursing C. Nursing can be resumed after several days without return of hyperbilirbinemia D. Significant elevation of conjugated bilirubin E. Kernicterus has never been reported to occur as result of breast milk jaundice alone Ans :- D 79) Persistent jaundice during the first month of life may associated with all of the following except:- A. Cytomegalic inclusion disease B. Congenital atresia of the bile duct C. Galactosemia D. Rh incompatibility E. Penicillin treatment Ans:- E 80) A 26-week gestation preterm infant is now 6 weeks old and weighs 1,250 g. He is receiving full-volume enteral nutrition. The only significant finding on physical examination is pallor. He has anemia (hematocrit of 28% [0.28]; reticulocyte count of 8% [0.08]) and receives iron supplementation. He is receiving a formula that is high in polyunsaturated fatty acids.Of the following, the MOST correct statement about his need for vitamin E is that it A. does not need to be supplemented in infancy B. has no effect on anemia C. needs to be supplemented now D. will be needed when the infant is 3 months old E. will prevent anemia Preferred Response: C Historically, inadequate vitamin E, a high level of polyunsaturated fatty acids (PUFAs) in infant formula, and exposure to the oxidizing effects of iron supplementation contributed to a hemolytic anemia seen in preterm infants. In the United States and many developed countries, infant formulas now provide an adequate vitamin E:PUFA ratio to eliminate this risk. However, preterm infants continue to have low vitamin E levels due to limited stores, especially when the birth is extremely premature, and limited enteral feedings early in their postnatal neonatal intensive care unit course. The neonate who has anemia and is receiving iron supplementation, such as the infant described in the vignette, requires the antioxidant effect of vitamin E to reduce red blood cell hemolysis. A total daily requirement of 10 to 25 IU of vitamin E meets the infant’s needs, only 50% of which is provided by dietary formula. Of note, human milk also is an incomplete source of vitamin E. 81) Amniocentesis is useful in establishing the prenatal diagnosis of:- A. Down syndrome B. Meningomyelocele
  • 22. ABDULRAHMAN BASHIRCHILDREN HOSPITAL –BENGHAZI 20 C. Erythroblastosis fetalis D. Achondroplasia Ans:-ABC 82) USS can be used during pregnancy to:- A. Determine crown rump length B. Determine fetal sex C. Determine biparietal diameter D. Accurately determine fetal weight Ans ;-AC 83) Gestation that produce multiple births:- A. Are classified as high risk B. Are always delivered by cesarean section C. Can produce infant with discordance in body size at birth D. Aren't associated with the premature onset of labour Ans:-AC 84) For newborn infant, the least important factor to consider in assessing the risk of kernicterus is:- A. Breast feeding B. Acidosis C. Sepsis D. Albumin level E. Moxalactam, cephalosporin type antibiotic Ans:- A 85) The following maternal conditions are known to cause adverse effects on the neonate: A. Diabetes insipidus. B. Toxoplasmosis. C. Chronic myeloid leukaemia. D. SLE. E. Hyperthyroidism. Ans:-BCDE Explanation:- Maternal SLE is a risk factor for neonatal lupus syndrome. The neonate presents with clinical features of SLE due to transplacental passage of maternal Ab. The skin is frequently involved with malar rashes and there can be haematological and cardiac abnormalities. The most frequent heart abnormality is congenital heart block – 90% of mothers whose infants have congenital heart block are anti-Ro(SSA)Ab positive. Toxoplasmosis leads to congenital infection by transplacental transmission. The rate of transmission is +/- 60% of third trimester infections and 20%-30% during the first two trimesters. The ewborn presents with the classic triad of hydrocephalus, chorioretinitis, and cerebral calcification. Maternal chronic myeloid leukaemia can have adverse effect on pregnancy – fetal/neonatal mortality is 16-38%. Pathology can be secondary to placental leukaemic infiltrates, anaemia and infectious complications. Splenomegaly can restrict intrauterine growth and lead to premature delivery.
  • 23. ABDULRAHMAN BASHIRCHILDREN HOSPITAL –BENGHAZI 21 If hyperthyroidism is due to Grave’s disease or Hashimoto thyroiditis, the neonate may present with thyrotoxicosis due to the transplacental passage of TSI. These symptoms are frequentely short-lived i.e as long as the circulating antibodies persist in the baby’s circulation. 86) Each of the following statement about GBS infection in the newborn is true except:- A. Incidence is correlates inversely with the presence of maternal antibodies B. The exotoxin has powerful pulmonary vasoconstrictive effects C. The disease has two distinct pattern; early & late onset D. Both the incidence & severity of the disease can be lessened by pretreatment of the mother who is colonized E. The risk of invasive disease isn't related to the amount of inoculum received by the infant Ans:- E 87) Newborn infant may present with bile stained vomiting and abdominal distension as the result of A. Oesophageal atresia B. Duodenal atresia C. Birth asphyxia D. Electrolyte disturbance E. Cystic fibrosis Ans:-BCDE 88) The following feature are consistent with newborn infant having oesophageal atresia and tracheo-oesophageal fistula:- A. Maternal polyhydramniose B. Passage of wide bore orogastric catheter into the stomach C. Plain x-ray evidence of air in the stomach and small bowel D. Plain x-ray evidence of hemivertebra E. Excessive mucus in the nostrils or mouth. Ans:-ACDE 89) Vitamin K A. Is an essential cofactor for the synthesis of coagulation factor II , VII, IC, C B. Is readily transported across the placenta C. Is present in the breast milk at higher concentration than in cow milk. D. Given in single oral dose after delivary effectively prevent s hemorrhagic disease E. Related hemorrhage in the newborn is commoner when mother have taken anticonvulsant during pregnancy Ans:-AE 90) The following are recognizable causes of neonatal convulsion:- A. Birth asphyxia B. Hypoglycemia C. Hypothermia D. Opiate withdrawal E. Hypernatremia Ans:-ABDE 91) With regard to Apgar score:-
  • 24. ABDULRAHMAN BASHIRCHILDREN HOSPITAL –BENGHAZI 22 A. 2 points given to pulse of 88/min B. 1 point is given for irregular gasps C. An initial satisfactory score gurarantees an eventual perinatal period D. Score of 2 at 10 min carries worse prognosis than score of 2 at 5 min E. The lowest possible score is 1 Ans:-BD 92) Intrauterine posture is commonly responsible for:- A. Congenital dislocation of the hip B. Plagiocephaly C. Sternomastoid shortening D. Syndactyly E. Mandibular asymmetry Ans:-ABCE 93) Established neonatal screening tests include:- A. Umbilical cord blood analysis to detect phenylketonuria B. Umbilical cord blood analysis to detect galactose C. Umbilical cord blood analysis to detect sickle cell disease D. Capillary blood analysis at 6-8 days to detect elevated TSH E. Capillary blood analysis at 6-8 days to detect elevated immunoreactive trypsin Ans:-CDE 94) Maternal condition that may have effect in the neonatal period include:- A. ITP B. Multiple sclerosis C. DM D. Varicella zoster E. Bornholm disease Ans:-ACDE 95) Neonatal polycythemia :- A. Occur in small for date infant as response to placental insufficiency B. Has an increased incidence if maternal diabetes is poorly controlled C. Carry an increased risk of cerebral venous sinus thrombosis D. Is recognizable feature of congenital hypothyroidism E. May occur as result of feto-maternal transfusion Ans:-ABCD 96) Polycythemia on the newborn is characterized by all of the following except:- A. Increased the incidence in IDM B. Placental transfusion decreased by late clamping of the umbilical cord C. Clinical presentation that includes jitteriness & poor feeding D. Complication including heart failure & NEC E. Elevated viscosity of whole blood Ans;- B 97) Causes of neonatal polycythemia include:- A. Congenital rubella infection
  • 25. ABDULRAHMAN BASHIRCHILDREN HOSPITAL –BENGHAZI 23 B. Pre-eclampsia C. Maternal diabtus mellitus D. Delayed clamping of umbilical cord E. Congenital adrenal hyperplasia Ans:- BCDE 98) Meconium aspiration pneumonia:- A. Occur with equal frequency in term and preterm infant B. In infant required ventilation for this condition, combination of high PEEP and rapid rate is advisable C. Has high risk of developing even if liquor is only thinly stained D. High dose steroid are the mainstay of treatment E. Antibiotic treatment is an important part of treatment Ans :- all False 99) The following statement are true regarding hemolytic disease of newborn ( HDN ):_ A. Hemolytic disease should be suspected if jaundice is noted in the first 24 hour of life B. HDN may occur if mother is group A +ve and the baby is group O+ve C. If due to Rh –incompatibility the severity of the hemolysis typically increase with each affected pregnancy D. HDN due to ABO incompatibility can be detected at 36 week gestation by amniocentesis E. As long as the level of unconjugated bilirubin never rises above20 mg/dl there is no dangerous of kernicterus Ans:-AC 100) Full term infant has tachypnea with grunting , chest X ray show well expanded lung with streaky shadows radiating from the bilateral hilar region , the most likely diagnosis is :- A. TTN B. Congenital pneumonia C. BPD D. Meconium aspiration E. Aspiration pneumonia Ans:- A 101) Condition associated with polyhydramniose include:- A. Oesophageal atresia B. Down's syndrome C. Renal agenesis D. Cord around the neck E. CNS malformation Ans:- ABE 102) The following are causes of polyhydramnios: A. Maternal diabetes mellitus. B. Potter syndrome. C. Anencephaly. D. Oesophageal atresia.
  • 26. ABDULRAHMAN BASHIRCHILDREN HOSPITAL –BENGHAZI 24 E. Polycystic kidneys. Ans:-ACD 103) The following are causes of polyhydramniose:- A. Pottrer syndrome B. TEF and oesophageal atresia C. Rh – incompatibility D. Anencephaly E. Maternal diabetus Ans:-BDE 104) For necrotizing enterocloitis:- A. Term baby are particularly at risk B. Perinatal asphyxia is risk factor C. Expressed breast milk from milk bank confer no protection D. Failure of temperature control is late sign E. High platelet count is frequently seen Ans:- B 105) You diagnose necrotizing enterocolitis in a preterm neonate who has abdominal distention and blood in the stool. You decide that this infant should be placed on a 14-day regimen of parenteral nutrition.Of the following, the micronutrient for which weekly monitoring is MOST recommended during this infant’s parenteral nutrition therapy is A. copper B. iron C. phosphorus D. selenium E. zinc Preferred Response: C Parenteral nutrition (PN), the intravenous administration of carbohydrates, lipids, amino acids, and micronutrients, is an important component in the management of a variety of chronic disorders, including surgical conditions (eg, omphalocele, gastroschisis, diaphragmatic hernia, short bowel syndrome), inflammatory conditions (eg, Crohn disease, ulcerative colitis, pseudomembranous colitis, pancreatitis, graft versus host disease), hypermetabolic states (eg, burns, trauma), and intestinal motility disorders (eg, pseudo-obstruction). PN is especially important in the support of very low-birthweight infants, who frequently have increased caloric requirements, decreased oral intake, and immature intestinal motility. In addition to glucose, amino acids, sodium, potassium, and chloride, PN provides additional mineral supplements, including calcium, phosphorus, magnesium, zinc, copper, selenium, chromium, manganese, molybdenum, and iodide. In preterm infants, the administration of both calcium and phosphorus is important to prevent metabolic bone disease. However, the amount of calcium and phosphorus that can be administered in PN must be limited because of the risk of precipitation in the PN and formation of calcium-phosphorus complexes that could embolize. It generally is thought that the chance of precipitation is high if the product of the concentrations (in mmol/L) of the calcium and phosphorus in PN is greater than 40. Because hypophosphatemia can have significant consequences (including impaired cardiac function, muscle weakness, and hemolysis), it is recommended that serum phosphorus concentrations be assessed at least once weekly for patients receiving PN. The serum
  • 27. ABDULRAHMAN BASHIRCHILDREN HOSPITAL –BENGHAZI 25 concentrations of copper, iron, selenium, and zinc, which are trace elements, require only periodic assessment (eg, once a month or less). 106) An 1,800-g preterm infant is recovering from surgery for gastroschisis. Of the following, the MOST correct statement regarding this infant’s immediate daily nutritional requirements is that he A. can meet energy needs from fat through enteral feeding B. can meet protein needs for growth through enteral feeding C. requires lysine supplementation to promote protein synthesis D. requires more protein per kilogram than a term infant E. requires 2.5 g/kg per day of protein Preferred Response: D Milk remains the principal source of nutrition for infants, who consume 120 to 150 mL/kg per day in the newborn period. Human milk has 67 kcal/100 mL, and most term infant formulas have a similar composition. The energy derived from ingesting milk comes primarily from fat calories (3.8 g/100 mL = 34kcal/100 mL), followed by carbohydrates (lactose, 7 g/100 mL = 28 kcal/100 mL) and minimally from protein (1.3 g/100 mL = 5 kcal/100 mL). Conditions such as prematurity, lung disease, or surgery may increase both caloric and specific nutrient requirements for newborns. The newborn described in the vignette can only take limited enteral nutrition and has an increased need for both protein and calories to facilitate healing and growth. The normal 2 to 3 g/kg per day of protein ingested by the term infant who is either formula- or breastfed is inadequate for this low-birthweight, preterm newborn. He will grow and heal best receiving 3.5 to 4.0 g/kg per day of protein. The potential benefits of single amino-acid supplementation, such as lysine, remain unknown. 107) Hypoxic-ischemic encephalopathy all are correct except:- A. Result from excessive use of oxytocin B. IUGR is be first indication of fetal hyoxia C. Associated with increased beat to beat variability D. Prognosis depend on gestational age E. Persistent of abnormal neurological sign at two week indicate poor prognosis Ans:- C 108) polyhydramniose:- A. Is defined as an amniotic fluid volume of more than 500 ml B. Occur with increased frequency in diabetic pregnancy C. Is associated with renal agenesis D. Is associated with tracheo-oesophageal fistula E. Is associated with increased risk of premature labor Ans :-BDE 109) Polyhydramnios is associated with all of the following condition except:- A. Amniotic fluid volume between 500-2000 ml B. Maternal diabetes
  • 28. ABDULRAHMAN BASHIRCHILDREN HOSPITAL –BENGHAZI 26 C. Twins D. Erythroblastosis fetalis E. Down syndrome Ans:- A 110) Condition associated with oligohydramnios include:- A. Infantile polycystic kidney B. Congenital heart disease C. IUGR D. High intestinal obstruction E. Posterior urethral valve Ans:- ACE 111) Regarding hyaline membrane disease ( HMD) A. Can occur in infants of diabetic mother of 37-40 week gestation B. Infant born to mothers who are heroin addicts are at increased risk of HMD C. Light-for –date infant of 33 weeks gestation has greater risk of developing HMD than 33 week infant of appropriate weight D. Administration of artificial surfactant is curative E. Chest x-ray finding are markedly different between cases of HMD and Group B streptococcal pneumonia Ans:- A 112) Hyaline membrane disease:- A. Is more common in babies of diabetic mother B. Is due to surfactant deficiency C. Is not seen in term babies D. Occur most commonly at 12 hour post-delivary E. Always require ventilation Ans:- AB 113) The following symptoms in an infants in the first month of life should alert one to possibility of hypothyroidism: A. Prolonged jaundice B. Vomiting C. Diarrhea D. Hoarse cry E. Voracious appetite Ans:- AD 114) The following malformation can occur in children born to mother who have IDDM: A. Cleft lip/ palate B. Caudal regression syndrome C. Femoral hypoplasia D. Holopresencaphaly E. Polydactyly Ans:- BCDE 115) With regard to RDS of the newborn
  • 29. ABDULRAHMAN BASHIRCHILDREN HOSPITAL –BENGHAZI 27 A. Meconium aspiration pneumonia is especially likely to be the cause if the infant is preterm B. HMD is unlikely to be the cause if the prepartum lecithin sphingomylin (L/S) ratio in amniotic fluid is > 2;1 C. Patchy opacities on chest X-ray are evidence in favour of diagnosis HMD D. In the artificial ventilation of infant with HMD , inflation pressure must never exceed 30 cm of water E. CPAP is an effective in treatment for Pneumothorax Ans : B 116) In RDS due to meconium aspiration A. Antibiotic therapy is of crucial importance B. Steroid therapy improve the prognosis C. There is high risk of Pneumothorax D. Chest X-ray typically shows ground glass opacity E. The infant may also suffer from cerebral oedema Ans:-C 117) Recognized association of small-for-date babies include A. Maternal smoking in pregnancy B. Hemolytic disease due to ABO incompatibility C. Pre-eclamptic toxaemia D. Congenital rubella infection E. Fetal alcohol syndrome Ans:- ACDE 118) Prolonged neonatal jaundice is recognized feature in infants with A. CMV infection B. Congenital hypothyroidism C. Untreated urinary tract infection D. Tracheo-oesophageal fistula E. Galactosemia Ans:- ABCE 119) In baby of 32 week gestation who has tachypnea and sternal recession at 4 hour of age: A. The ductus arteriosus is likely to be patent B. Meconium aspiration pneumonia is likely to be diagnosis C. The L/S ratio is likely to be low D. Fluid level on chest X-ray suggest pneumonia due to Group B streptococcus E. The presence of bowel shadows on the left side of the chest on X-ray is diagnostic of Tracheo-oesophageal fistula Ans :-AC 120) The following condition characteristically cause jaundice within the first 24 hour of life:- A. G6PD deficiency B. Congenital hypothyroidism C. Sever congenital CMV infection D. Choledochal cyst E. Primary tyrosinaemia Ans:- AC
  • 30. ABDULRAHMAN BASHIRCHILDREN HOSPITAL –BENGHAZI 28 121) Hypothermia can cause the following in LBW infant:- A. Decrease synthesis of surfactant B. Hypernatremia C. Hypoglycemia D. Increased oxygen consumption E. Hypercalcemia Ans :- ACD 122) The following substance are freely transmitted across the placenta:- A. Carbimazole B. Diazepam C. Warfarin D. Pethidine E. IgM antibodies Ans:-ABCD 123) In congenital diaphragmatic hernia:- A. Hernia most commonly occur on the left B. Pulmonary hypoplasia is the major cause of death C. Persistent fetal circulation occurs uncommonly D. Associated congenital anomalies are common E. Most present between 12-24 hours of life Ans:- AB The incidence of CDH is between 1/2,000 and 1/5,000 live births, with females affected twice as often as males. Defects are more common on the left (85%) and are occasionally (<5%) bilateral. Pulmonary hypoplasia and malrotation of the intestine are part of the lesion, not associated anomalies. Most cases of CDH are sporadic, but familial cases have been reported. Associated anomalies have been reported in up to 30% of cases; these include central nervous system lesions, esophageal atresia, omphalocele, and cardiovascular lesions. CDH is recognized as part of several chromosomal syndromes: Trisomy 21, Trisomy 13, Trisomy 18, Fryn, Brachmann–de Lange, Pallister-Killian, and Turner. Relative predictors of a poor prognosis include an associated major anomaly, symptoms before 24 hr of age, severe pulmonary hypoplasia, herniation to the contralateral lung, and the need for ECMO. 124) In the normal newborn infant in the first 24 hour of life:- A. Many significant heart defect may be clinically undetectable B. Normal arterial PaO2 help the ductus to close C. Hypoxia causes pulmonary artery vasoconstriction D. All children with murmur heard in the first 24 hour should be followed up for at least 6 months E. The systolic blood pressure is between 40-80 mmHg Ans:-ABCE 125) A light –for-date full term baby is at particular risk from the following condition:- A. HMD B. Physiological jaundice C. Milk aspiration D. Hypoglycemia
  • 31. ABDULRAHMAN BASHIRCHILDREN HOSPITAL –BENGHAZI 29 E. Apnoeic attacke Ans:- D 126) Recurrent apnoea of prematurity:- A. Characteristically develop within the first 24 hour of life B. Is more likely to occur in infant of < 32 week gestation C. Usually respond to naloxone D. Should be treated with 100% oxygen during attack E. May be accentuated by the presence of nasogastric tube Ans:- BE 127) In the management of HMD: A. Tolazoline may lead to systemic hypertension B. Early use of CPAP may reduce the need for subsequent ventilatory support C. Antibiotic improve ventilation- perfusion ratio D. The illness may be expected to increase in the severity for the first 5 days E. Corticosteroid used postnatally have beneficial effect on the course of the disease Ans:- B 128) Characteristic finding in preterm baby include:- A. Chin reaching only to the tip of shoulder B. Full wrist flexion C. Flat on couch when lying prone D. Incomplete ankle dorsiflexion E. Incomplete knee extension with hip fully flexed Ans:- C 129) Drug effects on the fetus:- A. Isotretinion can lead to CNS defects mainly if given in the third trimester B. Phenytoin can lead to cleft lip, finger and toe abnormalities mainly if given in the second trimester C. Carbimazole cause goiter, mainly if given in the third trimester D. Warfarine can lead to neonatal hemorrhage mainly if given in the first trimester E. Valproate can lead to neural tube defects mainly if given in the third trimester Ans:- C 130) The external criteria for the Dubowitz score for gestational age include:- A. Breast size B. Skin texture C. External genitalia D. Langue hair E. Nose firmness Ans:-ABCD 131) Problem of babies born to mothers with poorly controlled diabetus mellitus: A. Hypermagnesemia B. Polycythemia C. Hypocalcemia D. Sacral agenesis
  • 32. ABDULRAHMAN BASHIRCHILDREN HOSPITAL –BENGHAZI 30 E. Respiratory distress Ans:-BCDE 132) The following are recognised associations of maternal diabetes: A. Sacral agenesis B. Intrauterine growth retardation C. Macrosomia D. Hyaline membrane disease E. Hypertrophic cardiomyopathy Ans:-ABCDE Comments: Overall, malformations occur in 6%, with an increased incidence of cardiac malformations, sacral agenesis, hyperplastic left colon. Intrauterine growth retardation is 3 times as common due to small vessel disease in the mother. More common is macrosomia, and this is related to the degree of maternal hyperglycaemia. The glucose crosses the placenta while the insulin does not, so the fetus increases its production of insulin. This results in increased cell number and size. 25% of IDM are greater than 4kg compared with only 8% of non-diabetics. This gives problems with delivery such as CPD, shoulder dystosia resulting in an increased incidence of birth asphyxia and trauma. In the neonatal period, hypoglycaemia, respiratory distress and reversible hypotrophic cardiomyopathy and polychthaemia are all more common. Gestational diabetes is when carbohydrate intolerance occurs only during pregnancy. It is commonest in obese women and those from Afro-Caribbean and Asian ethnic groups. In these women there is no increase in congenital malformations, thoug macrosomia and other complications remain similar in frequency. 133) Characteristic finding in full term baby are:- A. Full knee extension with hips fully flexed B. Momentary neck extension when held sitting C. Chin reach beyond tip of shoulder D. Hip abducts and legs flat on couch when lying supine E. Full ankle dorsiflexion Ans:- BE 134) The external criteria for the Dubowitz score for gestational age include:- A. Nipple formation B. Ear form & firmness C. Skin colour & opacity D. Protruding tongue E. Planter creases Ans:- ABCE 135) In fetal circulation:- A. 30% of the fetal cardiac output goes through the placenta B. Oxygenated blood from the placenta passes through the ductus arteriosus toward the right atrium C. The oxygen saturation of blood in umbilical arteries is approximately 60%
  • 33. ABDULRAHMAN BASHIRCHILDREN HOSPITAL –BENGHAZI 31 D. Blood entering the heart from the inferior vena cava is diverted directly to the left atrium via patent foramen ovale E. There is one umbilical vein Ans:- CDE 136) The following disease can now be diagnosed prenatally:- A. Phenylketonuria B. Homocystinuria C. Sever combined immunodeficiency D. Fanconi's anaemia E. Lesh-Nyhan syndrome Ans:- ABCDE 137) Resuscitation of neonate with coarctation of aorta may require the use of :- A. Frusemide B. Bicarbonate C. Indomethacin D. Dopamine E. Prostaglandin E Ans:- ABDE 138) Recognized causes of the floppy baby include:- A. Trisomy 21 B. Zellweger syndrome C. Becker muscular dystrophy D. Spinal muscular atrophy E. Hypothyroidism Ans:- ABDE 139) Which of the following may cause apnoea in preterm infant A. Hypocalcemia B. Hypoglycemia C. RSV infection D. Caffeine E. Intraventricular hemorrahage Ans:- ABCE 140) Failure of resuscitation of newborn may be due to :- A. PDA B. VSD C. Congenital diaphragmatic hernia D. Pulmonary hypoplasia E. HMD Ans:-CD 141) Feature of IUGR include:- A. Neutropenia B. Hypoglycemia C. Necrotizing Enterocolitis D. Weight loss > 10% of birth weight in first week E. Thermal instability
  • 34. ABDULRAHMAN BASHIRCHILDREN HOSPITAL –BENGHAZI 32 Ans:- ABCE 142) Newborn infant has breathing difficulty with central cyanosis .Nitrogen washout test raised the PO2 to 15 kPa possible diagnosis include:- A. Fallot's tetralogy B. VSD C. Pneumonia D. Tricuspid atresia E. HMD Ans:- CE 143) Newborn infant has central cyanosis and fit with slow & shallow breathing , Nitogen washout test produce slight raise in the PO2 , the most likely cause of cyanosis is:- A. Methemoglobinemia B. Cerebral disorder C. Persistent fetal circulation D. Congenital cyanotic heart disease E. Lung disease Ans:- B 144) The following disease can now be diagnosed prenatally:- A. Hypercholesterolemia B. Polycystic kidney disease C. Cystinosis D. Gaucher's disease E. Beta thalassaemia Ans:-ABCDE 145) Common causes of seizure in the neonatal period include:- A. Intracranial hemorrhage B. Electrolyte disturbance C. Infection D. Drug withdrawal E. Febrile convulsion Ans:- ABCD 146) Neonatal convulsion may be caused by :- A. Hypomagnesaemia B. Hyperkalamia C. Pyridoxine dependency D. Cephalohematoma E. HSV infection Ans:- ACE 147) The following disease can now be diagnosed prenatally:- A. Sickle cell disease B. Maple syrup urine disease C. Retinoblastoma D. Hypophosphaturia E. Von-Willebrand disease
  • 35. ABDULRAHMAN BASHIRCHILDREN HOSPITAL –BENGHAZI 33 Ans:- ABCDE 148) Which of the following statement regarding surfactant therapy is correct :- A. The incidence of Pneumothorax is reduced B. The incidence of intraventricular hemorrhage is reduced C. Early therapy ( within 4 hour) is more beneficial than later administration ( after 12 hour) D. Prolonged courses of surfactant therapy confer advantages E. The incidence of chronic lung disease is increased Ans:- ABC 149) Which of the following statement regarding periventricular hemorrhage are correct: A. Most hemorrhage occur after the fourth day of life B. Over 80% of hemorrhage progress to ventricular dilatation C. Most cases of posthemorrhagic hydrocephalous are communicating D. Early ventricular tapping improve the neurological outcome E. Hemorrhage is usually caused by hypoglycemia Ans:- C 150) Complication of phototherapy include:- A. Diarrhea B. Erythematosus rash C. Hyponatremia D. Skin discoloration E. Peripheral desqumation Ans:- ABD 151) Which of the following are characteristic haemodynamic changes encountered at birth:- A. Increase the pulmonary vascular pressure B. Right to left flow through the ductus arteriosus C. Closure of the ductus arteriosus D. Increased right ventricular end diastolic pressure E. Reversal of flow across the foramen ovale Ans:- C 152) Feature suggestive of TEF with oesophageal atresia include:- A. FTT B. Recurrent pneumonia C. Oligohydramniose D. Large amount of mucus in the pharynx at delivary E. Slow to establish feed Ans:- D 153) The development of Pneumothorax is associated with :- A. Surfactant therapy B. Artificial paralysis C. Patient triggered ventilation D. Meconium aspiration E. Pulmonary interstitial emphysema Ans:- DE
  • 36. ABDULRAHMAN BASHIRCHILDREN HOSPITAL –BENGHAZI 34 154) Cyanotic congenital heart disease in the newborn include:- A. TGA B. Pulmonary stenosis C. Fallot's tetrallogy D. VSD E. Tricuspid atresia Ans:- AE 155) The following congenital disorder require immediate ( within first week) treatment after birth:- A. Hemangioma B. Imperforate anus C. Choanal atresia D. Hypospadius E. Congenital diaphragmatic hernia Ans:-BCE 156) Causes of neonatal hypoglycemia include;- A. Erythroblastosis fetalis B. Glycogen storage disease type I C. Maternal treatment with sodium valproate D. Galactosemia E. Congenital adrenal hypoplasia Ans:- ABDE Plasma glucose level of <40 mg/dL, During gestation, glucose is freely transferred across the placenta by the process of facilitated diffusion. However, after birth, the infant must adjust to the sudden withdrawal of this transplacental supply. In all infants, there is a nadir in blood sugar between 1 and 3 hours of life. During the first 12-24 hours of life, newborns are at increased risk for hypoglycemia because gluconeogenesis and especially ketogenesis are incompletely developed. These factors are accentuated in preterm infants, infants of diabetic mothers, infants with erythroblastosis fetalis, asphyxiated infants, and infants who are small or large for gestational age. Transient hypoglycemia causes:- prematurity, hypothermia, Birth asphyxia, sepsis, erythroblastosis fetalis, infants with Beckwith-Wiedemann syndrome, maternal diabetes, maternal glucose infusion in labor, and intrauterine growth restriction (IUGR)and maternal drugs( b sympathomimetic and chlorpropamide). Persistent hypoglycemia may be due to: - inborn error of metabolism (Glycogen storage disease type I , P & IV, maple syrup urine disease, Nesidioblastosis, galactosemia,and mitochondrial fatty acid oxidation defects such as MCAD deficiency. Hormonal deficiency suc as congenital hypopituitrism,congenital glucagons deficiency and cortisol deficiency states including congenital adrenal hypoplasia. Features on physical examination suggest the etiology of hypoglycemia:- · Macrosomia: This occurs in infants of diabetic mothers, infants with severe congenital hyperinsulinism, and infants with Beckwith-Wiedemann syndrome; recall that insulin is a growth factor and that hyperinsulinism leads to macrosomia. · Midline defects: Congenital pituitary deficiency can be associated with midline defects such as cleft lip, cleft palate, single central incisor, and micro-ophthalmia.
  • 37. ABDULRAHMAN BASHIRCHILDREN HOSPITAL –BENGHAZI 35 · Micropenis: Congenital gonadotropin deficiency and possible pituitary abnormalities cause this condition. · Hepatomegaly: This is associated with glycogen storage diseases and fatty acid oxidation disorders. 157) Healthy term neonate differ from adult in the following way:- A. Less complement B. Decrease IgG level C. Fewer B lymphocytes D. Lower level of secretory IgA E. Higher level of C reactive protein Ans:- AD 158) Necrotizing enterocloitis is recognized complication of:- A. Polycythemia B. Umbilical arterial catheterization C. Parenteral feeding D. Maternal Crohn's disease E. Birth asphyxia in term neonate Ans:- ABE 159) NEC is associated with:- A. Epidemic B. Thrombocytopenia C. Malabsorption D. Bile stained aspirate E. Air in the portal tree on abdominal X –ray Ans:- ABCDE 160) The presenting feature of NEC include:- A. Apnoea B. Bradycardia C. Abdominal distension D. Intramural gas on abdominal X ray E. Bloody stool Ans:- ABCDE 161) A 3-week-old breastfed infant presents to the emergency department with irritability, fever, jaundice, and hepatomegaly. A laboratory evaluation shows a normal complete blood count and a bilirubin concentration of 6.5 mg/dL (111.2 mcmol/L). A urinalysis is positive for reducing substances. A blood culture is positive for Escherichia coli. You initiate antibiotic therapy Of the following, the MOST appropriate dietary management of this patient is to A. continue breastfeeding B. switch to a cow milk-based formula C. switch to a soy-based formula D. switch to a whey hydrolysate formula E. switch to an elemental formula Preferred Response: C
  • 38. ABDULRAHMAN BASHIRCHILDREN HOSPITAL –BENGHAZI 36 The clinical features of jaundice, hepatomegaly, and invasive Escherichia coli infection described for the neonate in the vignette suggest the possible diagnosis of galactosemia. Galactosemia is an autosomal recessive disorder most commonly caused by a deficiency of the enzyme galactose-1-phosphate uridyltransferase. The reducing substances in the urine represent the accumulation of galactose. In addition to recognizing and treating the gram-negative infection in the infant, it is important to remove lactose, which is comprised of glucose and galactose, from the diet as soon as the diagnosis is suspected. Soy protein formulas are the first choice of nutrition for infants who have suspected or proven galactosemia because the carbohydrate source in these formulas is sucrose or corn syrup rather than lactose. Protein hydrolysate and elemental formulas also contain other carbohydrates than lactose, but they can be more expensive and less readily available than soy formulas. Lactose is the primary carbohydrate in human milk, cow milk-based formulas, and most whey hydrolysate formulas. 162) Hydrops fetalis may be associated with :- A. Rhesus isoimmunization B. Paroxysmal supraventricular tachycardia C. CMV infection D. Achondroplasia E. Renal vein thrombosis Ans:- ABCDE 163) The following drugs are correctly paired with their potential teratogenic effect:- A. Alcohol & macrocephaly with congenital heart disease B. Phenytoin & meningomyelocele C. Isotretinoic acid & cutis laxis syndrome D. Penicillamine & facial abnormalities with pinna defects E. Lithium & Ebtein's anomaly Ans:-BCDE Warfarine Hypoplastic nasal bridge ,chrondroplasi puncta Isotretinoic acid facial , ear , cardiovascular abnormality Phenytoin Hypoplastic nail, IUGR, typical facies & may be associated with neural tube defect Tetracyclin Enamel hypoplasia Sodium Valproate Neural tube defect 164) The neonate of mother with SLE may demonstrate:- A. Polythycemia B. Rash C. Neutropenia D. Atrial fibrillation E. Anti-Ro antibodies Ans:- BCE 165) Pulmonary hypoplasia is consequence of :-
  • 39. ABDULRAHMAN BASHIRCHILDREN HOSPITAL –BENGHAZI 37 A. Congenital varicella zoster B. Anencephaly C. Posterior urethral valve D. Congenital diaphragmatic hernia E. Exomphalos Ans:- BCDE 166) The following conditions may present with bile-stained vomiting in the first week of life:- A. Duodenal atresia B. Cystic fibrosis C. Inguinal hernia D. GER E. NEC Ans:- ABCE 167) Renal immaturity in normal neonate born at term is manifested as:- A. Reduced number of nephrons B. Decreased glucose reabsorption C. Increased glomerular filteration rate D. Decreased renal bicarbonate reabsorption E. Decreased urea excretion Ans:- BDE 168) The following maternal factors increase the incidence of surfactant deficient RDS:- A. Steroid therapy B. Opiate C. Placental insufficiency leading to IUGR D. Diabetus E. Alcoholism Ans:- D 169) Bacteria commonly isolated in case of neonatal meningitis include:- A. Escherichia coli B. Haemophilus infuenzae C. Group B streptococcus D. Staphylococcu epidermidis E. Neisseria meningitis Ans:- AC 170) In the infant of diabetic mother :- A. The infants brain size is increased beyond normal B. The infant's liver size is increased beyond normal C. He can be small for gestational age D. There is an increased incidence of polycythemia E. He has an increased incidence of hypertrophic Cardiomyopathy Ans:- BCDE 171) Galactosaemia:- A. Is caused by deficiency of the enzyme galactokinase
  • 40. ABDULRAHMAN BASHIRCHILDREN HOSPITAL –BENGHAZI 38 B. Causes jaundice in the newborn C. May present with cataract at birth D. Is associated with Escherichia coli septicemia E. Is diagnosed as result of screening in the majority of cases. Ans:- BCD 172) SVT in neonate:- A. Is the most common abnormal tachycardia B. Reflect underlying congenital heart disease in the majority of cases C. Show regular rate of 160-220 beats /min on the ECG D. Recurrent episodes usually persist into adulthood E. May be stopped with rapid I/V bolus of adenosine Ans:- A E 173) Neonates suffering withdrawal from in utero exposure to narcotics may show sign of A. Irritability B. Vomiting C. Photophobia D. Hypotonia E. Diarrhea Ans:- ABCE 174) Clinical feature of congenital hypothyroidism diagnosed in the newborn period include:- A. Large tongue B. Presence of third fontenalle C. Umbilical hernia D. Loose stools E. High incidence of mental retardation Ans:- A C 175) Causes of persistent neonatal unconjugated hyperbilirubinemia after 2 weeks include:- A. Rhesus incompatibility B. Hypothyroidism C. Breast milk jaundice D. Rotor syndrome E. Sepsis Ans:- ABCE 176) Growth retardation babies are at increased risk of:- A. Polycythemia B. HMD C. Hypoglycemia D. Group B streptococcal infection E. Sudden intrapartum death Ans:- ACE 177) Oesophageal atresia is associated with:- A. Maternal polyhydramniose B. Vertebral anomalies C. Diaphragmatic hernia
  • 41. ABDULRAHMAN BASHIRCHILDREN HOSPITAL –BENGHAZI 39 D. Low birthweight E. Duplex collecting system Ans:- ABDE 178) Congenital Rubella:- A. Is associated with cerebral calcification B. Frequently lead to cataract C. Is associated with VSD D. Rarely occur following maternal infection in the third trimester E. Should be prevented by vaccinating women found to be seronegative during the first trimester Ans:- B D 179) Human breast milk contain:- A. Secretory IgA B. Macrophages C. Lysozyme D. Vitamin c E. Zinc Ans:- ABCDE 180) The following are recommended daily requirement for 1 month old child /hg of B wt:- A. Fluids 150-180 ml orally B. Calories 90-115 kcal C. Sodium 1.25-2.5 mmol D. Protein 2.2-3.5 gm E. Potassium 2.0-3.5 mmol Ans:- ABCDE 181) Compared with cow's milk, human breast milk contains: A. Less sodium. B. Less calcium. C. Less protein. D. Less fat. E. Less carbohydrate. Ans:-A B C 182) UK infants are routinely immunised against: A. Meningococcus type B. B. Diphtheria. C. Cholera. D. Polio.Pertussis. Ans:-B D E 183) Mature breast milk contain per 100 ml;- A. 60-75 kcal B. 8-10 gm of protein C. 0.1-0.5 gm of fat D. 0.1-0.5 gm of carbohydrate
  • 42. ABDULRAHMAN BASHIRCHILDREN HOSPITAL –BENGHAZI 40 E. 5.2 mg of folic acid Ans:- AE MATURE BREAST MILK COWS MILK carbohydrate 7.4 gm 4.6 gm Fat 4.2 gm 3.9 gm protein 1.1 gm 3.4 gm Calories 70 67 folic acid 5.2 mg 3.7 mg 184) Treatment of PDA with Indomethacin:- A. Should be first line therapy B. Is safe in renal failure C. Should not be given if there is thrombocythenia D. Should be avoided in jaundiced babies E. Should not be given in the presence of IVH Ans:- D E 185) Retinopathy of prematurity ;- A. Develop in the first week of life B. Is more likely to occur in VLBW infants C. Is recognized complication of hypoglycemia D. Rarely resolve spontaneously E. May be treated effectively with Laser therapy Ans:- B E 186) Alpha fetoprotein level from maternal serum may be helpful in diagnosing:- A. Congenital cardiac malformation B. Prune-belly syndrome C. Cleft lip & palate D. Down syndrome E. Fetal alcohol syndrome Ans:- D 187) A 2-week-old infant is jaundiced. Findings include weight and length at the 75th percentile for age; icterus; with hepatosplenomegaly; total bilirubin, 6.3 mg/dL; direct bilirubin, 5.5 mg/dL; alanine aminotransferase activity, 130 U/L; aspartate aminotransferase activity, 143 U/L; and gamma-glutamyl transpeptidase activity, 950 U/L. Of the following, the BEST study to evaluate the excretion of bile from the liver is A. computed tomography of the liver B. hepatic ultrasonography C. hepatobiliary scintigraphy D. measurement of galactose-1-phosphate uridyltransferase activity E. measurement of the serum alpha1-antitrypsin level Ans:- C
  • 43. ABDULRAHMAN BASHIRCHILDREN HOSPITAL –BENGHAZI 1 188) Which of the following constellations of features BEST describes the fetal alcohol syndrome? A. Elfin facies, irritability, and supravalvular aortic stenosis B. Growth deficiency with microcephaly, developmental delay, and short palpebral fissures C. Intrauterine growth retardation, triangular-shaped face, and clinodactyly of the fifth finger D. Short stature, webbed neck, and pulmonic stenosis E. Weakness, club feet, immobile face, and inadequate respirations Ans:- B 189) A female infant born to a 24-year-old woman has been diagnosed clinically as having Down syndrome. The mother is concerned about her risk of having another child who has a chromosomal abnormalityThe statement that you are MOST likely to include in your discussion is that her risk A. can be estimated by determination of maternal serum alpha-fetoprotein in all future pregnancies B. cannot be estimated until her infant's chromosome complement has been determined C. is increased for Down syndrome, but not for any other chromosomal abnormality D. is no greater than that of any other woman her age E. is not increased until she reaches the age of 35 Ans:- B 190) During delivery of an infant who has an estimated gestational age of 42 weeks, you note that the amniotic fluid looks like pea soup and contains thick particles of meconium. Of the following, the MOST important initial step in resuscitation of the infant is to :- A. aspirate the gastric contents B. determine the Apgar score initiate tracheal intubation C. provide positive pressure ventilation D. suction the hypopharynx Ans:- D 191) In addition to irritability, sweating, and difficulty breathing with feeding, the symptom that is MOST indicative of congestive heart failure in a 3-week-old infant is A. ascites B. cough C. cyanosis D. diminished feeding volume E. pretibial edema Ans:- D 192) The decreased incidence of enteric infections noted in breastfed infants compared with formula-fed infants is MOST likely due to the A. more alkaline stool pH in breastfed infants B. nutritional benefits of human milk on the infant's immune system C. predominance of Bacteroides and Clostridium in the gut of breastfed infants D. presence of protective antibodies against enteric infection in human milk
  • 44. ABDULRAHMAN BASHIRCHILDREN HOSPITAL –BENGHAZI 42 E. sterility of human milk Ans:- D 193) A newborn who weighs 600 g and whose estimated gestational age is 24 weeks at birth is admitted to the neonatal intensive care unit after successful resuscitation in the delivery room. Arterial blood gas measurements on room air are: pH, 7.35; PCO2, 42 mm Hg; PO2, 68 mm Hg; base deficit, 2 mEq/L. Of the following, the MOST appropriate initial management is to A. begin intravenous vancomycin B. begin phototherapy C. initiate enteral feeding D. provide bicarbonate infusion E. provide glucose infusion Ans:- E 194) A 3-day-old infant presents to the emergency department with vomiting, lethargy, hypotonia, and jaundice. Physical examination reveals hepatomegaly and neurologic depression. A full sepsis evaluation is undertaken, and the Gram stain of the cerebrospinal fluid reveals gram-negative organisms. Of the following, the BEST additional laboratory test to obtain is A. erythrocyte galactose-1-phosphate B. liver glycogen content C. plasma insulin level D. plasma very long-chain fatty acids. E. stool porphyrins Ans:- A 195) You are examining a term newborn in the nursery. His weight is 3.27 kg (50th percentile), and his length is 50.5 cm (50th percentile). The pregnancy, labor, and delivery were unremarkable. There are no significant findings on physical examination. The MOST likely head circumference in this child, if it is consistent with his other growth parameters, is A. 31 cm B. 33 cm C. 35 cm D. 37 cm E. 39 cm Ans:- D 196) previously healthy 5-day-old male who was born at home develops bruising and melena. The pregnancy, delivery, and postnatal course were unremarkable. The infant is breastfeeding vigorously every 2 hours. Findings on physical examination are unremarkable except for several large bruises. Laboratory testing reveals: hemoglobin, 81 g/L (8.1 g/dL); white blood cell count, 9.4 x 109/L (9,400/mm3); prothrombin time, 37 seconds; partial thromboplastin time, 98 seconds; platelet count, 242 x 109/L (242,000/mm3); and fibrinogen, 2.34 g/L (234 mg/dL). Of the following, the MOST likely cause of the bleeding is A. disseminated intravascular coagulation B. factor VIII deficiency hemophilia C. liver disease D. vitamin K deficiency
  • 45. ABDULRAHMAN BASHIRCHILDREN HOSPITAL –BENGHAZI 43 E. von Willebrand disease Ans:- D 197) A newborn whose estimated gestational age is 42 weeks is stained with meconium. Tracheal intubation reveals meconium in the hypopharynx as well as below the vocal cords. The infant has respiratory distress. A chest radiograph is obtained. Of the following, the MOST likely radiographic finding is A. coarse infiltrates B. decreased lung volumes C. mediastinal shift D. pleural effusion E. reticulogranular pattern Ans:- A 198) A 20-year-old primigravida at 30 weeks of gestation has a blood pressure of 160/112 mm Hg, serum total bilirubin level of 44.5 mcmol/L (2.6 mg/dL), serum alanine aminotransferase level of 150 U/L, and platelet count of 75 x 109/L (75,000/mm3). She is hospitalized for observation and electronic fetal heart rate monitoring. Of the following, the MOST ominous sign of fetal distress during monitoring would be A. early decelerations B. increased beat-to-beat variability C. late decelerations D. spontaneous accelerations E. variable decelerations Ans:- C 199) A term newborn presents with bilious vomiting shortly after birth. Her abdomen is distended slightly, and facial features are characteristic of Down syndrome. She has passed a normal meconium stool. The pregnancy was complicated by polyhydramnios. Of the following, the MOST likely diagnosis is A. duodenal atresia B. Hirschsprung disease C. meconium ileus D. midgut volvulus E. pyloric stenosis Ans:- A 200) An 18-year-old primigravida at 32 weeks' gestation has a blood pressure of 148/96 mm Hg, proteinuria, oliguria, and visual disturbances. Labor is induced, and the infant is delivered. His weight is 850 g (<10th percentile), crown-heel length is 38 cm (10th percentile), and head circumference is 30 cm (50th percentile). Of the following, the MOST likely complication in this infant is A. anemia of prematurity B. hyaline membrane disease C. hyperglycemia D. meconium aspiration E. perinatal asphyxia Ans:- E
  • 46. ABDULRAHMAN BASHIRCHILDREN HOSPITAL –BENGHAZI 44 201) An infant is born at 27 weeks' gestation following a pregnancy complicated by preterm labor that progressed despite administration of a tocolytic agent. Of the following, the most appropriate INITIAL management is to A. measure transcutaneous oxygen saturation B. perform endotracheal intubation C. place an umbilical arterial catheter D. place the infant in an open bed warmer E. provide nasal continuous positive airway pressure Ans:- D 202) Of the following, the MOST important determinant of neurodevelopmental outcome of VLBW infants is A. antenatal obstetric management B. infant gender C. length of gestation D. maternal education E. socioeconomic status Ans:- C 203) You are called to the newborn nursery to examine an infant who appears dysmorphic. On physical examination, the baby is normally grown and vigorous. You note overfolded pinnae, deviation of the nose to one side (Item Q25A), and a small chin. The feet are maintained in dorsiflexion (Item Q25B), but can be corrected passively. You review the pregnancy history. Of the following pregnancy complications, the one MOST likely to be associated with this baby’s features is A. maternal hypertension B. polyhydramnios C. preeclampsia D. transverse lie E. vaginal bleeding Preferred Response: D The infant described in the vignette displays the characteristic features of a deformation sequence. Unlike malformations, which occur due to intrinsic problems within a developing structure, deformations are due to mechanical forces acting on an otherwise normally developing embryo or fetus. Causes of fetal deformation are many and include oligohydramnios, prolonged breech positioning, a small or malformed uterus, fibroid tumors of the uterus, and multiple gestations. Often, the affected infant has a pugilistic facies, with deviation of the nose to one side. Limb positioning defects are common. Barring any association with malformations or disruptions, the appearance of the affected infant typically normalizes over time. Although maternal hypertension and preeclampsia can be associated with placental insufficiency and, ultimately, decreased fetal movement, typically they are not associated with fetal deformation. Polyhydramnios usually is associated with unrestricted fetal movement, and fetuses are not deformed. Vaginal bleeding has multiple potential causes, most of which are not associated with deformation.
  • 47. ABDULRAHMAN BASHIRCHILDREN HOSPITAL –BENGHAZI 45 Transverse lie is the only pregnancy complication listed that is likely to be associated with deformation due to unusual fetal position 204) A 10-day-old infant who weighed 1,750 g at birth and whose gestational age was 34 weeks is jaundiced. His total serum bilirubin concentration is 10.0 mg/dL and the direct fraction is 0.8 mg/dL. He is receiving intermittent orogastric feeding of expressed human milk and supplemental parenteral nutrition. Of the following, the MOST likely explanation for these findings is A. Crigler-Najjar syndrome B. jaundice due to parenteral nutrition C. neonatal hepatitis D. physiologic jaundice E. pyloric stenosis Ans:- D 205) Early hospital discharge is defined as the discharge of a newborn earlier than 48 hours following vaginal delivery or 96 hours following cesarean delivery. Of the following, the MOST common reason for readmission to the hospital within 7 days following an early discharge is A. bacterial sepsis B. congenital heart disease C. gastrointestinal malformation D. hyperbilirubinemia E. metabolic disorders Ans:- D 206) A newborn infant is delivered by emergent cesarean section at 41 weeks' gestation following a pregnancy complicated by a prolapsed umbilical cord and meconium-stained amniotic fluid. At 6 hours of age, the infant has a generalized tonic-clonic seizure. Of the following, the MOST likely explanation for this seizure is A. hyponatremia B. hypoxic-ischemic encephalopathy C. intracranial hemorrhage D. meningitis E. pyridoxine dependency Ans:- B 207) A 4-hour-old newborn has copious oral secretions and episodes of coughing, choking, and cyanosis. The pregnancy was complicated by polyhydramnios. You suspect esophageal atresia with tracheoesophageal fistula. Of the following, the MOST helpful test to confirm the diagnosis is to A. inject a contrast medium through an orogastric catheter and obtain a neck radiograph B. obtain computed tomography of the neck C. perform flexible bronchoscopy D. place an endotracheal tube and examine the endotracheal fluid E. place an orogastric suction catheter and obtain a chest radiograph Ans:- E
  • 48. ABDULRAHMAN BASHIRCHILDREN HOSPITAL –BENGHAZI 46 208) A newborn is delivered by emergent cesarean section because of fetal distress following acute abruption of the placenta. The infant is resuscitated and transferred to the nursery. On physical examination, she appears pale, and her extremities are cold to touch. The capillary refill is 8 seconds. Results of an arterial blood gas analysis show a Po2 of 48 mm Hg. Of the following, the MOST likely additional finding is A. decreased bicarbonate concentration B. hypercalcemia C. hyperglycemia D. hyponatremia E. normal anion gap Ans:- A 209) Of the following, erythromycin prophylaxis is MOST likely to prevent ocular infection due to A. Chlamydia trachomatis B. group B streptococci C. Neisseria gonorrhoeae D. Staphylococcus aureus E. Trichomonas vaginalis Ans:- C 210) A 1,300 g infant who is born at 34 weeks' gestation has a head circumference of 27 cm and crown-heel length of 40 cm. At 48 hours of age, she is irritable, tremulous, and inconsolable. Her systolic blood pressure is 65 mm Hg and heart rate is 180 beats/min. Her face appears normal, and her cry is high-pitched. Cranial ultrasonography reveals bilateral echo densities suggestive of periventricular leukomalacia. Of the following, the MOST likely explanation for the findings in this infant is maternal exposure to A. alcohol B. barbiturates C. cocaine D. marijuana E. opiates Ans:- C 211) A 4.3 kg infant is delivered to a woman whose diabetes mellitus is poorly controlled. Of the following, the MOST likely neonatal manifestation of maternal diabetes is A. diabetic ketoacidosis B. Hirschsprung disease C. hypercalcemia D. polycythemia E. renal vein thrombosis Ans:- D 212) Of the following, the MOST helpful finding to distinguish GBS pneumonia from RDS is A. a normal C-reactive protein level B. an elevated erythrocyte sedimentation rate C. diffuse alveolar infiltrates on chest radiography D. increased ratio of bands to segmented neutrophils
  • 49. ABDULRAHMAN BASHIRCHILDREN HOSPITAL –BENGHAZI 47 E. persistent hypoxemia on blood gas analysis Ans:- D 213) An 18-hour-old infant of a diabetic mother develops abdominal distension. Physical examination reveals a protuberant, firm, but nontender abdomen; patent anus; and no grossly visible anomalies. The infant has passed no meconium stool since birth. A supine abdominal radiograph reveals multiple dilated loops of intestine. Of the following, a contrast enema would MOST likely confirm a diagnosis of A. atresia of the colon B. Hirschsprung disease C. hypoplastic left colon syndrome D. meconium ileus E. midgut volvulus with malrotation Ans:- C 214) You are attending the emergency delivery by cesarean section of a primiparous woman. The gestation was complicated by pregnancy-induced hypertension. Deep variable fetal heart rate decelerations were noted during labor. At delivery, the infant is acrocyanotic with poor tone; spontaneous movement and minimal respiratory effort are present. Of the following, your INITIAL management is to A. ascertain the heart rate and assign a 1-minute Apgar score B. begin tactile stimulation and provide blow-by oxygen supplementation C. dry all skin surfaces and clear the oropharynx D. initiate bag-mask ventilation E. insert an umbilical catheter and administer naloxone Ans:-C 215) A vigorous, normal-appearing term male newborn has not voided by 18 hours after delivery. Perinatal history is negative for maternal illness or medications. Amniotic fluid volume was reportedly normal, and the delivery was uneventful, with Apgar scores of 6 and 9 at 1 and 5 minutes, respectively. Of the following, the MOST likely reason why this 18- hour-old infant has not voided is A. bilateral ureteropelvic junction obstruction B. intravascular volume depletion C. neurogenic bladder D. posterior urethral valve E. undocumented void in the delivery room Ans:- E 216) A 2-week-old neonate who was born at 32 weeks’ gestation has recovered from respiratory distress syndrome. He has been tolerating increasing volumes of enteral feedings via gavage. Over the past several feedings, abdominal distension, gastric residuals, and stools that are positive for blood have been noted. Of the following, the radiographic finding MOST supportive of the diagnosis of necrotizing enterocolitis is A. absence of luminal bowel gas B. generalized bowel distension C. intraperitoneal fluid D. pneumatosis intestinalis E. thickening of the bowel wall
  • 50. ABDULRAHMAN BASHIRCHILDREN HOSPITAL –BENGHAZI 48 Ans:- D 217) A term infant is placed under a radiant warmer, the skin is dried, and the oropharynx and nose are suctioned. After tactile stimulation, there is minimal respiratory effort, dusky color, and a heart rate of 86 beats/min. Bag/mask ventilation is performed for 30 seconds with 100% oxygen at a rate of 40 to 60 breaths/min. The heart rate increases to 100 beats/min. Of the following, the NEXT best step is to: A. administer sodium bicarbonate B. continue bag/mask ventilation at a rate of 20 to 40 breaths/min C .continue ventilation and begin chest compressions D. observes for spontaneous respiration and discontinues ventilation E. perform endotracheal intubation Ans:-D 218) A 900 g male infant is delivered vaginally to a woman who had no prenatal care. Of the following, the physical finding that is MOST consistent with prematurity rather than intrauterine growth restriction is A. creases over entire sole of foot B. descended testes with deep rugae of the scrotum C. formed and firm pinna with instant recoil D. gelatinous translucent skin E. raised areola and 3 mm breast buds Ans:-D 219) A 1-day-old infant develops bilious vomiting and gastric distension. She has been afebrile and has been passing meconium-laden stools. Of the following, the most appropriate INITIAL step in the management of this infant is A. abdominal radiography to look for the “double-bubble” sign B .culture of a catheterized urine specimen C .insertion of a rectal tube for decompression D .placement of a nasogastric tube and initiation of intravenous fluid therapy E. upper gastrointestinal radiographic series to look for malrotation of the small bowel Ans:- D 220) While performing ultrasonography on a 31-week fetus, an obstetrician notes that the fetal heart rate ranges from 62 to 66 beats/min. Fetal growth appears normal, and no structural cardiac anomalies are identified. Fetal echocardiography reveals that the fetal atria appear to be contracting at 140 beats/min, with a ventricular rate of 65 beats/min. Of the following, the NEXT step in the management of this infant is to:- A. administer beta-agonist drug therapy to the mother B. assess the cardiac status of the infant following labor and delivery C. counsel the parents that intrauterine fetal death is likely D. perform amniocentesis to confirm lung maturity and if mature, perform immediate cesarean section E. repeat the fetal echocardiography and fetal ultrasonography in 1 week Ans:-E 221) The mother of a 2-week-old infant is concerned because her baby has not had a stool in 7 days. She has been exclusively breastfeeding him every 2 to 3 hours since her milk came in, but the
  • 51. ABDULRAHMAN BASHIRCHILDREN HOSPITAL –BENGHAZI 49 baby has only passed flatus. The infant is gaining weight well. Results of physical examination are normal. Of the following, the best course of INITIAL management for this infant is…………………………………… 222) Term infant is cyanotic and requires intubation. Findings include: heart rate, 175 beats/min; blood pressure, 60/30 mm Hg; increased right ventricular activity; single S2; short systolic murmur; and equal arm and leg pulses; chest radiography, normal heart size and pulmonary congestion. Arterial blood gases (right radial artery on 100% FIO2): pH, 7.31; PO2, 43 torr; PCO2, 48 torr. Of the following, the MOST likely diagnosis is:- A. hyaline membrane disease B. hypoplastic left heart C. intrauterine constriction of the ductus arteriosus D. tetralogy of Fallot E. total anomalous pulmonary venous connection Ans:-E 223) A 7-day-old infant has copious purulent discharge from both eyes. The 17-year-old mother currently complains of a yellowish vaginal discharge. The only medications received by the infant were vitamin K and topical erythromycin prophylaxis following delivery. Giemsa stain of a conjunctival scraping reveals intracytoplasmic inclusions. After obtaining appropriate diagnostic studies, the BEST management includes treatment with :- A. oral erythromycin B. oral penicillin C. topical erythromycin D. topical gentamicin E. topical sulfonamide Ans:-A 224) Of the following, the condition that is MOST likely to present with seizures during the first 24 hours of life is A .fetal alcohol syndrome B .herpes simplex infection C. hypoxic-ischemic encephalopathy D. organic acidemia E. urea cycle defect Ans:-C 225) Maternal serum alpha fetoprotein measured at 12 weeks gestation A. If normal then neural tube defect are excluded B. Is increased in twin pregnancy C. Is reduced in Turner syndrome D. Is increased in Down syndrome E. AFT is the main blood protein fraction in the first trimester Ans :- BE Serum a fetoprotein is usually measured at 16-18 week gestation, normal screening would not rule out neural tube defect or any other lesion such as twins, Turner's syndrome, Exomphalus, which can all have raised value .